1 Star 0 Fork 0

sierxue/sc4f

加入 Gitee
与超过 1200万 开发者一起发现、参与优秀开源项目,私有仓库也完全免费 :)
免费加入
该仓库未声明开源许可证文件(LICENSE),使用请关注具体项目描述及其代码上游依赖。
克隆/下载
sc4f410.tex 199.89 KB
一键复制 编辑 原始数据 按行查看 历史
sierxue 提交于 2022-06-27 18:40 . update
12345678910111213141516171819202122232425262728293031323334353637383940414243444546474849505152535455565758596061626364656667686970717273747576777879808182838485868788899091929394959697989910010110210310410510610710810911011111211311411511611711811912012112212312412512612712812913013113213313413513613713813914014114214314414514614714814915015115215315415515615715815916016116216316416516616716816917017117217317417517617717817918018118218318418518618718818919019119219319419519619719819920020120220320420520620720820921021121221321421521621721821922022122222322422522622722822923023123223323423523623723823924024124224324424524624724824925025125225325425525625725825926026126226326426526626726826927027127227327427527627727827928028128228328428528628728828929029129229329429529629729829930030130230330430530630730830931031131231331431531631731831932032132232332432532632732832933033133233333433533633733833934034134234334434534634734834935035135235335435535635735835936036136236336436536636736836937037137237337437537637737837938038138238338438538638738838939039139239339439539639739839940040140240340440540640740840941041141241341441541641741841942042142242342442542642742842943043143243343443543643743843944044144244344444544644744844945045145245345445545645745845946046146246346446546646746846947047147247347447547647747847948048148248348448548648748848949049149249349449549649749849950050150250350450550650750850951051151251351451551651751851952052152252352452552652752852953053153253353453553653753853954054154254354454554654754854955055155255355455555655755855956056156256356456556656756856957057157257357457557657757857958058158258358458558658758858959059159259359459559659759859960060160260360460560660760860961061161261361461561661761861962062162262362462562662762862963063163263363463563663763863964064164264364464564664764864965065165265365465565665765865966066166266366466566666766866967067167267367467567667767867968068168268368468568668768868969069169269369469569669769869970070170270370470570670770870971071171271371471571671771871972072172272372472572672772872973073173273373473573673773873974074174274374474574674774874975075175275375475575675775875976076176276376476576676776876977077177277377477577677777877978078178278378478578678778878979079179279379479579679779879980080180280380480580680780880981081181281381481581681781881982082182282382482582682782882983083183283383483583683783883984084184284384484584684784884985085185285385485585685785885986086186286386486586686786886987087187287387487587687787887988088188288388488588688788888989089189289389489589689789889990090190290390490590690790890991091191291391491591691791891992092192292392492592692792892993093193293393493593693793893994094194294394494594694794894995095195295395495595695795895996096196296396496596696796896997097197297397497597697797897998098198298398498598698798898999099199299399499599699799899910001001100210031004100510061007100810091010101110121013101410151016101710181019102010211022102310241025102610271028102910301031103210331034103510361037103810391040104110421043104410451046104710481049105010511052105310541055105610571058105910601061106210631064106510661067106810691070107110721073107410751076107710781079108010811082108310841085108610871088108910901091109210931094109510961097109810991100110111021103110411051106110711081109111011111112111311141115111611171118111911201121112211231124112511261127112811291130113111321133113411351136113711381139114011411142114311441145114611471148114911501151115211531154115511561157115811591160116111621163116411651166116711681169117011711172117311741175117611771178117911801181118211831184118511861187118811891190119111921193119411951196119711981199120012011202120312041205120612071208120912101211121212131214121512161217121812191220122112221223122412251226122712281229123012311232123312341235123612371238123912401241124212431244124512461247124812491250125112521253125412551256125712581259126012611262126312641265126612671268126912701271127212731274127512761277127812791280128112821283128412851286128712881289129012911292129312941295129612971298129913001301130213031304130513061307130813091310131113121313131413151316131713181319132013211322132313241325132613271328132913301331133213331334133513361337133813391340134113421343134413451346134713481349135013511352135313541355135613571358135913601361136213631364136513661367136813691370137113721373137413751376137713781379138013811382138313841385138613871388138913901391139213931394139513961397139813991400140114021403140414051406140714081409141014111412141314141415141614171418141914201421142214231424142514261427142814291430143114321433143414351436143714381439144014411442144314441445144614471448144914501451145214531454145514561457145814591460146114621463146414651466146714681469147014711472147314741475147614771478147914801481148214831484148514861487148814891490149114921493149414951496149714981499150015011502150315041505150615071508150915101511151215131514151515161517151815191520152115221523152415251526152715281529153015311532153315341535153615371538153915401541154215431544154515461547154815491550155115521553155415551556155715581559156015611562156315641565156615671568156915701571157215731574157515761577157815791580158115821583158415851586158715881589159015911592159315941595159615971598159916001601160216031604160516061607160816091610161116121613161416151616161716181619162016211622162316241625162616271628162916301631163216331634163516361637163816391640164116421643164416451646164716481649165016511652165316541655165616571658165916601661166216631664166516661667166816691670167116721673167416751676167716781679168016811682168316841685168616871688168916901691169216931694169516961697169816991700170117021703170417051706170717081709171017111712171317141715171617171718171917201721172217231724172517261727172817291730173117321733173417351736173717381739174017411742174317441745174617471748174917501751175217531754175517561757175817591760176117621763176417651766176717681769177017711772177317741775177617771778177917801781178217831784178517861787178817891790179117921793179417951796179717981799180018011802180318041805180618071808180918101811181218131814181518161817181818191820182118221823182418251826182718281829183018311832183318341835183618371838183918401841184218431844184518461847184818491850185118521853185418551856185718581859186018611862186318641865186618671868186918701871187218731874187518761877187818791880188118821883188418851886188718881889189018911892189318941895189618971898189919001901190219031904190519061907190819091910191119121913191419151916191719181919192019211922192319241925192619271928192919301931193219331934193519361937193819391940194119421943194419451946194719481949195019511952195319541955195619571958195919601961196219631964196519661967196819691970197119721973197419751976197719781979198019811982198319841985198619871988198919901991199219931994199519961997199819992000200120022003200420052006200720082009201020112012201320142015201620172018201920202021202220232024202520262027202820292030203120322033203420352036203720382039204020412042204320442045204620472048204920502051205220532054205520562057205820592060206120622063206420652066206720682069207020712072207320742075207620772078207920802081208220832084208520862087208820892090209120922093209420952096209720982099210021012102210321042105210621072108210921102111211221132114211521162117211821192120212121222123212421252126212721282129213021312132213321342135213621372138213921402141214221432144214521462147214821492150215121522153215421552156215721582159216021612162216321642165216621672168216921702171217221732174217521762177217821792180218121822183218421852186218721882189219021912192219321942195219621972198219922002201220222032204220522062207220822092210221122122213221422152216221722182219222022212222222322242225222622272228222922302231223222332234223522362237223822392240224122422243224422452246224722482249225022512252225322542255225622572258225922602261226222632264226522662267226822692270227122722273227422752276227722782279228022812282228322842285228622872288228922902291229222932294229522962297229822992300230123022303230423052306230723082309231023112312231323142315231623172318231923202321232223232324232523262327232823292330233123322333233423352336233723382339234023412342234323442345234623472348234923502351235223532354235523562357235823592360236123622363236423652366236723682369237023712372237323742375237623772378237923802381238223832384238523862387238823892390239123922393239423952396239723982399240024012402240324042405240624072408240924102411241224132414241524162417241824192420242124222423242424252426242724282429243024312432243324342435243624372438243924402441244224432444244524462447244824492450245124522453245424552456245724582459246024612462246324642465246624672468246924702471247224732474247524762477247824792480248124822483248424852486248724882489249024912492249324942495249624972498249925002501250225032504250525062507250825092510251125122513251425152516251725182519252025212522252325242525252625272528252925302531253225332534253525362537253825392540254125422543254425452546254725482549255025512552255325542555255625572558255925602561256225632564256525662567256825692570257125722573257425752576257725782579258025812582258325842585258625872588258925902591259225932594259525962597259825992600260126022603260426052606260726082609261026112612261326142615261626172618261926202621262226232624262526262627262826292630263126322633263426352636263726382639264026412642264326442645264626472648264926502651265226532654265526562657265826592660266126622663266426652666266726682669267026712672267326742675267626772678267926802681268226832684268526862687268826892690269126922693269426952696269726982699270027012702270327042705270627072708270927102711271227132714271527162717271827192720272127222723272427252726272727282729273027312732273327342735273627372738273927402741274227432744274527462747274827492750275127522753275427552756275727582759276027612762276327642765276627672768276927702771277227732774277527762777277827792780278127822783278427852786278727882789279027912792279327942795279627972798279928002801280228032804280528062807280828092810281128122813281428152816281728182819282028212822282328242825282628272828282928302831283228332834283528362837283828392840284128422843284428452846284728482849285028512852285328542855285628572858285928602861286228632864286528662867286828692870287128722873287428752876287728782879288028812882288328842885288628872888288928902891289228932894289528962897289828992900290129022903290429052906290729082909291029112912291329142915291629172918291929202921292229232924292529262927292829292930293129322933293429352936293729382939294029412942294329442945294629472948294929502951295229532954295529562957295829592960296129622963296429652966296729682969297029712972297329742975297629772978297929802981298229832984298529862987298829892990299129922993299429952996299729982999300030013002300330043005300630073008300930103011301230133014301530163017301830193020302130223023302430253026302730283029303030313032303330343035303630373038303930403041304230433044304530463047304830493050305130523053305430553056305730583059306030613062306330643065306630673068306930703071307230733074307530763077307830793080308130823083308430853086308730883089309030913092309330943095309630973098309931003101310231033104310531063107310831093110311131123113311431153116311731183119312031213122312331243125312631273128312931303131313231333134313531363137313831393140314131423143314431453146314731483149315031513152315331543155315631573158315931603161316231633164316531663167316831693170317131723173317431753176317731783179318031813182318331843185318631873188318931903191319231933194319531963197319831993200320132023203320432053206320732083209321032113212321332143215321632173218321932203221322232233224322532263227322832293230323132323233323432353236323732383239324032413242324332443245324632473248324932503251325232533254325532563257325832593260326132623263326432653266326732683269327032713272327332743275327632773278327932803281328232833284328532863287328832893290329132923293329432953296329732983299330033013302330333043305330633073308330933103311331233133314331533163317331833193320332133223323332433253326332733283329333033313332333333343335333633373338333933403341334233433344334533463347334833493350335133523353335433553356335733583359336033613362336333643365336633673368336933703371337233733374337533763377337833793380338133823383338433853386338733883389339033913392339333943395339633973398339934003401340234033404340534063407340834093410341134123413341434153416341734183419342034213422342334243425342634273428342934303431343234333434343534363437343834393440344134423443344434453446344734483449345034513452345334543455345634573458345934603461346234633464346534663467346834693470347134723473347434753476347734783479348034813482348334843485348634873488348934903491349234933494349534963497349834993500350135023503350435053506350735083509351035113512351335143515351635173518351935203521352235233524352535263527352835293530353135323533353435353536353735383539354035413542354335443545354635473548354935503551355235533554355535563557355835593560356135623563356435653566356735683569357035713572357335743575357635773578357935803581358235833584358535863587358835893590359135923593359435953596359735983599360036013602360336043605360636073608360936103611361236133614361536163617361836193620362136223623362436253626362736283629363036313632363336343635363636373638363936403641364236433644364536463647364836493650365136523653365436553656365736583659366036613662366336643665366636673668366936703671367236733674367536763677367836793680368136823683368436853686368736883689369036913692369336943695369636973698369937003701370237033704370537063707370837093710371137123713371437153716371737183719372037213722372337243725372637273728372937303731373237333734373537363737373837393740374137423743374437453746374737483749375037513752375337543755375637573758375937603761376237633764376537663767376837693770377137723773377437753776377737783779378037813782378337843785378637873788378937903791379237933794379537963797379837993800380138023803380438053806380738083809381038113812381338143815381638173818381938203821382238233824382538263827382838293830383138323833383438353836383738383839384038413842384338443845384638473848384938503851385238533854385538563857385838593860386138623863386438653866386738683869387038713872387338743875387638773878387938803881388238833884388538863887388838893890389138923893389438953896389738983899390039013902390339043905390639073908390939103911391239133914391539163917391839193920392139223923392439253926392739283929393039313932393339343935393639373938393939403941394239433944394539463947394839493950395139523953395439553956395739583959396039613962396339643965396639673968396939703971397239733974397539763977397839793980398139823983398439853986398739883989399039913992399339943995399639973998399940004001400240034004400540064007400840094010401140124013401440154016401740184019402040214022402340244025402640274028402940304031403240334034403540364037403840394040404140424043404440454046404740484049405040514052405340544055405640574058405940604061406240634064406540664067406840694070407140724073407440754076407740784079408040814082408340844085408640874088408940904091409240934094409540964097409840994100410141024103410441054106410741084109411041114112411341144115411641174118411941204121412241234124412541264127412841294130413141324133413441354136413741384139414041414142414341444145414641474148414941504151415241534154415541564157415841594160416141624163416441654166416741684169417041714172417341744175417641774178417941804181418241834184418541864187418841894190419141924193419441954196419741984199420042014202420342044205420642074208420942104211421242134214421542164217421842194220422142224223422442254226422742284229423042314232423342344235423642374238423942404241424242434244424542464247424842494250425142524253425442554256425742584259426042614262426342644265426642674268426942704271427242734274427542764277427842794280428142824283428442854286428742884289429042914292429342944295429642974298429943004301430243034304430543064307430843094310431143124313431443154316431743184319432043214322432343244325432643274328432943304331433243334334433543364337433843394340434143424343434443454346434743484349435043514352435343544355435643574358435943604361436243634364436543664367436843694370437143724373437443754376437743784379438043814382438343844385438643874388438943904391439243934394439543964397439843994400440144024403440444054406440744084409441044114412441344144415441644174418441944204421442244234424442544264427442844294430443144324433443444354436443744384439444044414442444344444445444644474448444944504451445244534454445544564457445844594460446144624463446444654466446744684469447044714472447344744475447644774478447944804481448244834484448544864487448844894490449144924493449444954496449744984499450045014502450345044505450645074508450945104511451245134514451545164517451845194520452145224523452445254526452745284529453045314532453345344535453645374538453945404541454245434544454545464547454845494550455145524553455445554556455745584559456045614562456345644565456645674568456945704571457245734574457545764577457845794580458145824583458445854586458745884589459045914592459345944595459645974598459946004601460246034604460546064607460846094610461146124613461446154616461746184619462046214622462346244625462646274628462946304631463246334634463546364637463846394640464146424643464446454646464746484649465046514652465346544655465646574658465946604661466246634664466546664667466846694670467146724673467446754676467746784679468046814682468346844685468646874688468946904691469246934694469546964697469846994700470147024703470447054706470747084709471047114712471347144715471647174718471947204721472247234724472547264727472847294730473147324733473447354736473747384739474047414742474347444745474647474748474947504751475247534754475547564757475847594760476147624763476447654766476747684769477047714772477347744775477647774778477947804781478247834784478547864787478847894790479147924793479447954796479747984799480048014802480348044805480648074808480948104811481248134814481548164817481848194820482148224823482448254826482748284829483048314832483348344835483648374838483948404841484248434844484548464847484848494850485148524853485448554856485748584859486048614862486348644865486648674868486948704871487248734874487548764877487848794880488148824883488448854886488748884889489048914892489348944895489648974898489949004901490249034904490549064907490849094910491149124913491449154916491749184919492049214922492349244925492649274928492949304931493249334934493549364937493849394940494149424943494449454946494749484949495049514952495349544955495649574958495949604961496249634964496549664967496849694970497149724973497449754976497749784979498049814982498349844985498649874988498949904991499249934994499549964997499849995000500150025003500450055006500750085009501050115012501350145015501650175018501950205021502250235024502550265027502850295030503150325033503450355036503750385039504050415042504350445045504650475048504950505051505250535054505550565057505850595060506150625063506450655066506750685069507050715072507350745075507650775078507950805081508250835084508550865087508850895090509150925093509450955096509750985099510051015102510351045105510651075108510951105111511251135114511551165117511851195120512151225123512451255126512751285129513051315132513351345135513651375138513951405141514251435144514551465147514851495150515151525153515451555156515751585159516051615162516351645165516651675168516951705171517251735174517551765177517851795180518151825183518451855186518751885189519051915192519351945195519651975198519952005201520252035204520552065207520852095210521152125213521452155216521752185219522052215222522352245225522652275228522952305231523252335234523552365237523852395240524152425243524452455246524752485249525052515252525352545255525652575258525952605261526252635264526552665267526852695270527152725273527452755276527752785279528052815282528352845285528652875288528952905291529252935294529552965297529852995300530153025303530453055306530753085309531053115312531353145315531653175318531953205321532253235324532553265327532853295330533153325333533453355336533753385339534053415342534353445345534653475348534953505351535253535354535553565357535853595360536153625363536453655366536753685369537053715372537353745375537653775378537953805381538253835384538553865387538853895390539153925393539453955396539753985399540054015402540354045405540654075408540954105411541254135414541554165417541854195420542154225423542454255426542754285429543054315432543354345435543654375438543954405441544254435444544554465447544854495450545154525453545454555456545754585459546054615462546354645465546654675468
%!TEX program = xelatex
%!TEX encoding = UTF-8 Unicode
\documentclass[
cn,
% result=noanswer,
device=pad,
12pt,
% fontset=none,
math=newtx,
citestyle=gb7714-2015,
bibstyle=gb7714-2015
]{elegantbook410}
% cite=authoryear % ganx: it does not work.
\title{金融随机分析讲义}
% \subtitle{}
\author{甘湘华}
% \institute{}
\date{\zhtoday}
\version{0.0.1}
% \bioinfo{自定义}{信息}
\extrainfo{}
\setcounter{tocdepth}{3}
% \newcommand{\dollar}{\mbox{\textdollar}}
\lstset{
mathescape = false}
\logo{logo_swufe.png}
\cover{cover.jpg}
\DeclareCaptionLabelFormat{nospace}{#1#2}
\captionsetup[table]{labelfont=bf, labelsep=period}
\renewcommand{\thetable}{L~\arabic{table}}
% \addbibresource{../bib/ganx_biblatex.bib}
% 本文档命令
% \usepackage{ganx_xeCJK}
\usepackage{fontawesome}
\usepackage[
automake, % This option = one runs 'makeglossaries sc4f' in terminal.
% counter=subsection,
style=listgroup,
]{glossaries-extra}
\makeglossaries{}
\input{glossaries} % Load glossaries.tex
\usepackage{float}
% Set the color of links to glossaries to be the same as current text.
% Reference: https://tex.stackexchange.com/a/392724/104838
\makeatletter
\newcommand*{\glsplainhyperlink}[2]{%
\colorlet{currenttext}{.}% store current text color
\colorlet{currentlink}{\@linkcolor}% store current link color
\hypersetup{linkcolor=currenttext}% set link color
\hyperlink{#1}{#2}%
\hypersetup{linkcolor=currentlink}% reset to default
}
\let\@glslink\glsplainhyperlink
\makeatother
\usepackage[final]{pdfpages}%插入pdf页宏
\usepackage{array}
\newcommand{\ccr}[1]{\makecell{{\color{#1}\rule{1cm}{1cm}}}}
\usepackage[lined,dotocloa,algochapter]{algorithm2e}
\SetKwData{Left}{left}\SetKwData{This}{this}\SetKwData{Up}{up}
\SetKwFunction{Union}{Union}\SetKwFunction{FindCompress}{FindCompress}
\SetKwInOut{Input}{input}\SetKwInOut{Output}{output}
\usetikzlibrary{mindmap,trees}
% \usepackage{todonotes_ganx} % Notworking.
\usepackage{comment}
\specialcomment{commentShow}{\begingroup\footnotesize\color{red}}{\endgroup}
% \excludecomment{commentShow} % Comment out to exclude commentShow.
% \includecomment{teachingNotes}
\excludecomment{teachingNotes}
\newcommand{\teach}[1]{}
\begin{teachingNotes}
\renewcommand{\teach}[1]{{\footnotesize\color{red} #1}}
\end{teachingNotes}
\def\baselinestretch{1.35}
\begin{document}
% \maketitle
% \frontmatter
% \makecover
\includepdf[pages={1}]{sc4f-title.pdf}
\tableofcontents
%\listofchanges
\mainmatter%
% \chapter*{序言}
% \markboth{序言}{Introduction}
% 为了方便西南财经大学数学荣誉课《金融随机分析 II》的教学,
% 我们编写了本讲义。
\chapter{概率论基础知识}%
\label{cha:prob_theory_finite_space}
\section{概率空间}%
\label{sec:prob_space}
% \subsection{基础知识}%
% \label{sub:prob_space_knowledge}
\begin{definition}[有限概率空间]\label{def:finite_prob_space}
\gls{yxglkj} (\gls{finite_probability_space})
包含
\gls{ybkj} (\gls{sample_space})
\gls{glcd} (\gls{probability_measure}).
习惯上我们一般用
\gls{O} 表示样本空间,
\gls{o} 表示其中的样本点,
\gls{P} 表示概率测度。
样本空间是一个非空有限集合,
概率测度 $ \mathbb{P}$ 为一个函数,
该函数将 $\Omega$ 中每个元素映射到 $[0, 1]$ 中的某个数,
使得:
\begin{equation}\label{eq:finite_prob_space_1}
\sum_{\omega\in\Omega}\mathbb{P}(\omega) = 1.
\end{equation}
一个 \gls{sj} (\gls{event}) 是 $\Omega$ 中的一个子集,
我们按如下方式定义事件 $A$ 的概率为:
\begin{equation}\label{eq:finite_prob_space_event}
\mathbb{P}(A) = \sum_{\omega\in A}\mathbb{P}(\omega).
\end{equation}
\end{definition}
\begin{definition}[一维抛掷硬币概率空间]\label{def:coin_toss_space}
如果试验由抛掷一枚硬币所构成,那么抛掷一次的样本空间为
\begin{equation}\label{eq:one_period_space}
\{H, T\},
\end{equation}
其中 $H$, $T$ 分别表示正反面。
习惯上我们一般用
$\Omega$ 表示样本空间,用 $\omega$ 表示其中的样本点。
对于 $\{H, T\}$ 这个特殊的样本空间,
我们用 $\mathbb{C}$ 来表示。
我们在样本空间上定义一个概率测度 $\mathbb{P}$:
\begin{equation}\label{eq:p_q_definition}
\mathbb{P}\{\omega = H\} = p
\mbox{ and }
\mathbb{P}\{\omega = T\} = q,
\end{equation}
其中 $p, q > 0$$p + q = 1$.
我们称
\begin{equation*}
(\mathbb{C}, \mathbb{P})
\end{equation*}
为一维抛掷硬币概率空间。
\end{definition}
很自然地,
我们将此定义推广到一般的抛掷硬币空间。
\begin{definition}[$N$ 维抛掷硬币概率空间]\label{def:multi_coin_space}
我们已经在定义~\ref{def:coin_toss_space}
定义了一维抛掷硬币样本空间 $\mathbb{C}$
当我们独立做 $N$ 次试验,
则样本空间为 $\mathbb{C}$$N$ 次笛卡尔乘积,
记此样本空间为 $\mathbb{C}^{(N)}$,
其中的样本点为 $\omega = \omega_1\cdots\omega_N$.
计算每个样本点的概率,
我们就定义了在 $\mathbb{C}^{(N)}$ 上的一个测度 $\mathbb{P}$.
我们称
\begin{equation}\label{eq:N_period_prob_space}
\left(\mathbb{C}^{(N)}, \mathbb{P}\right)
\end{equation}
$N$\gls{pzybglkj} (\gls{coin_toss_space}).
为了方便记录抛掷硬币的次数,
我们也使用$\omega^{(N)}$ 来表示 $\omega = \omega_1\cdots\omega_N$.
\end{definition}
% \subsection{练习}%
% \label{sub:finite_prob_space_exercise}
\begin{problem}\label{pr:two_period_coin_space}
如果将定义~\ref{def:coin_toss_space} 中的试验独立第做两次,
那么样本空间为笛卡尔乘积 $\mathbb{C}\times\mathbb{C}$.
我们用 $\mathbb{C}^{(2)}$ 表示二维抛掷硬币样本空间,
$\omega$ 表示其中的样本点 $\omega_1\omega_2$.
我们有
\begin{equation}\label{eq:two_period_sample_space}
\mathbb{C}^{(2)} = \{HH, HT, TH, TT\}.
\end{equation}
请计算每个样本点的概率,
并验证其为 $\mathbb{C}^{(2)}$ 上的一个概率测度 $\mathbb{P}$.
我们称
\begin{equation}\label{eq:two_period_coin_space}
\left(\mathbb{C}^{(2)}, \mathbb{P}\right)
\end{equation}
为二维抛掷硬币概率空间。
\end{problem}
\begin{solution}\label{sol:pr:two_period_coin_space}
计算每个样本点的概率,我们得到:
\begin{equation}\label{eq:two_coin_space_prob}
\begin{aligned}
\mathbb{P}\{\omega = HH\} &= p^2 \\
\mathbb{P}\{\omega = HT\} &= pq \\
\mathbb{P}\{\omega = TH\} &= pq \\
\mathbb{P}\{\omega = TT\} &= q^2,
\end{aligned}
\end{equation}
% next: align with better look.
其中 $p, q $ 由式 \eqref{eq:p_q_definition} 定义。
由于
\begin{equation*}
\sum_{\omega\in\mathbb{C}^{(2)}}\mathbb{P}(\omega) = 1,
\end{equation*}
式子 \eqref{eq:finite_prob_space_1} 成立。
所以我们就得到了 $\mathbb{C}^{(2)}$
上的一个概率测度 $\mathbb{P}$.
\end{solution}
\begin{definition}[$\sigma$-代数]\label{def:sigma_algebra}
$\Omega$ 为非空集合,
$\mathcal{F}$$\Omega$ 的子集族。
$ \mathcal{F}$ 是一个 $\sigma$-代数(也称 $\sigma$-域),如果:
\begin{enumerate}[label=\roman*)]
\item 空集 $\varnothing$ 属于 $ \mathcal{F}$;
\item 只要 $A$ 属于 $\mathcal{F}$, 则余集 $A^c$ 也属于 $\mathcal{F}$.
\item 只要一列集合 $A_1,A_2,\ldots$ 属于 $\mathcal{F}$,
则并集 $\bigcup_{n=1}^{\infty}$ 也属于 $\mathcal{F}$.
\end{enumerate}
\end{definition}
\pagebreak
\begin{definition}[概率空间]\label{def:prob_space}
$\Omega$ 为非空集合,
$\mathcal{F}$$\Omega$ 子集的一个 $\sigma$-代数。
% \gls{glkj} 包含 \gls{ybkj},$\sigma$-代数,和 \gls{glcd}。
概率测度 \gls{P} 是定义在 $\mathcal{F}$ 上的一个函数,满足:
\begin{enumerate}[label=\roman*)]
\item 对所有的 $A \in \mathcal{F}$, $0 \le \mathbb{P}(A) \leq 1$;
\item $\mathbb{P}(\Omega) = 1$;
\item (可数可加性)设 $A_1,A_2,\ldots$
$\mathcal{F}$ 中的一列互不相交的集合,则
\begin{equation}\label{eq:prob_space_event}
\mathbb{P}\left(\bigcup_{n=1}^{\infty}A_n\right) =
\sum_{n=1}^{\infty}\mathbb{P}(A_n).
\end{equation}
\end{enumerate}
我们称三元组 $(\Omega, \mathcal{F}, \mathbb{P})$ 为一个概率空间。
\end{definition}
\pagebreak
\begin{problem}[(概率空间的一些性质)]\label{pr:prob_space_properties}
由以上定义证明:
\begin{enumerate}
\item $\mathbb{P}(\varnothing) = 0$ ;
\item $\mathbb{P}(\bigcup_{n=1}^N A_n) = \sum_{n=1}^N\mathbb{P}(A_n)$;
\item $\mathbb{P}(A^c) = 1 - \mathbb{P}(A)$.
\end{enumerate}
\end{problem}
% \begin{hint}\label{hnt:pr:prob_space_properties}
% \end{hint}
% \noindent\faHandPointerO ~\hyperlink{target:pr:prob_space_properties}{前往答案}
% \noindent\faHandPointerO ~\hyperref[pr:prob_space_properties]{返回问题~\ref{pr:prob_space_properties}}
% \hypertarget{target:pr:prob_space_properties}{}
% \begin{solution}\label{sol:pr:prob_space_properties}
%
% \end{solution}
\begin{problem}[(均匀(勒贝格)测度)]\label{pr:L_measure}
定义闭区间 $[a,b]$ 的概率为:
\begin{equation}\label{eq:L_measure_0_1}
\mathbb{P}[a,b] = b - a, \quad 0 \le a \le b \le 1.
\end{equation}
我们称由全体闭区间出发而生成的 $\sigma$-代数称为 $[0,1]$ 的子集的
{\bf 伯雷尔 (Borel) $\sigma$-代数},
记为 $\mathbb{B}[0,1]$.
这一 $\sigma$-代数中的集合称为 {\bf 博雷尔集}
证明:
\begin{enumerate}
\item 单点集的概率为 0;
\item 开区间 $(a,b)\in \mathbb{B}[0,1]$.
\end{enumerate}
\end{problem}
% \begin{hint}\label{hnt:pr:L_measure}
% \end{hint}
% \noindent\faHandPointerO ~\hyperlink{target:pr:L_measure}{前往答案}
% \noindent\faHandPointerO ~\hyperref[pr:L_measure]{返回问题~\ref{pr:L_measure}}
% \hypertarget{target:pr:L_measure}{}
% \begin{solution}\label{sol:pr:L_measure}
%
% \end{solution}
\pagebreak
\begin{problem}[(无穷次独立抛掷硬币的空间)]\label{pr:toss_infty}
假设连续抛掷硬币无穷多次,定义
\begin{equation}\label{eq:toss_infty}
\Omega_{\infty} = \mbox{\{$H$$T$ 的无穷序列\}}
\end{equation}
\begin{enumerate}
\item 构造 $\Omega_{\infty}$ 上的 $\sigma$-代数 $\mathcal{F}_{\infty}$.
\item 证明 $\mathbb{P}(\mbox{每次抛掷硬币结果为正面}) = 0$;
\item 考虑满足以下条件的硬币抛掷结果序列 $\omega = \omega_1\omega_2\ldots$
的集合 $A$:
\begin{equation}\label{eq:omega_sequence_limit}
\lim_{n \to \infty} \frac{H_n(\omega_1\ldots\omega_n)}{n} = 1.
\end{equation}
证明 $A \in \mathcal{F}_{\infty}$.
\item 求解 $\mathbb{P}(A)$.
\end{enumerate}
\end{problem}
\pagebreak
\begin{hint}\label{hnt:pr:toss_infty}
\begin{enumerate}
\item$\mathcal{F}_n$, $n = 1, 2, \ldots$, 构造;
\item 用集合来表示;
\item 考虑集合
\begin{equation}\label{eq:construct_set}
S_{n,m} = \left\{ \omega;
\left| \frac{H_n(\omega_1\ldots\omega_n)}{n} - \frac{1}{2}\right|
\le \frac{1}{m} \right\}.
\end{equation}
\item 大数定律
\end{enumerate}
\end{hint}
% \noindent\faHandPointerO ~\hyperlink[sol:pr:toss_infty]{前往答案}
% \noindent\faHandPointerO ~\hyperref[pr:toss_infty]{返回问题~\ref{pr:toss_infty}}
% \hypertarget{sol:pr:toss_infty}{}答案
% \begin{proof}\label{pf:pr:toss_infty}
%
% \end{proof}
\section{随机变量、分布和期望}%
\label{sec:rv_dist_expectation}
% \subsection{基础知识}%
% \label{sub:rv_dist_expectation_knowledge}
\begin{definition}[有限概率空间下的随机变量]\label{def:finite_rv}
\footnote{此定义即书 \cite{shreve_stochastic_2004} 中的定义 2.2.1.}
$(\Omega, \mathbb{P})$ 为有限概率空间,
\gls{sjbl} (\gls{random_variable})
为定义在 $\Omega$ 上的一个实值函数。
\end{definition}
\begin{definition}[随机变量]\label{def:rv}
\footnote{此定义即书 \cite{shreve_stochastic_2008} 中的定义 1.2.1.}
$(\Omega, \mathcal{F}, \mathbb{P})$ 为一个概率空间,
$X$ 为定义在 $\Omega$ 上的一个实值函数。
如果对于 $\mathbb{R}$ 的任何博雷尔子集 $B$,
下列 $\Omega$ 的子集
\begin{equation}\label{eq:def:rv}
\{X\in B\} = \{\omega\in \Omega; X(\omega)\in B\}
\end{equation}
都在 $\sigma$-代数 $\mathcal{F}$ 中,
我们称 $X$\gls{sjbl}
\end{definition}
\begin{definition}[分布测度]\label{def:rv_distribution}
$X$ 是概率空间 $(\Omega, \mathcal{F}, \mathbb{P})$ 上的一个随机变量。
$X$\gls{fbcd} (\gls{distribution_measure})
是如下的概率测度 $\mu_X$:
$\mathbb{R}$ 的每一个博雷尔子集 $B$,
指定质量 $\mu_X(B) = \mathbb{P}\{X\in B\}$.
\end{definition}
除了用分布测度来描述随机变量的分布的方法,
我们可以用 {\bf 累积分布函数}
\begin{equation}\label{eq:cdf}
F(x) = \mathbb{P}\{X\le x\}, \quad x\in \mathbb{R}
\end{equation}
来刻画随机变量的分布。
\begin{definition}[有限概率空间下的期望]\label{def:expectation_finite}
\footnote{此定义对应于书 \cite{shreve_stochastic_2004} 中的定义 2.2.4.}
$X$ 为定义在有限概率空间 $\left( \Omega, \mathbb{P}\right)$
上的随机变量。
$X$ 的期望定义为:
\begin{equation*}
\mathbb{E}[X] = \sum_{\omega\in\Omega}X(\omega)\mathbb{P}(\omega).
\end{equation*}
$X$ 的函数 $g(X)$ 的期望定义为:
\begin{equation*}
\mathbb{E}[g(X)] =
\sum_{\omega\in\Omega} g(X)(\omega)\mathbb{P}(\omega).
\end{equation*}
$g(X) = (X - \mathbb{E}[X])^2$,
我们定义方差为 $\mathbb{E}[g(X)]$.
\end{definition}
\pagebreak
% 黎曼积分
\begin{figure}[H]
\centering
\includegraphics[width=0.8\textwidth]{image/lm.JPG}
\caption{黎曼积分}
\label{fig:integral_lm}
\end{figure}
% 勒贝格积分
\begin{figure}[H]
\centering
\includegraphics[width=0.9\textwidth]{image/lbg.JPG}
\caption{勒贝格积分}
\label{fig:integral_lbg}
\end{figure}
\begin{definition}[期望]\label{def:expectation}
\footnote{此定义对应于书 \cite{shreve_stochastic_2008} 中的定义 1.3.3.}
$X$ 为定义在概率空间 $\left( \Omega, \mathcal{F}, \mathbb{P}\right)$
上的随机变量。
$X$ 的期望定义为:
\begin{equation*}
\mathbb{E}[X] =
\int_{\omega\in\Omega}X(\omega)
\mathop{}\!\mathrm{d}\mathbb{P}(\omega).
\end{equation*}
$\mathbb{E}[X^+]$$\mathbb{E}[X^-]$ 中至少一个为有限是,
上式有意义。
$\mathbb{E}[X^+]$ 为无穷,而 $\mathbb{E}[X^-]$ 有限,
$\mathbb{E}[x]=+\infty$.
$\mathbb{E}[X^+]$ 为有限,而 $\mathbb{E}[X^-]$ 无穷,
$\mathbb{E}[x]=-\infty$.
$\mathbb{E}[X^+]$$\mathbb{E}[X^-]$ 均为有限,
$\mathbb{E}[x]$ 也有限。此时我们称 $X$ 可积。
$\mathbb{E}[X^+]$$\mathbb{E}[X^-]$ 均为无穷,
$\mathbb{E}[x]$ 没有定义。
\end{definition}
{\bf 勒贝格积分对于离散和连续随机变量的期望是统一的。}
\pagebreak
\begin{theorem}[詹森 (Jensen) 不等式]\label{thm:bookI_2_2_5}
\footnote{此定理对应于书 \cite{shreve_stochastic_2004} 中的定理 2.2.5.}
$X$ 为定义在有限概率空间
$\left( \Omega, \mathbb{P}\right) $ 上的随机变量,
$\varphi(\cdot)$ 为凸函数,
则有:
\begin{equation*}
\mathbb{E}[\varphi(X)] \geq \varphi(\mathbb{E}(X)).
\end{equation*}
\end{theorem}
% \subsection{练习}%
% \label{sub:rv_dist_expectation_exercise}
\begin{problem}[(随机变量的表示)]\label{pr:rv_3_coin_toss}
假设股票现在的价格为 $S_0 = 4$,
我们抛掷一枚硬币 3 次。
每一次抛掷硬币后,
如果是正面,则股票价格在当前的基础上加倍;
如果是反面,则股票价格在当前的基础上减半。
$i$ 次抛掷一枚硬币后,
记股票的价格为 $S_i$, $i = 1, 2, 3$.
请把 $S_i$, $i = 0, 1, 2, 3$,
表示为在 3 维抛掷硬币概率空间上的随机变量。
\end{problem}
\begin{solution}
[练习~\ref{pr:rv_3_coin_toss} (随机变量的表示)]
\label{sol:pr:rv_3_coin_toss}
\noindent
对于 $S_0$, 我们有
\begin{equation*}
S_0\left(\omega_1\omega_2\omega_3\right) = 4,
\quad \forall \omega_1\omega_2\omega_3\in \mathbb{C}^{(3)}.
\end{equation*}
对于 $S_1$, 我们有
\begin{equation*}
S_1\left(\omega_1\omega_2\omega_3\right) =
\begin{cases}
8, & \text{如果~} \omega_1 = H, \\
2, & \text{如果~} \omega_1 = T.
\end{cases}
\end{equation*}
对于 $S_2$, 我们有
\begin{equation*}
S_2\left(\omega_1\omega_2\omega_3\right) =
\begin{cases}
16, & \text{如果~} \omega_1 = \omega_2 = H, \\
4, & \text{如果~} \omega_1 \neq \omega_2, \\
1, & \text{如果~} \omega_1 = \omega_2 = T.
\end{cases}
\end{equation*}
对于 $S_3$, 我们有
\begin{equation*}
S_2\left(\omega_1\omega_2\omega_3\right) =
\begin{cases}
32, & \text{如果~} \omega_1 = \omega_2 = \omega_3 = H, \\
8, & \text{如果~抛掷结果为 2 次正面 1 次反面} , \\
2, & \text{如果~抛掷结果为 1 次正面 2 次反面} , \\
0.5, & \text{如果~} \omega_1 = \omega_2 = \omega_3 = T.
\end{cases}
\end{equation*}
\end{solution}
此练习的答案可以表示在图 \ref{fig:3_period_model} 中。
\begin{figure}[H]
\centering
\includegraphics[width=0.9\textwidth]{image/BT-3.pdf}
\caption{随机变量的表示 (练习~\ref{pr:rv_3_coin_toss})}
\label{fig:3_period_model}
\end{figure}
\section{信息和条件期望}%
\label{sec:conditional_expectation}
\subsection{基础知识}%
\label{sub:conditional_expectation_knowledge}
\begin{problem}[(二叉树模型中信息的表示)]\label{pr:infomation}
设想我们在进行某个随机实验,
其结果是所有可能结果的集合 $\Omega$ 中的一个元素 $\omega$.
我们可能获知一些信息,
虽不足以确切地得知 $\omega$,
但可以逐步缩小可能结果的范围。
虽然我们并不确切地知道真实的 $\omega$,
但我们可以列出那些肯定包含 $\omega$ 的集合以及肯定不含 $\omega$ 的集合。
这就是 {\bf 依信息分解}得到的集合。
\begin{enumerate}
\item 写出依第一次抛掷硬币结果分解得到的集合,以及其生成的 $\sigma$-代数。
\item 写出依前两次抛掷硬币结果分解得到的集合,以及其生成的 $\sigma$-代数。
\item 写出依前三次抛掷硬币结果分解得到的集合,以及其生成的 $\sigma$-代数。
\end{enumerate}
\end{problem}
% \begin{hint}\label{hnt:pr:infomation}
% \end{hint}
% \noindent\faHandPointerO ~\hyperlink{target:pr:infomation}{前往答案}
% \noindent\faHandPointerO ~\hyperref[pr:infomation]{返回问题~\ref{pr:infomation}}
% \hypertarget{target:pr:infomation}{}
% \begin{solution}\label{sol:pr:infomation}
%
% \end{solution}
\begin{definition}[域流]\label{def:filtration}
$\Omega$ 是非空集合。
$T$ 是固定的正数,并且对每一个 $t\in [0,T]$,
有一个 $\sigma$-代数 $ \mathcal{F}$.
进一步假定:
如果 $s\le t$,
$ \mathcal{F}(s)$
中的所有事件都在
$ \mathcal{F}(t)$ 中,
则称 $\sigma$-代数族 $ \mathcal{F}(t)$, $0 \le s \le t$,
是一个 {\bf 域流 (filtration)}.
\end{definition}
\pagebreak
\begin{definition}[由随机变量生成的$\sigma$-代数]\label{def:X_sigma_algebra}
$X$ 是定义在非空样本空间 $\Omega$ 上的随机变量。
{\bf$X$ 生成的$\sigma$-代数} [记为 $\sigma$-代数]
是所有形如 $\{X\in B\}$$\Omega$ 的子集族,
其中 $B$$\mathbb{R}$ 的博雷尔子集。
\end{definition}
\begin{problem}[($\sigma(S_2)$)]\label{pr:S_2_sigma_algebra}
在问题~\ref{pr:rv_3_coin_toss} 的三时段模型中,
$\Omega$ 是三次硬币抛掷的 8 个可能的集合。
在图~\ref{fig:3_period_model} 中,
我们将 $S_2$ 写成前两次硬币抛掷结果的函数。
请写出 $\sigma(S_2)$.
\end{problem}
% \begin{hint}\label{hnt:pr:S_2_sigma_algebra}
% \end{hint}
% \noindent\faHandPointerO ~\hyperlink{target:pr:S_2_sigma_algebra}{前往答案}
% \noindent\faHandPointerO ~\hyperref[pr:S_2_sigma_algebra]{返回问题~\ref{pr:S_2_sigma_algebra}}
% \hypertarget{target:pr:S_2_sigma_algebra}{}
% \begin{solution}\label{sol:pr:S_2_sigma_algebra}
%
% \end{solution}
\pagebreak
\begin{definition}[$ \mathcal{G}$-可测]\label{def:G_measurable}
$X$ 是定义在非空样本空间 $\Omega$ 上的随机变量。
$ \mathcal{G}$$\Omega$ 子集的 $\sigma$-代数。
如果 $\sigma(X)$ 中的所有集合都在 $ \mathcal{G}$ 中,
则称 $X$$ \mathcal{G}$-可测的。
\end{definition}
\begin{definition}[适应的随机过程]\label{def:adaptive_sp}
$\Omega$ 是非空样本空间,
并有域流 $ \mathcal{F}(t)$, $0\le t\le T$.
$X(t)$ 是以 $t\in [0,T]$ 标记的一族随机变量。
我们称这一族随机变量是一个
{\bf 适应的随机过程}
如果对每一个 $t$,
随机变量 $X(t)$ 都是 $ \mathcal{F}$-可测的。
\end{definition}
\pagebreak
\begin{definition}[独立]\label{def:independence}
$(\Omega, \mathcal{F}, \mathbb{P})$ 是概率空间,
$\mathcal{G}$$\mathcal{H}$$ \mathcal{F}$ 的子 $\sigma$-代数。
我们称这两个 $\sigma$-代数是 {\bf 独立的}
如果:
\begin{equation}\label{eq:independence}
\mathbb{P}(A\cap B) = \mathbb{P}(A)\cdot \mathbb{P}(B),
\quad \forall A\in \mathcal{G}, B\in \mathcal{H}.
\end{equation}
$X$$Y$$(\Omega, \mathcal{F}, \mathbb{P})$ 上的随机变量。
我们称这两个随机变量是 {\bf 独立的}
如果它们生成的 $\sigma$-代数 $\sigma(X)$$\sigma(Y)$
{\bf 独立的}
我们称这随机变量 $X$$\sigma$-代数 $\mathcal{G}${\bf 独立的}
如果它们生成的 $\sigma(X)$$\mathcal{G}$
{\bf 独立的}
\end{definition}
\pagebreak
\begin{problem}[(独立与否的验证)]\label{pr:verify_independence}
\footnote{此问题的解答可以参考书 \cite{shreve_stochastic_2008} 中的第 45-46 页.}
$\Omega_3$ 是三次独立抛掷硬币的空间。
书~\cite{shreve_stochastic_2004}
中的例 1.2.1 中构造了定义在其上的股价随机变量。
设概率测度 $\mathbb{P}$ 为:
\begin{multline*}
\mathbb{P}(HHH) = p^3,
\mathbb{P}(HHT) = p^2q,
\mathbb{P}(HTH) = p^2q,
\mathbb{P}(HTT) = pq^2, \\
\mathbb{P}(THH) = p^2q,
\mathbb{P}(THT) = pq^2,
\mathbb{P}(TTH) = pq^2,
\mathbb{P}(TTT) = q^3.
\end{multline*}
验证 $S_2$$S_3$ 不是独立的,
$S_2$$S_3 / S_2$ 独立。
\end{problem}
% \begin{hint}\label{hnt:pr:verify_independence}
% \end{hint}
% \noindent\faHandPointerO ~\hyperlink{target:pr:verify_independence}{前往答案}
% \noindent\faHandPointerO ~\hyperref[pr:verify_independence]{返回问题~\ref{pr:verify_independence}}
% \hypertarget{target:pr:verify_independence}{}
% \begin{solution}\label{sol:pr:verify_independence}
%
% \end{solution}
\pagebreak
\begin{definition}[二叉树模型中的条件期望]\label{def:conditional_exp_binomial}
\footnote{此定义对应于书~\cite{shreve_stochastic_2004} 中的定义 2.3.1.}
$X$ 是定义在
$\left(\mathbb{C}^{(N)}, \mathbb{P}\right)$
上的随机变量。
给定前 $n$ 次抛掷硬币的结果 $\omega^{(n)}$, $1\leq n\leq N$,
我们定义 $X$ 基于时刻 $n$ 信息(前 $n$ 次抛掷结果)的条件期望为:
\begin{equation}\label{eq:conditional_exp_binomial}
\mathbb{E}_n[X]\left(\omega^{(n)}\right) =
\sum_{\omega_{n+1}\cdots\omega_N}
p^{\#H_{n+1}^N}
q^{\#T_{n+1}^N}
X\left(\omega^{(n)}\omega_{n+1}\cdots\omega_N\right),
\end{equation}
% \footnote{
% 表达式 $\omega^{(n)}$ 对应于书~\cite{shreve_stochastic_2004}
% 中的 $\omega_1\cdots\omega_n$,
% 表达式 $\#H_{n+1}^N$ 对应于书~\cite{shreve_stochastic_2004} 中的
% $\#H(\omega_{n+1}\cdots\omega_N)$,
% 表达式 $\#T_{n+1}^N$ 对应于书~\cite{shreve_stochastic_2004} 中的
% $\#T(\omega_{n+1}\cdots\omega_N)$.
% }
其中
$p^{\#H_{n+1}^N}$ 是第 $n+1$ 次到第 $N$ 次抛掷中出现正面的次数,
$q^{\#T_{n+1}^N}$ 是第 $n+1$ 次到第 $N$ 次抛掷中出现反面的次数。
我们定义 $\mathbb{E}_n[X]$ 为定义在
$\left(\mathbb{C}^{(n)}, \mathbb{P}\right)$
上的随机变量,
其在每个样本点上的值由式子 \eqref{eq:conditional_exp_binomial} 决定。
\end{definition}
\pagebreak
\begin{definition}[条件期望]\label{def:bookI_2_3_1}
\footnote{此定义对应于书~\cite{shreve_stochastic_2008} 中的定义 2.3.1.}
$(\Omega, \mathcal{F}, \mathbb{P})$ 是概率空间,
$\mathcal{G}$$ \mathcal{F}$ 的子 $\sigma$-代数。
$X$ 是非负或者可积的随机变量。
(基于 $ \mathcal{G}$$X$ 的条件期望(记为 $ \mathbb{E}[X|\mathcal{G}]$
是满足以下性质的随机变量:
\begin{enumerate}[label=\roman*)]
\item (可测性)$\mathbb{E}[X|\mathcal{G}]$$\mathcal{G}$-可测的。
\item (部分平均)
\begin{equation}\label{eq:partial_averaging}
\int_{A} \mathbb{E}[X|\mathcal{G}](\omega)
\mathop{}\!\mathrm{d} \mathbb{P}(\omega)
=
\int_{A} X(\omega)
\mathop{}\!\mathrm{d} \mathbb{P}(\omega),
\quad \forall A\in \mathcal{G}.
\end{equation}
\end{enumerate}
如果 $ \mathcal{G}$ 是由另一随机变量 $W$ 生成的
$\sigma$-代数(即 $ \mathcal{G} = \sigma(W)$),
则我们一般将 $\mathbb{E}[X|\sigma(W)]$ 记为 $\mathbb{E}[X|W]$.
\end{definition}
\begin{theorem}[二叉树模型中条件期望的基本性质]\label{thm:bookI_2_3_2}
\footnote{此定理对应于书 \cite{shreve_stochastic_2004} 中的定理 2.3.2.}
$N$ 为正整数,
$X$$Y$ 为依赖于前 $N$ 次抛掷硬币结果的随机变量。
对于给定的 $0\leq n\leq N$,
以下性质成立。
\begin{enumerate}[label=\roman*)]\label{enu:properties_binomial}
\item {\bf 条件期望的线性性} \label{itm:property_1_binomial}
对于所有常数 $c_1, c_2$,
我们有:
\begin{equation}\label{eq:property_1_binomial}
\mathbb{E}_n[c_1X + c_2Y] =
c_1 \mathbb{E}[X]
+
c_2 \mathbb{E}[Y].
\end{equation}
\item {\bf 提取已知量} \label{itm:property_2_binomial}
如果 $X$ 实际上只依赖于前 $n$ 次硬币抛掷,那么:
\begin{equation}\label{eq:property_2_binomial}
\mathbb{E}_n[XY] = X \cdot \mathbb{E}_n[Y].
\end{equation}
\item {\bf 累次条件期望} \label{itm:property_3_binomial}
如果 $0\leq n\leq m\leq N$, 那么:
\begin{equation}\label{eq:property_3_binomial}
\mathbb{E}_n[\mathbb{E}_m[X]] = \mathbb{E}_n[X].
\end{equation}
特别地,
\begin{equation}\label{eq:property_3_2_binomial}
\mathbb{E}[\mathbb{E}_m[X]] = \mathbb{E}[X].
\end{equation}
\item {\bf 独立性} \label{itm:property_4_binomial}
如果 $X$ 只依赖于从第 $n+1$ 次至第 $N$ 次抛掷硬币的结果,那么:
\begin{equation}\label{eq:4_binomial}
\mathbb{E}_n[X] = \mathbb{E}[X].
\end{equation}
\item {\bf 条件詹森不等式} \label{itm:property_5_binomial}
如果 $\varphi(\cdot)$ 为凸函数,
则有:
\begin{equation}\label{eq:property_5_binomial}
\mathbb{E}_n[\varphi(X)] \geq \varphi(\mathbb{E}_n(X)).
\end{equation}
\end{enumerate}
\end{theorem}
\begin{theorem}[条件期望的基本性质]\label{thm:bookII_2_3_2}
\footnote{此定理对应于书 \cite{shreve_stochastic_2008} 中的定理 2.3.2.}
$(\Omega, \mathcal{F}, \mathbb{P})$ 是概率空间,
$\mathcal{G}$$ \mathcal{F}$ 的子 $\sigma$-代数。
\begin{enumerate}[label=\roman*)]
\item {\bf 条件期望的线性性}
对于所有常数 $c_1, c_2$,
我们有:
\begin{equation}\label{eq:property_1}
\mathbb{E}[c_1X + c_2Y | \mathcal{G}] =
c_1 \mathbb{E}[X | \mathcal{G}]
+
c_2 \mathbb{E}[Y | \mathcal{G}].
\end{equation}
\item {\bf 提取已知量}
如果 $X$$\mathcal{G}$-可测,那么:
\begin{equation}\label{eq:property_2}
\mathbb{E}[XY | \mathcal{G}] =
X \cdot \mathbb{E}[Y | \mathcal{G}].
\end{equation}
\item {\bf 累次条件期望}
如果 $\mathcal{H}$$\mathcal{G}$ 的子 $\sigma$-代数, 那么:
\begin{equation}\label{eq:property_3}
\mathbb{E}[\mathbb{E}[X | \mathcal{G}] | \mathcal{H}] =
\mathbb{E}[X | \mathcal{H}].
\end{equation}
\item {\bf 独立性} \label{itm:property_4}
如果 $X$ 独立于 $\mathcal{G}$
\begin{equation}\label{eq:4}
\mathbb{E}[X | \mathcal{G}]] = \mathbb{E}[X].
\end{equation}
\item {\bf 条件詹森不等式} \label{itm:property_5}
如果 $\varphi(\cdot)$ 为凸函数,
则有:
\begin{equation}\label{eq:property_5}
\mathbb{E}[\varphi(X) | \mathcal{G}]] \geq
\varphi(\mathbb{E}(X) | \mathcal{G}]).
\end{equation}
\end{enumerate}
\end{theorem}
% \subsection{练习}%
% \label{sub:conditional_expectation_exercise}
\begin{problem}\label{pr:conditional_distribution_3}
练习~\ref{pr:rv_3_coin_toss}
中的 $S_3$ 是定义在
$\left(\mathbb{C}^{(3)}, \mathbb{P}\right)$
上的随机变量。
% 参考图 \ref{fig:3_period_model},
基于
练习~\ref{pr:rv_3_coin_toss}
中同样的假设,
并且假设 $p = q = 0.5$,
计算
$\mathbb{E}_2[S_3]$
以及
$\mathbb{E}_1[S_3]$.
\end{problem}
\begin{figure}[htpb]
\centering
\includegraphics[width=0.9\textwidth]{image/BT-3-CE.pdf}
\caption{}
\label{fig:binomial_tree_3_CE}
\end{figure}
下面,
我们在问题
\ref{pr:conditional_expectation_linear}
-
\ref{pr:conditional_expectation_independence}
中分别验证条件期望的前四个性质。
如图 \ref{fig:binomial_tree_3_CE},
$p = 2 / 3$, $q = 1 / 3$.
\begin{problem}[(条件期望的线性性)]\label{pr:conditional_expectation_linear}
验证:
\begin{equation}\label{eq:conditional_expectation_linear}
\mathbb{E}_1[S_2 + S_3]
=
\mathbb{E}_1[S_2] + \mathbb{E}_1[S_3].
\end{equation}
\end{problem}
% \begin{solution}\label{sol:conditional_expectation_linear}
%
% \end{solution}
\begin{problem}[(提取已知量)]\label{pr:conditional_expectation_known}
验证:
\begin{equation}\label{eq:conditional_expectation_known}
\mathbb{E}_1[S_1S_2]
=
S_1 \mathbb{E}_1[S_2].
\end{equation}
\end{problem}
% \begin{solution}\label{sol:conditional_expectation_known}
%
% \end{solution}
\begin{problem}[(累次条件期望)]\label{pr:conditional_expectation_iterated}
验证:
\begin{equation}\label{eq:conditional_expectation_iterated}
\mathbb{E}_1\left[ \mathbb{E}_2[S_3]\right]
=
\mathbb{E}_1[S_3].
\end{equation}
\end{problem}
% \begin{solution}\label{sol:conditional_expectation_iterated}
%
% \end{solution}
\begin{problem}[(独立性)]\label{pr:conditional_expectation_independence}
\begin{equation}\label{eq:conditional_expectation_independence}
\mathbb{E}_1\left[ \frac{S_2}{S_1}\right]
=
\mathbb{E}\left[ \frac{S_2}{S_1}\right].
\end{equation}
\end{problem}
% \begin{solution}\label{sol:conditional_expectation_independence}
%
% \end{solution}
\section{随机游动及布朗运动}%
\label{sec:random_walk}
\subsection{随机游动}%
\label{sub:random_walk}
我们反复抛掷一枚均匀(每次抛掷,出现正面和背面的概率都等于 1/2)的硬币,
其结果记为 $\omega_1\omega_2\omega_3\ldots$.
设:
\begin{equation}\label{eq:X_j}
X_j =
\begin{cases}
1, & \text{if } \omega_j = H, \\
-1, & \text{if } \omega_j = T.
\end{cases}
\end{equation}
定义 $M_0 = 0$,
并且:
\begin{equation}\label{eq:random_walk_symmetric}
M_n = \sum_{j=1}^{n} X_j, \quad n = 1, 2, \ldots.
\end{equation}
过程 $M_n$, $n = 1, 2, \ldots$,
就是一个 {\bf 对称随机游动}
随机游动具有 {\bf 独立增量}
这意味着:
对于非负整数 $0 = k_0 < k_1 < \ldots < k_m$,
随机变量
\begin{equation}\label{eq:independent_increments}
(M_{k_1} - M_{k_0}), (M_{k_2} - M_{k_1}), \ldots, (M_{k_m} - M_{k_{m-1}})
\end{equation}
是两两独立的。
\begin{problem}[(增量的方差)]\label{pr:variance_independent_increments}
计算 $(M_{k_{i+1}} - M_{k_i})$ 的方差。
\end{problem}
\begin{hint}\label{hnt:pr:variance_independent_increments}
此问题的解答可以参考书 \cite{shreve_stochastic_2008} 中的第 69 页.
\end{hint}
% \noindent\faHandPointerO ~\hyperlink{target:pr:variance_independent_increments}{前往答案}
% \noindent\faHandPointerO ~\hyperref[pr:variance_independent_increments]{返回问题~\ref{pr:variance_independent_increments}}
% \hypertarget{target:pr:variance_independent_increments}{}
% \begin{solution}\label{sol:pr:variance_independent_increments}
%
% \end{solution}
\begin{definition}[矩母函数]\label{def:moment_generating_function}
我们称 $\mathbb{E}[e^{uX}]$ 为随机变量 $X$ 的矩母函数。
\end{definition}
\pagebreak
\subsection{按比例缩小型对称随机游动}%
\label{sub:an_bi_li_suo_xiao_xing_dui_cheng_sui_ji_you_dong_}
为了逼近一个布朗运动,
我们加快时间并且缩小随机游动的步幅。
确而言之,
我们固定正整数 $n$,
并且定义 {\bf 按比例缩小型随机游动}
\begin{equation}\label{eq:random_walk_scaled}
W^{(n)}(t) = \frac{1}{\sqrt{n}} M_{nt},
\end{equation}
其中 $nt$ 为整数。
\begin{problem}[(按比例缩小型对称随机游动)]\label{pr:random_walk_scaled}
考察 $n = 100$ 的按比例缩小型对称随机游动。计算:
\begin{enumerate}
\item $W^{(100)}(0.70) - W^{(100)}(0.20)$ 的均值和方差;
\item 二次变差 $[W^{(100)}, W^{(100)}](1.37)$.
\end{enumerate}
\end{problem}
\begin{hint}\label{hnt:pr:random_walk_scaled}
此问题的解答可以参考书 \cite{shreve_stochastic_2008} 中的第 71 页.
\end{hint}
% \noindent\faHandPointerO ~\hyperlink{target:pr:random_walk_scaled}{前往答案}
% \noindent\faHandPointerO ~\hyperref[pr:random_walk_scaled]{返回问题~\ref{pr:}}
% \hypertarget{target:pr:random_walk_scaled}{}
% \begin{solution}\label{sol:pr:random_walk_scaled}
%
% \end{solution}
% \section{随机过程%
% \label{sec:stochastic_process}
%
\subsection{按比例缩小型对称随机游动的极限分布}%
\label{sub:an_bi_li_suo_xiao_xing_dui_cheng_sui_ji_you_dong_limit}
\begin{problem}[]\label{pr:limit_distribution}
画出 $W^{(100)}(0.25)$ 分布的直方图。
\end{problem}
\begin{hint}\label{hnt:pr:limit_distribution}
此问题的解答可以参考书 \cite{shreve_stochastic_2008} 中的第 73 页.
\end{hint}
% \noindent\faHandPointerO ~\hyperlink{target:pr:limit_distribution}{前往答案}
% \noindent\faHandPointerO ~\hyperref[pr:limit_distribution]{返回问题~\ref{pr:limit_distribution}}
% \hypertarget{target:pr:limit_distribution}{}
% \begin{solution}\label{sol:pr:limit_distribution}
%
% \end{solution}
\begin{theorem}[中心极限定理]\label{thm:bookII_3_2_1}
\footnote{此定理对应于书 \cite{shreve_stochastic_2008} 中的定理 3.2.1.}
固定 $t\ge 0$,
按比例缩小型随机游动 $W^{(n)}(t)$ 在时刻 $t$ 取值的分布
$n\to\infty$ 时收敛于均值为 0、方差为 $t$ 的正态分布。
\end{theorem}
\pagebreak
\subsection{对数正态分布作为二叉树模型的极限}%
\label{sub:dui_shu_zheng_tai_fen_bu_zuo_wei_er_cha_shu_mo_xing_de_ji_xian_}
利用中心极限定理(\ref{thm:bookII_3_2_1}) 可以证明:
适当的二叉树资产定价模型的极限分布给出了服从对数正态分布的股票价格。
下面,我们在利率 $r$ 为 0 的假定下给出这一极限论证。
通过选取整数 $n$ 并构造单位时间内取 $n$ 步的二叉树模型,
我们来构建从 0 到 $t$ 时间区间上股票价格的模型。
假定 $n$$t$ 的选取使得 $nt$ 是整数。
取上升因子和下降因子分别为
\begin{equation}\label{eq:random_walk_scaled_sd}
u_n = 1 + \frac{\sigma}{\sqrt{n}}, \quad
d_n = 1 - \frac{\sigma}{\sqrt{n}}.
\end{equation}
其中 $\sigma$ 为正,代表极限股价过程的波动率。
\begin{problem}[]\label{pr:stock_price_expression}
请写出时刻 $t$ 股价的表示式 $S_n(t)$
\end{problem}
\begin{hint}\label{hnt:pr:stock_price_expression}
此问题的解答可以参考书 \cite{shreve_stochastic_2008} 中的第 75 页.
\end{hint}
% \noindent\faHandPointerO ~\hyperlink{target:pr:stock_price_expression}{前往答案}
% \noindent\faHandPointerO ~\hyperref[pr:stock_price_expression]{返回问题~\ref{pr:stock_price_expression}}
% \hypertarget{target:pr:stock_price_expression}{}
% \begin{solution}\label{sol:pr:stock_price_expression}
%
% \end{solution}
\begin{theorem}[在时刻 $t$ 股价的极限分布]\label{thm:bookII_3_2_2}
\footnote{此定理对应于书 \cite{shreve_stochastic_2008} 中的定理 3.2.2.}
$n\to\infty$ 时,
$S_n(t)$ 的分布收敛于
\begin{equation}\label{eq:stock_price_expression_limit}
S(t) = S(0) e^{\sigma W(t) - \frac{1}{2} \sigma^2t}
\end{equation}
的分布,
其中 $W(t)$ 是均值为 0, 方差为 $t$ 的正态随机变量。
\end{theorem}
\pagebreak
\subsection{布朗运动}%
\label{sub:bu_lang_yun_dong_}
作为按比例缩小型随机游动 $W^{(n)}(t)$$n\to\infty$ 时的极限,
布朗运动继承了这些随机游动的性质。
\begin{definition}[布朗运动]\label{def:bookII_3_3_1}
\footnote{此定义对应于书~\cite{shreve_stochastic_2008} 中的定义 3.3.1.}
$(\Omega, \mathcal{F}, \mathbb{P})$ 是概率空间。
对每个 $\omega\in\Omega$,
假设存在依赖于 $\omega$ 的,
满足 $W(0) = 0$ 的连续函数 $W(t) (t\ge 0)$.
$W(t) (t\ge 0)$ 是一个 {\bf 布朗运动}
如果对所有 $0 = t_0 < t_1 < \ldots < t_m$,
增量
\begin{equation}\label{eq:brownian_motion}
W(t_1) - W(t_0),
W(t_2) - W(t_1),
\ldots,
W(t_m) - W(t_{m-1}),
\end{equation}
两两独立,
每个增量服从正态分布,并且
\begin{equation}\label{eq:brownian_motion_E}
\mathbb{E}\left[ W(t_{i+1}) - W(t_i)\right] = 0
\end{equation}
\begin{equation}\label{eq:brownian_motion_V}
Var\left[ W(t_{i+1}) - W(t_i)\right] = t_{i+1} - t_i.
\end{equation}
\end{definition}
\pagebreak % phone model
\begin{problem}[(定义的理解)]\label{pr:brownian_motion_understanding}
对于定义~\ref{def:bookII_3_3_1},
怎样理解布朗运动概率空间 $(\Omega, \mathcal{F}, \mathbb{P})$ 中的
$\omega\in\Omega$?
\end{problem}
\begin{hint}\label{hnt:pr:brownian_motion_understanding}
此问题的解答可以参考书 \cite{shreve_stochastic_2008} 中的第 77 页.
\end{hint}
% \noindent\faHandPointerO ~\hyperlink{target:pr:brownian_motion_understanding}{前往答案}
% \noindent\faHandPointerO ~\hyperref[pr:brownian_motion_understanding]{返回问题~\ref{pr:brownian_motion_understanding}}
% \hypertarget{target:pr:brownian_motion_understanding}{}
% \begin{solution}\label{sol:pr:brownian_motion_understanding}
%
% \end{solution}
\begin{problem}[(与按比例缩小型对称随机游动的联系)]\label{pr:brownian_motion_understanding2}
布朗运动中的 $\omega$$W(t)$ 分别对应于按比例缩小型随机游动中什么量?
\end{problem}
% \begin{hint}\label{hnt:pr:brownian_motion_understanding2}
% \end{hint}
% \noindent\faHandPointerO ~\hyperlink{target:pr:brownian_motion_understanding2}{前往答案}
% \noindent\faHandPointerO ~\hyperref[pr:]{返回问题~\ref{pr:brownian_motion_understanding2}}
% \hypertarget{target:pr:brownian_motion_understanding2}{}
% \begin{solution}\label{sol:pr:brownian_motion_understanding2}
%
% \end{solution}
\begin{problem}[(求概率)]\label{pr:brownian_motion_understanding3}
求使得布朗运动的路径满足 $0\le W(0.25)\le 0.2$ 的所有 $\omega\in \Omega$
的集合的概率。
\end{problem}
\begin{hint}\label{hnt:pr:brownian_motion_understanding3}
此问题的解答可以参考书 \cite{shreve_stochastic_2008} 中的第 77 页.
\end{hint}
% \noindent\faHandPointerO ~\hyperlink{target:pr:brownian_motion_understanding2}{前往答案}
% \noindent\faHandPointerO ~\hyperref[pr:brownian_motion_understanding3]{返回问题~\ref{pr:brownian_motion_understanding3}}
% \hypertarget{target:pr:brownian_motion_understanding3}{}
% \begin{solution}\label{sol:pr:brownian_motion_understanding3}
%
% \end{solution}
\pagebreak % phone model
\begin{theorem}[布朗运动的等价刻划]\label{thm:brownian_motion}
\footnote{此定理对应于书 \cite{shreve_stochastic_2008} 中的定理 3.3.2.}
$(\Omega, \mathcal{F}, \mathbb{P})$ 是概率空间。
对每个 $\omega\in\Omega$,
假设存在依赖于 $\omega$ 的,
满足 $W(0) = 0$ 的连续函数 $W(t) (t\ge 0)$,
则以下三条性质等价:
\begin{enumerate}[label=\roman*)]
\item
如果对所有 $0 = t_0 < t_1 < \ldots < t_m$,
增量
\begin{equation}\label{eq:brownian_motion_2}
W(t_1) - W(t_0),
W(t_2) - W(t_1),
\ldots,
W(t_m) - W(t_{m-1}),
\end{equation}
两两独立,
每个增量服从正态分布,并且
\begin{equation}\label{eq:brownian_motion_E_2}
\mathbb{E}\left[ W(t_{i+1}) - W(t_i)\right] = 0
\end{equation}
\begin{equation}\label{eq:brownian_motion_V_2}
Var\left[ W(t_{i+1}) - W(t_i)\right] = t_{i+1} - t_i.
\end{equation}
\item
如果对所有 $0 = t_0 < t_1 < \ldots < t_m$,
$W(t_1), W(t_2), \ldots, W(t_m)$
为联合正态随机变量,
均值为 0,
并且协方差矩阵为:
\begin{equation}\label{eq:covriance_matrix}
\begin{bmatrix}
t_1 & t_1 & \ldots & t_1\\
t_1 & t_2 & \ldots & t_2\\
\vdots & \vdots & \ddots & \vdots\\
t_1 & t_2 & \ldots & t_m\\
\end{bmatrix}
.
\end{equation}
\item
如果对所有 $0 = t_0 < t_1 < \ldots < t_m$,
$W(t_1), W(t_2), \ldots, W(t_m)$
具有给定联合矩母函数。
\end{enumerate}
如果 i), ii), iii) 之一成立(从而全都成立),则 $W(t)$ 是布朗运动。
\end{theorem}
\begin{problem}[(随机游动 $M_n$ 的鞅和马尔可夫性)]\label{pr:M_n_martingale_markov}
证明 $M_n$, $n = 1, 2, \ldots$, 既是鞅又是马尔可夫过程。
\end{problem}
% \begin{hint}\label{hnt:pr:M_n_martingale_markov}
% \end{hint}
% \noindent\faHandPointerO ~\hyperlink[sol:pr:M_n_martingale_markov]{前往答案}
% \noindent\faHandPointerO ~\hyperref[pr:M_n_martingale_markov]{返回问题~\ref{pr:M_n_martingale_markov}}
% \hypertarget{sol:pr:M_n_martingale_markov}{}答案
% \begin{proof}\label{pf:pr:M_n_martingale_markov}
%
% \end{proof}
\begin{theorem}[布朗运动的鞅和马尔可夫性]\label{thm:bookII3_3_4-3_5_1}
\footnote{此定理对应于书 \cite{shreve_stochastic_2008} 中的定理 3.3.4 和 3.5.1.}
布朗运动既是鞅,
也是马尔可夫过程。
\end{theorem}
\subsection{二次变差}%
\label{sub:er_ci_bian_chai_}
\begin{definition}[随机游动的二次变差]\label{def:ecbc}
\footnote{此定义在书 \cite{shreve_stochastic_2008} 中的第 70 页.}
截至时刻 $k${\bf 二次变差}定义为:
\begin{equation}\label{eq:ecbc}
[M, M]_k = \sum_{j=1}^{k} (M_j - M_{j-1})^2 = k.
\end{equation}
\end{definition}
% \pagebreak
\begin{definition}[二次变差]\label{def:bookII_3_4_1}
\footnote{此定义对应于书~\cite{shreve_stochastic_2008} 中的定义 3.4.1.}
$f(t)$ 是关于 $0\le t\le T$ 有定义的函数。
截止时刻 $T$,
$f$ 的二次变差为:
\begin{equation}\label{eq:quadratic_variation}
[f,f](T) = \lim_{ \parallel \varPi \parallel \to 0}
\sum_{j=0}^{n-1} [f(t_{j+1}) - f(t_{i})]^2,
\end{equation}
其中 $\varPi = \{t_0,t_1,\ldots,t_n\}$
并且 $0 = t_0 < t_1 < \ldots < t_n = T$.
\end{definition}
\begin{theorem}[布朗运动的二次变差]\label{thm:bookII_3_4_3}
\footnote{此定理对应于书 \cite{shreve_stochastic_2008} 中的定理 3.4.3.}
$W$ 是布朗运动,
则对所有 $T\ge 0$,
$[W,W](T) = T$
几乎必然成立。
\end{theorem}
\begin{definition}[几何布朗运动]\label{def:geometric_brownian_motion}
\footnote{此定义对应于书~\cite{shreve_stochastic_2008} 中的 86 - 87 页。}
$\alpha$$\sigma>0$ 是常数,
定义 {\bf 几何布朗运动}
\begin{equation}\label{eq:geometric_brownian_motion}
S(t) = S_0 \exp \left\{
\sigma W(t)+ \left(\alpha - \frac{1}{2}\sigma^2 \right)t
\right\}.
\end{equation}
这是布莱克-斯科尔斯-墨顿期权公式中使用的资产价格模型。
\end{definition}
\begin{problem}[(波动率的估计)]\label{pr:volotility_estimation}
给定 $0\le T_1\le T_2$,
观测几何布朗运动 $S(t)$, $T_1\le t\le T_2$.
假设 $S(t)$ 服从具有常数波动率 $\sigma$ 的几何布朗运动,
怎样估计波动率?
\end{problem}
\begin{hint}\label{hnt:pr:volotility_estimation}
此问题的解答可以参考书 \cite{shreve_stochastic_2008} 中的第 87 页.
\end{hint}
% \noindent\faHandPointerO ~\hyperlink{target:pr:volotility_estimation}{前往答案}
% \noindent\faHandPointerO ~\hyperref[pr:volotility_estimation]{返回问题~\ref{pr:volotility_estimation}}
% \hypertarget{target:pr:volotility_estimation}{}
% \begin{solution}\label{sol:pr:volotility_estimation}
%
% \end{solution}
\subsection{首达时间分布}%
\label{sub:shou_da_shi_jian_fen_bu_}
给定整数 $m$,
随机游动首次达到水平 $m$ 的时刻记为 $\tau_m$, 即:
\begin{equation}\label{eq:first_passage_time}
\tau_m = \min \{n; M_n = m\}.
\end{equation}
如果随机游动永远达不到 $m$,
我们定义 $\tau_m = \infty$.
随机变量 $\tau_m$ 是一个停时,
称为{\bf 随机游动}关于水平 $m${\bf 首达时间}
给定实数 $m$,
布朗运动首次达到水平 $m$ 的时刻记为 $\tau_m$, 即:
\begin{equation}\label{eq:first_passage_time_brownian}
\tau_m = \min \{t; W(t) = m\}.
\end{equation}
如果布朗运动永远达不到 $m$,
我们定义 $\tau_m = \infty$.
随机变量 $\tau_m$ 是一个停时,
称为{\bf 布朗运动}关于水平 $m${\bf 首达时间}
\begin{lemma}[和对称随机游动相关的一个鞅]\label{lem:bookI_5_2_1}
$M_n$ 是一个对称随机游动。给定 $\sigma$ 并定义过程:
\begin{equation}\label{eq:temp}
S_n = e^{\sigma M_n}\left( \frac{2}{e^{\sigma}+e^{-\sigma}}\right)^n
\end{equation}
$S_n$, $n = 0, 1, 2, \ldots$,
是一个鞅。
\end{lemma}
\begin{equation}\label{eq:temp_2}
\mathbb{E}
\left[
\mathbb{I}_{\{\tau_m < \infty \}}
e^{\sigma m}\left( \frac{2}{e^{\sigma}+e^{-\sigma}}\right)^{\tau_m}
\right]
=1.
\end{equation}
\begin{theorem}[指数鞅]\label{thm:bookII_3_6_1}
\footnote{此定理对应于书 \cite{shreve_stochastic_2008} 中的定理 3.6.1.}
$W(t)$, $t\ge 0$, 是一个布朗运动,
$\mathcal{F}$, $t\ge 0$ 是关于该布朗运动的域流。
给定 $\sigma$ 并定义过程:
\begin{equation}\label{eq:exp_martingale_brownian}
Z(t) =
\exp \left\{
\sigma W(t)+ - \frac{1}{2}\sigma^2 t
\right\}.
\end{equation}
则此过程是一个鞅。
\end{theorem}
% \pagebreak
\begin{theorem}[首达时间几乎必然有限]\label{thm:bookI_5_2_2}
\footnote{此定理即书 \cite{shreve_stochastic_2004} 中的定理 5.2.2.}
$m$ 是任一非零整数。
对称随机游动几乎必然达到水平 $m$,
即水平 $m$ 的首达时间 $\tau_m$ 几乎必然有限。
\end{theorem}
\begin{theorem}[计算 $ \alpha^{\tau_m}$ 的期望]\label{thm:bookI_5_2_3}
\footnote{此定理即书 \cite{shreve_stochastic_2004} 中的定理 5.2.3.}
$m$ 是非零整数。
对称随机游动的首达时间 $\tau_m$ 满足:
\begin{equation}\label{eq:expectation_alpha_tau_m}
\mathbb{E}\left[ \alpha^{\tau_m}\right]
=
\left( \frac{1 - \sqrt{1-\alpha^2} }{\alpha} \right)^{|m|},
\quad \forall \alpha\in (0,1).
\end{equation}
\end{theorem}
\begin{theorem}[首达时间必然有限及其分布的拉普拉斯变换]\label{thm:bookII3_6_2}
对于 $m\in \mathbb{R}$,
布朗运动关于水平 $m$ 的首达时间几乎必然有限,
并且其分布的拉普拉斯变换为:
\begin{equation}\label{eq:first_passage_time_laplace}
\mathbb{E}[\exp\{-\alpha \tau_m\}]
=
\exp\left\{-|m|\sqrt{2\alpha} \right\},
\quad \forall \alpha>0.
\end{equation}
\end{theorem}
\section{伊藤积分}%
\label{sec:yi_teng_ji_fen_}
\subsection{简单被积函数的伊藤积分}%
\label{sub:jian_dan_bei_ji_han_shu_de_yi_teng_ji_fen_}
$\varPi = \{t_0,t_1,\ldots,t_n\}$
$[0,T]$ 的一个分划,即
$0 = t_0 < t_1 < \ldots < t_n = T$.
假定 $\Delta(t)$ 在每个子区间 $[t_j, t_{j+1})$ 中是常量。
这样的过程 $\Delta(t)$ 称为{\bf 简单过程}
\begin{problem}[(简单过程的伊藤积分)]\label{pr:ito_integral_simple}
记简单过程的伊藤积分为
\begin{equation}\label{eq:ito_integral_simple}
I(t) = \int_0^t \Delta(u) \mathop{}\!\mathrm{d} W(u).
\end{equation}
怎样定义简单过程的伊藤积分?
\end{problem}
\begin{hint}\label{hnt:pr:ito_integral_simple}
此问题的解答可以参考书 \cite{shreve_stochastic_2008} 中的第 102 页.
\end{hint}
% \noindent\faHandPointerO ~\hyperlink{target:pr:ito_integral_simple}{前往答案}
% \noindent\faHandPointerO ~\hyperref[pr:ito_integral_simple]{返回问题~\ref{pr:ito_integral_simple}}
% \hypertarget{target:pr:ito_integral_simple}{}
% \begin{solution}\label{sol:pr:ito_integral_simple}
%
% \end{solution}
\begin{theorem}[鞅性]\label{thm:bookII4_2_1}
\footnote{此定理对应于书 \cite{shreve_stochastic_2008} 中的定理 4.2.1.}
简单过程的伊藤积分是一个鞅。
\end{theorem}
\begin{theorem}[伊藤等距]\label{thm:bookII_4_2_2}
\footnote{此定理对应于书 \cite{shreve_stochastic_2008} 中的定理 4.2.2.}
简单过程的伊藤积分满足:
\begin{equation}\label{eq:ito_integral_simple_var}
\mathbb{E}[I^2(t)] =
\mathbb{E} \left[\int_0^t \Delta^2(u) \mathop{}\!\mathrm{d}(u)
\right].
\end{equation}
\end{theorem}
\begin{theorem}[伊藤积分的二次变差]\label{thm:bookII_4_2_3}
\footnote{此定理对应于书 \cite{shreve_stochastic_2008} 中的定理 4.2.3.}
截至时刻 $t$,
简单过程的伊藤积分累积的二次变差为:
\begin{equation}\label{eq:ito_integral_simple_quadratic}
[I,I](t) =
\int_0^t \Delta^2(u) \mathop{}\!\mathrm{d}(u).
\end{equation}
\end{theorem}
\pagebreak
\subsection{一般被积函数的伊藤积分}%
\label{sub:yi_ban_bei_ji_han_shu_de_yi_teng_ji_fen_}
对于一般被积函数 $\Delta(t)$,
我们可以选取一列简单过程 $\Delta_n(t)$,
$n\to\infty$ 时收敛于连续变化的 $\Delta(t)$.
一般函数的伊藤积分定义为:
\begin{equation}\label{eq:ito_integral_general}
\int_0^t \Delta(u) \mathop{}\!\mathrm{d} W(u)
=
\lim_{n \to \infty} \int_0^t \Delta_n(u) \mathop{}\!\mathrm{d} W(u)
\end{equation}
这一积分继承了简单过程伊藤积分的大部分性质。
\subsection{伊藤---德布林公式}%
\label{sub:yi_teng_de_bu_lin_gong_shi_}
% \begin{definition}[伊藤---德布林公式]\label{def:ito_doeblin}
% \footnote{此定义在书 \cite{shreve_stochastic_2008} 中的第 110 页。}
% {\bf 微分形式的伊藤---德布林公式}:
% \begin{equation}\label{eq:ito_doeblin_dif}
% \mathop{}\!\mathrm{d} f(W(t))
% =
% f'(W(t)) \mathop{}\!\mathrm{d} W(t) +
% \frac{1}{2} f''(W(t)) \mathop{}\!\mathrm{d} t,
% \end{equation}
% 两边积分,我们得到
% {\bf 积分形式的伊藤---德布林公式}:
% \begin{equation}\label{eq:ito_doeblin_integral}
% f(W(t)) - f(W(0))
% =
% \int_0^t f'(W(u)) \mathop{}\!\mathrm{d}W(u) +
% \frac{1}{2} \int_0^t f''(W(u)) \mathop{}\!\mathrm{d}u.
% \end{equation}
% \end{definition}
%
\begin{theorem}[关于布朗运动的伊藤---德布林公式]\label{thm:bookII_4_4_1}
\footnote{此定理对应于书 \cite{shreve_stochastic_2008} 中的定理 4.4.1.}
设函数 $f(t,x)$ 的偏导数
$f_t(t,x)$, $f_x(t,x)$, 和 $f_{x x}(t,x)$
都有定义并且连续,
$W(t)$ 是布朗运动,
则对于每个 $T\ge 0$, 有:
\begin{multline}\label{eq:bookII_4_4_3}
f(T,W(t)) = f(0,W(0) + \int_0^T f_t(t,W(t)) \mathop{}\!\mathrm{d}t\\
\int_0^T f_x(t,W(t)) \mathop{}\!\mathrm{d}W(t) +
\frac{1}{2} \int_0^T f_{x x}(t,W(t)) \mathop{}\!\mathrm{d}t.
\end{multline}
\end{theorem}
\begin{definition}[伊藤过程]\label{def:bookII_4_4_3}
\footnote{此定义对应于书 \cite{shreve_stochastic_2008} 中的定义 4.4.3.}
$W(t)$, $t\ge 0$ 是布朗运动,
$ \mathcal{F}$, $t\ge 0$ 是相应的域流。
{\bf 伊藤过程}是以下形式的随机过程:
\begin{equation}\label{eq:ito_process}
X(t) = X(0) + \int_{0}^{u} \Delta(u) \mathop{}\!\mathrm{d}W(u) +
\int_0^t \varTheta(u) \mathop{}\!\mathrm{d}u,
\end{equation}
其中 $X(0)$ 非随机,
$\Delta(u)$$\varTheta(u)$ 是适应的随机过程。
\end{definition}
\pagebreak
到目前为止,
我们讨论了关于时间的积分,
以及伊藤积分(关于布朗运动的积分),
下面我们定义关于伊藤过程的积分。
\begin{definition}[关于伊藤过程的积分]\label{def:bookII_4_4_5}
\footnote{此定义对应于书 \cite{shreve_stochastic_2008} 中的定义 4.4.5.}
$X(t)$, $t\ge 0$ 是伊藤过程,
$\Gamma(t), t\ge 0$ 是一个适应过程。
{\bf 关于伊藤过程的积分} 定义为:
\begin{equation}\label{eq:bookII_4_4_20}
\int_{0}^{t} \Gamma(u) \mathop{}\!\mathrm{d}{X(u)}
=
\int_{0}^{t} \Gamma(u)\Delta(u) \mathop{}\!\mathrm{d}{W(u)} +
\int_{0}^{t} \Gamma(u) \varTheta(u) \mathop{}\!\mathrm{d}{u}.
\end{equation}
\end{definition}
\begin{theorem}[关于伊藤过程的伊藤---德布林公式]\label{thm:bookII_4_4_6}
\footnote{此定理对应于书 \cite{shreve_stochastic_2008} 中的定理 4.4.6.}
设函数 $f(t,x)$ 的偏导数
$f_t(t,x)$, $f_x(t,x)$, 和 $f_{x x}(t,x)$
都有定义并且连续,
$X(t)$ 是伊藤过程,
则对于每个 $T\ge 0$, 有:
\begin{multline}\label{eq:bookII_4_4_16}
f(T,X(t)) = f(0,W(0) + \int_0^T f_t(t,X(t)) \mathop{}\!\mathrm{d}t\\
\int_0^T f_x(t,X(t)) \mathop{}\!\mathrm{d}X(t) +
\frac{1}{2} \int_0^T f_{x x}(t,W(t)) \mathop{}\!\mathrm{d}[X,X](t).
\end{multline}
\end{theorem}
\pagebreak
{\bf 微分形式的伊藤---德布林公式}
\begin{equation}\label{eq:bookII_4_4_23}
\mathop{}\!\mathrm{d} f(t, X(t))
=
f_t(t,X(t)) \mathop{}\!\mathrm{d}t +
f_x(t,X(t)) \mathop{}\!\mathrm{d}X(t) +
\frac{1}{2} f_{x x}(t,W(t))
\mathop{}\!\mathrm{d} X(t) \mathop{}\!\mathrm{d} X(t).
\end{equation}
\begin{problem}[(广义几何布朗运动)]\label{pr:bookII_4_4_8}
$W(t), t\ge 0$ 是布朗运动,
$\mathcal{F}, t\ge 0$ 是相应的域流,
$\alpha(t)$$\sigma(t)$ 是适应过程。
定义伊藤过程
\begin{equation}\label{eq:bookII_4_4_25}
X(t) = \int_{0}^{t} \sigma(s) \mathop{}\!\mathrm{d}{W(s)} +
\int_{0}^{t} \left( \alpha(s) - \frac{1}{2}\sigma^2(s)\right)
\mathop{}\!\mathrm{d}{s}.
\end{equation}
对于资产价格过程 $S(t) = S(0) e^{X(t)}$,
计算 $\mathop{}\!\mathrm{d}S(t)$.
\end{problem}
\begin{hint}\label{hnt:pr:bookII_4_4_8}
此问题的解答可以参考书 \cite{shreve_stochastic_2008} 中的第 119 页。
\end{hint}
% \noindent\faHandPointerO ~\hyperlink{target:pr:bookII_4_4_8}{前往答案}
% \noindent\faHandPointerO ~\hyperref[pr:bookII_4_4_8]{返回问题~\ref{pr:bookII_4_4_8}}
% \hypertarget{target:pr:bookII_4_4_8}{}
% \begin{solution}\label{sol:pr:bookII_4_4_8}
%
% \end{solution}
\pagebreak
\section{布莱克---斯科尔斯---默顿方程}%
\label{sec:bu_lai_ke_si_ke_er_si_mo_dun_fang_cheng_}
这一节中,我们推导关于原生资产以几何布朗运动建模的期权定价的
布莱克---斯科尔斯---默顿偏微分方程。
其思想与第一卷第一章二叉树模型中的相同,
都是旨在确定为了完全对冲期权空头所需要的初始资本。
\subsection{资产组合价值的演化}%
\label{sub:zi_chan_zu_he_jie_zhi_de_yan_hua_}
设每个时刻 $t$,
投资者资产组合的价值为 $X(t)$,
这一组合投资于支付常数利率 $r$ 的货币市场账户以及股票市场,
股票价格过程服从几何布朗运动
\begin{equation}\label{eq:bookII_4_5_1}
dS(t) = \alpha S(t)dt + \sigma S(t)dW(t).
\end{equation}
假设在时刻 $t$ 投资者持有股票的份额为 $\Delta(t)$,
资产组合中余下部分 $X(t) - \Delta(t) S(t)$
投资于货币市场账户。
\begin{problem}[]\label{pr:bookII_page_124}
计算 $ \mathop{}\!\mathrm{d}X(t)$,
$\mathop{}\!\mathrm{d}\left( e^{-rt}S(t)\right)$,
以及
$\mathop{}\!\mathrm{d}\left( e^{-rt}X(t)\right)$.
\end{problem}
\begin{hint}\label{hnt:pr:bookII_page_124}
此问题的解答可以参考书 \cite{shreve_stochastic_2008} 中的第 124--125 页。
\end{hint}
% \noindent\faHandPointerO ~\hyperlink{target:pr:bookII_page_124}{前往答案}
% \noindent\faHandPointerO ~\hyperref[pr:bookII_page_124]{返回问题~\ref{pr:bookII_page_124}}
% \hypertarget{target:pr:bookII_page_124}{}
% \begin{solution}\label{sol:pr:bookII_page_124}
%
% \end{solution}
\subsection{期权价值的演化}%
\label{sub:qi_quan_jie_zhi_de_yan_hua_}
考虑在时刻 $T$ 支付为 $(S(T) - K)^+$ 的欧式看涨期权。
敲定价格 $K$ 是某个非负常数。
如果时刻 $t$ 的股价 $S(t) = x$,
我们用 $c(t,x)$ 表示看涨期权在时刻 $t$ 的价值。
\begin{problem}[]\label{pr:bookII_page_125}
计算 $\mathop{}\!\mathrm{d} c(t,S(t))$
$\mathop{}\!\mathrm{d} \left(e^{-rt} c(t,S(t))\right)$.
\end{problem}
\begin{hint}\label{hnt:pr:bookII_page_125}
此问题的解答可以参考书 \cite{shreve_stochastic_2008} 中的第 125--126 页。
\end{hint}
% \noindent\faHandPointerO ~\hyperlink{target:pr:bookII_page_125}{前往答案}
% \noindent\faHandPointerO ~\hyperref[pr:bookII_page_125]{返回问题~\ref{pr:bookII_page_125}}
% \hypertarget{target:pr:bookII_page_125}{}
% \begin{solution}\label{sol:pr:bookII_page_125}
%
% \end{solution}
\pagebreak
\subsection{演化相等}%
\label{sub:yan_hua_xiang_deng_}
对冲组合以初始资本 $X(0)$ 投资于股票和货币市场账户,
使得在每个时刻 $t\in [0,T]$,
资产组合价值 $X(t)$$c(t,S(t))$ 相同。
这等价于对所有 $t$,
$e^{-rt}X(t) = e^{-rt}c(t,S(t))$.
\begin{problem}[]\label{pr:bookII_page_126}
$e^{-rt}X(t) = e^{-rt}c(t,S(t))$ 推导出
布莱克---斯科尔斯---默顿偏微分方程
\begin{equation}\label{eq:bookII_4_5_14}
c_t(t,x) + rxc_x(t,x) + \frac{1}{2}\sigma^2 x^2 c_{x x}(t,x)
=
rc(t,x),
\quad \forall t\in[0,T], x\ge 0.
\end{equation}
\end{problem}
\begin{hint}\label{hnt:pr:bookII_page_126}
此问题的解答可以参考书 \cite{shreve_stochastic_2008} 中的第 126 页。
\end{hint}
% \noindent\faHandPointerO ~\hyperlink{target:pr:bookII_page_126}{前往答案}
% \noindent\faHandPointerO ~\hyperref[pr:bookII_page_126]{返回问题~\ref{pr:bookII_page_126}}
% \hypertarget{target:pr:bookII_page_126}{}
% \begin{solution}\label{sol:pr:bookII_page_126}
%
% \end{solution}
\chapter{二叉树无套利定价模型}%
\label{cha:the_binomial_no_arbitrage_pricing_model}
\section{单时段二叉树模型}%
\label{sec:one_period_binomial_model}
\subsection{预备知识}%
\label{sub:one_period_binomial_model_0}
\begin{definition}[套利]\label{def:arbitrage}
如果一个策略获得正收益的概率大于 0 而获得负收益的概率为 0,
则称其为 \gls{tl} (\gls{arbitrage})。
\end{definition}
\begin{definition}[单时段股票模型]\label{def:one_period_stock}
我们称研究单时段 (one-period) 问题的模型为单时段模型。
我们称该时段的起点为时刻 $n = 0$, 终点为时刻 $n = 1$.
$S_0$ 为每份股票在 $n = 0$ 的价格。
在时刻 $n = 1$ 抛掷一次硬币,
如果是正面,
则股票的价格涨至 $uS_0$,
反之,
跌至 $dS_0$.
其中 $\gls{u} > 1$\gls{ssyz} (\gls{up_factor}),
$\gls{d} < 1$\gls{xjyz} (\gls{down_factor}).
我们用 $\mathbb{C}$ (参见定义~\ref{def:coin_toss_space})
上的随机变量 $S_1(\omega)$ 来表示股票在 $n = 1$ 的价格:
\begin{align}\label{eq:rv_stock_value_period_1}
S_1(\omega) =
\begin{cases}
uS_0 & \text{如果~} \omega = H, \\
dS_0 & \text{如果~} \omega = T.
\end{cases}
\end{align}
为简化符号,
我们也记 $S_1(H)$$S_1^H$,
$S_1(T)$$S_1^T$.
\end{definition}
\begin{definition}[单时段衍生证券模型]\label{def:option_one_period}
我们称当 $n = 1$ 执行的衍生证券为单时段衍生证券。
单时段衍生证券当 $n = 1$ 的价值为随机变量 $S_1$ 的函数,
我们通常以随机变量 $V_1$ 来表示。
其当 $n = 0$ 的价格,
我们用 $V_0$ 来表示。
\end{definition}
\begin{definition}[单时段衍生证券的复制]\label{def:one_period_replication}
$n = 0$,
记一个包含股票和货币的投资组合为 $(\varDelta_0, \varXi_0)$,
其中 $\varDelta_0$ 为股票的份数,
$\varXi_0$ 为货币的数量。
记此组合当 $n = 0, 1$ 的价值分别为 $X_0$$X_1$.%
\footnote{$n = 0$, $X_0$ 为数, $X_1$ 为随机变量。}
记单时段衍生证券当 $n = 1$ 的价值为 $V_1$
$n = 1$,
如果 $X_1 = V_1$,
我们称此组合为该衍生证券的复制 (replication).
\end{definition}
\begin{definition}[单时段欧式看涨期权]\label{def:eco_one_period}
如果一个期权赋予持有人当 $n = 1$\gls{qdjg} (\gls{strike_price})
$\gls{K}$
购买一份股票的权利(但非义务),
我们称之为单时段\gls{oskzqq} (\gls{european_call_option}).
$n = 1$,
$V_1(\omega) = (S_1(\omega) - K)^+$,
$\omega \in \{H, T\}$.
为简化符号,
在不影响理解的情况下,
我们也记 $V_1(\omega)$$V_1^{\omega}$.
\end{definition}
\begin{definition}[单时段欧式看跌期权]\label{def:epo_one_period}
如果一个期权赋予持有人当 $n = 1$\gls{qdjg}
$\gls{K}$
卖出一份股票的权利(但非义务),
我们称之为单时段\gls{oskdqq} (\gls{european_put_option}).
$n = 1$,
$V_1(\omega) = (\gls{K} - S_1(\omega))^+$,
$\omega \in \{H, T\}$.
为简化符号,
在不影响理解的情况下,
我们也记 $V_1(\omega)$$V_1^{\omega}$.
\end{definition}
\subsection{关键问题}%
\label{sub:sec_1_1_problem_to_solve}
我们先来看一个典型的单时段期权定价问题。
在这个问题中,
我们基于如下假设:
\begin{itemize}
\item 股票份额可以无限细分和买卖;
\item 投资与借贷的利率相同;
\item 股票的买卖价格一致 (即买入与卖出的价差为 0);
\item 当时刻 $n = 1$, 股票价格只有两个值,即 $S_1(H)$$S_1(T)$.
\end{itemize}
\begin{figure}[htpb]
\centering
\includegraphics[width=0.9\textwidth]{image/BT-1-UC.pdf}
\caption{单时段欧式看涨期权的定价 (问题~\ref{pr:one_period} 参考图)}
\label{fig:pr:one_period}
\end{figure}
\begin{problem}[(单时段欧式看涨期权的定价)]\label{pr:one_period_Delta_X}
考虑图~\ref{fig:pr:one_period} 中的一个单时段模型。
\footnote{
此问题来源于书~\cite{shreve_stochastic_2004} 中例 1.1.1.
}
$S_0 = 4$, $u = 2$, $d = 0.5$, $r = 0.25$.
设单时段欧式看涨期权的\gls{qdjg} $K = 5$.
假设我们时刻 $n = 0$ 投入的资金为 $X_0$,
如果我们买入 $\varDelta_0$ 数量的股票,
则我们在时刻 1 的财富是如下的一个随机变量:
\begin{align}\label{eq:one_period_Delta_X_pv}
X_1(\omega) =
\begin{cases}
\varDelta_0 S_1(H) + (1 + r) (X_0 - \varDelta_0 S_0), &
\text{如果~} \omega = H, \\
\varDelta_0 S_1(T) + (1 + r) (X_0 - \varDelta_0 S_0), &
\text{如果~} \omega = T.
\end{cases}
\end{align}
记一个包含股票和货币的投资组合为 $(\varDelta_0, \varXi_0)$,
其中 $\varDelta_0$ 为股票的份数,
$\varXi_0$ 为货币的数量。
记此组合当 $n = 0, 1$ 的价值分别为 $X_0$$X_1$.
\begin{enumerate}
% next \item 证明期权的方差大于股票的方差;
\item
在时刻 $n = 0$, 计算一份欧式看涨期权在时刻 $n = 1$ 的价值 $V_1$.
\item\label{itm:pr:one_period_Delta_X_2}
使得 $X_1 = V_1$$(X_0, \varDelta_0)$ 是否存在?
如果存在,请求出 $(X_0, \varDelta_0)$ 的值。
\item\label{itm:pr:one_period_Delta_X_4}
证明此期权的 \emph{无套利价格}~为 $X_0$.
\end{enumerate}
\end{problem}
\bigskip
% \noindent
% \faHandPointerO ~
% \hyperref[hnt:pr:one_period_Delta_X]{前往答案}
\begin{hint}\label{hnt:pr:one_period_Delta_X}
参考书~\cite{shreve_stochastic_2004} 中例 1.1.1,
以及关于从方程组求出的 $X_0$ 无套利的阐述。
\end{hint}
\bigskip
\begin{problem}[(单时段欧式看涨期权的定价---方法二)]\label{pr:one_period}
考虑图~\ref{fig:pr:one_period} 中的一个单时段模型。
$S_0 = 4$, $u = 2$, $d = 0.5$, $r = 0.25$.
设单时段欧式看涨期权的\gls{qdjg} $\gls{K} = 5$.
$n = 0$,
记一个包含股票和货币的投资组合为 $(\varDelta_0, \varXi_0)$,
其中 $\varDelta_0$ 为股票的份数,
$\varXi_0$ 为货币的数量。
记此组合当 $n = 0, 1$ 的价值分别为 $X_0$$X_1$.
\begin{enumerate}
% next \item 证明期权的方差大于股票的方差;
\item
$n = 0$, 计算一份欧式看涨期权当 $n = 1$ 的价值 $V_1$.
\item\label{itm:pr:one_period_2}
使得 $X_1 = V_1$ 的投资组合 $(\varDelta_0, \varXi_0)$ 是否存在?
如果存在,请求出此组合。
\item
求出此投资组合当 $n = 0$ 的价值 $X_0$
\item\label{itm:pr:one_period_4}
证明此期权的 \emph{无套利价格}~为 $X_0$.
\end{enumerate}
\end{problem}
\bigskip
% \noindent
% \faHandPointerO ~
% \hyperref[sol:pr:one_period]{前往答案}
此问题的 4 个小问给出了另外一种方法来
求解 \emph{欧式看涨期权在时刻 $0$ 的无套利价格}
在第~\ref{itm:pr:one_period_2} 个小问中,
也可以用更简洁和直观的矩阵运算来求解复制期权的组合
(参见练习~\ref{pr:matrix_approach})。
这个方法在本质上和书~\cite{shreve_stochastic_2004} 中例 1.1.1 的方法是一致的。
注意到期初财富 $X_0$, 股票数量 $\varDelta_0$, 和货币数量 $\varXi_0$ 这三个量中,
任意两个量都唯一确定了当 $n = 0$ 的投资组合。
本讲义中的方法用 $(\varDelta_0, \varXi_0)$ 来表示投资组合,
而书~\cite{shreve_stochastic_2004} 中例 1.1.1 用 $(\varDelta_0, X_0)$ 表示。
\bigskip
\begin{definition}[单时段模型的矩阵表示]\label{def:notation_matrix_1}
我们用如下 $2 \times 1$ 矩阵分别表示
$S_1(\omega)$, $V_1(\omega)$, $X_1(\omega)$, $\omega\in\{H, T\}$:
\begin{equation}\label{eq:matrix_S_V_X_1}
\begin{pmatrix} S_1(H) \\ S_1(T) \end{pmatrix},
\begin{pmatrix} V_1(H) \\ V_1(T) \end{pmatrix},
\begin{pmatrix} X_1(H) \\ X_1(T) \end{pmatrix}.
\end{equation}
$M_1$$2 \times 1$ 矩阵
\begin{equation}\label{eq:matrix_M_1}
\begin{pmatrix} 1 + r \\ 1 + r \end{pmatrix}.
\end{equation}
记投资组合为 $2 \times 1$ 矩阵
\begin{equation}\label{eq:matrix_portfolio}
\begin{pmatrix} \varDelta_0 \\ \varXi_0 \end{pmatrix}.
\end{equation}
由~\eqref{eq:matrix_portfolio} 表示的投资组合当 $n = 0$ 的价值为
\begin{equation}\label{eq:matrix_X_0_computation}
X_0
=
\begin{pmatrix} S_0 & 1 \end{pmatrix}
\begin{pmatrix} \varDelta_0 \\ \varXi_0 \end{pmatrix}.
\end{equation}
由~\eqref{eq:matrix_portfolio} 表示的投资组合当 $n = 1$ 的价值为
\begin{equation}\label{eq:matrix_X_1_computation}
X_1
=
\begin{pmatrix} S_1 & M_1 \end{pmatrix}
\begin{pmatrix} \varDelta_0 \\ \varXi_0 \end{pmatrix}.
\end{equation}
$X_1 = V_1$ 等同于
\begin{equation}\label{eq:system_portfolio_derivative}
\begin{pmatrix} S_1 & M_1 \end{pmatrix}
\begin{pmatrix} \varDelta_0 \\ \varXi_0 \end{pmatrix}
=
V_1,
\end{equation}
也可以写成
\begin{equation}\label{eq:system_portfolio_derivative_matrix}
\begin{pmatrix}
S_1(H) & 1 + r \\
S_1(T) & 1 + r
\end{pmatrix}
\begin{pmatrix} \varDelta_0 \\ \varXi_0 \end{pmatrix}
=
\begin{pmatrix} V_1(H) \\ V_1(T) \end{pmatrix}.
\end{equation}
\end{definition}
\bigskip
\begin{problem}\label{pr:matrix_approach}
用矩阵运算求解问题~\ref{pr:one_period}
中的小问~\ref{itm:pr:one_period_2},
并证明此组合一定存在。
\end{problem}
% \bigskip
%
% \begin{hint}\label{hnt:pr:matrix_approach}
% \end{hint}
% \noindent
% \faHandPointerO ~
% \hyperref[sol:pr:matrix_approach]{前往答案}
\bigskip
在问题~\ref{pr:one_period}中,
我们解决了一个较为特殊的期权定价问题。
下面我们将把这个问题推广到更一般的问题。
\begin{figure}[htpb]
\centering
\includegraphics[width=0.9\textwidth]{image/BT-1-general.pdf}
\caption{单时段一般衍生证券的定价
(问题~\ref{pr:one_period_general} 参考图)}
\label{fig:pr:one_period_general}
\end{figure}
\begin{problem}[(单时段一般衍生证券的定价)]\label{pr:one_period_general}
如图所示,
考虑一个单时段模型。
\gls{ssyz} $u$, 下降因子 $d$, 和利率 $r$ 满足
\begin{equation}\label{eq:dru_constraint}
0 < d < 1 + r < u.
\end{equation}
在时刻 1 抛掷一次硬币,
如果是正面,
则股票的价格涨至 $uS_0$,
反之,
跌至 $dS_0$.
此问题的样本空间为 $\mathbb{C}$ (参见定义~\ref{def:coin_toss_space})。
在时刻 1,
股票和衍生证券的价值分别为 $\mathbb{C}$ 上的随机变量 $S_1$$V_1$.
假设在时刻 0,
一位投资者的初始财富值为 $X_0$.
此投资者将 $X_0$ 投资于股票和货币市场。
如果投资于股票的份额为 $\varDelta_0$,
则此投资者的投资组合为 $(\varDelta_0, \varXi_0)$,
其中 $\varDelta_0$ 为股票的份数,
$\varXi_0 = X_0 - \varDelta_0 S_0$ 为现金头寸。
记此组合在时刻 1 的价值为 $X_1$.
\begin{enumerate}
% next \item 证明期权的方差大于股票的方差;
\item
$S_0$ 表示成 $S_1(H)$$S_1(T)$ 的贴现线性组合,
即:
\begin{equation}\label{eq:linear_combination_s}
S_0 = \frac{1}{1+r}
\left[ \tilde{p} S_1(H) + \tilde{q} S_1(T)\right],
\quad \tilde{p} + \tilde{q} = 1.
\end{equation}
求满足 \eqref{eq:linear_combination_s}$\tilde{p}, \tilde{q}$.
\item
证明存在唯一的一组 $X_0, \varDelta_0$,
使得 $X_1$ 能够复制 $V_1$.
并且有
\begin{equation}\label{eq:linear_combination_v}
X_0 = \frac{1}{1+r}
\left[ \tilde{p} V_1(H) + \tilde{q} V_1(T)\right].
\end{equation}
\item
证明此衍生证券的 \emph{无套利价格}~为 $X_0$.
\end{enumerate}
% \begin{enumerate}
% % next \item 证明期权的方差大于股票的方差;
% \item
% 使得 $X_1 = V_1$ 的投资组合 $(\varDelta_0, \varXi_0)$ 是否存在?
% 如果存在,请求出此组合。
% \item
% 求出此投资组合在时刻 0 的价值 $X_0$。
% \item
% 证明此衍生证券的 \emph{无套利价格}~为 $X_0$.
% \end{enumerate}
\end{problem}
% \bigskip
%
% \begin{hint}\label{hnt:pr:one_period_general}
% \end{hint}
% \noindent
% \faHandPointerO ~
% \hyperref[sol:pr:one_period_general]{前往答案}
\bigskip
如果 $u, d, r$ 满足式子 \eqref{eq:dru_constraint},
\eqref{eq:linear_combination_s} 存在唯一的解。
这个解在二叉树资产定价模型中,
有着非常重要的作用。
下面我们严格地定义它。
\begin{definition}[风险中性概率测度及风险中性期望]\label{def:risk_neutral_measure}
在一维抛掷硬币概率空间中,
如果结果为正面和反面的概率
\footnote{式子~\eqref{eq:risk_neutral_prob_def:risk_neutral_measure}
即书~\cite{shreve_stochastic_2004}中的~\eqref{eq:risk_neutral_prob}.}
分别为:
\begin{equation}\label{eq:risk_neutral_prob_def:risk_neutral_measure}
\tilde{p} = \frac{1+r-d}{u-d},
\quad
\tilde{q} = \frac{u-1-r}{u-d},
\end{equation}
其中\gls{ssyz} $u$, 下降因子 $d$, 和利率 $r$ 满足限制条件
\footnote{式子~\eqref{eq:dru_constaint:risk_neutral_measure}
即书~\cite{shreve_stochastic_2004}中的~\eqref{eq:dru_constraint}.}
:
\begin{equation}\label{eq:dru_constaint:risk_neutral_measure}
0 < d < 1 + r < u,
\end{equation}
则称此概率测度为
\gls{fxzxglcd} (\gls{risk-neutral_probability_measure}).
我们记此测度为
\gls{Pn},
记基于此测度的期望为
$\widetilde{\mathbb{E}}$.
\end{definition}
\begin{problem}\label{pr:dru_constraint_no_arbitrage}
证明如果式子~\eqref{eq:dru_constraint} 不满足,
则一定存在套利机会。
\end{problem}
\subsection*{答案}%
\label{sub:lian_xi_}
\noindent
% \faHandPointerO ~
\hyperref[pr:one_period_Delta_X]{返回问题~\ref{pr:one_period_Delta_X}}
\noindent
% \faHandPointerO ~
\hyperref[pr:one_period]{返回问题 \ref{pr:one_period}}
\begin{solution}[问题~\ref{pr:one_period} 单时段欧式看涨期权的定价]
\label{sol:pr:one_period}
\begin{enumerate}
\item
根据定义~\ref{def:one_period_stock},
\begin{align}\label{eq:sol_stock_value_period_1}
S_1(\omega) =
\begin{cases}
uS_0 = 8 & \text{如果~} \omega = H, \\
dS_0 = 2 & \text{如果~} \omega = T.
\end{cases}
\end{align}
根据定义~\ref{def:eco_one_period},
\begin{align}
V_1(H)
&= (S_1(H) - \gls{K})^+ \notag \\
&= (8 - 5)^+ \notag \\
&= 3 \notag
\end{align}
\begin{align}
V_1(T)
&= (S_1(T) - \gls{K})^+ \notag \\
&= (2 - 5)^+ \notag \\
&= 0 \notag
\end{align}
据此,$V_1$ 可以表示为
\begin{align}\label{eq:eco_value_period_1}
V_1(\omega) =
\begin{cases}
3 & \text{如果~} \omega = H, \\
0 & \text{如果~} \omega = T.
\end{cases}
\end{align}
\item
$X_1$ 可以表示为
\begin{align}\label{eq:pv_period_1}
X_1(\omega) =
\begin{cases}
\varDelta_0 S_1(H) + (1 + r) \varXi_0, &
\text{如果~} \omega = H, \\
\varDelta_0 S_1(T) + (1 + r) \varXi_0, &
\text{如果~} \omega = T.
\end{cases}
\end{align}
代入 $r = 1.25$,
并由式~\eqref{eq:sol_stock_value_period_1},
我们有
\begin{align}\label{eq:pv_period_1_2}
X_1(\omega) =
\begin{cases}
8 \varDelta_0 + 1.25 \varXi_0, &
\text{如果~} \omega = H, \\
2 \varDelta_0 + 1.25 \varXi_0, &
\text{如果~} \omega = T.
\end{cases}
\end{align}
由式~\eqref{eq:eco_value_period_1}
及~\eqref{eq:pv_period_1_2},
$X_1 = V_1$ 等价于
\begin{align*}%\label{eq:system_portfolio_derivative}
\begin{cases}
8 \varDelta_0 + 1.25 \varXi_0 = 3 \\
2 \varDelta_0 + 1.25 \varXi_0 = 0.
\end{cases}
\end{align*}
求解此方程组,
我们得到
\begin{align}\label{eq:system_portfolio_derivative_solution}
\begin{cases}
\varDelta_0 = 0.5 \\
\varXi_0 = -0.8.
\end{cases}
\end{align}
\item
由式~\eqref{eq:system_portfolio_derivative_solution},
我们有
\begin{align}\label{eq:pv_period_1_solution}
X_0
&= \varDelta_0 S_0 + \varXi_0 \notag \\
&= 1.2.
\end{align}
\item
$\gamma_0$ 表示期权在时刻 $0$ 的价格,
则原命题等价于
``如果 $\gamma_0 \neq 1.2$,
则存在套利策略''。
在时刻 0,
包含欧式看涨期权、股票、以及货币的投资组合可以表示为
$3 \times 1$ 矩阵
\begin{equation}\label{eq:portfolio_DSM}
\begin{pmatrix}
\varGamma_0 \\ \varDelta_0 \\ \varXi_0
\end{pmatrix},
\end{equation}
其中 $\varGamma_0$ 为期权的份额。
在此问题中,
一个交易策略可以由式~\eqref{eq:portfolio_DSM}
表示。
\begin{enumerate}
\item\label{itm:a}
如果 $\gamma_0 > 1.2$ (比如 $\gamma_0 = 1.21$),
考虑如下策略:
\begin{equation}\label{eq:portfolio_DSM_high}
\begin{pmatrix}
\varGamma_0 \\ \varDelta_0 \\ \varXi_0
\end{pmatrix}
=
\begin{pmatrix} -1 \\ 0.5 \\ -0.79 \end{pmatrix}.
\end{equation}
$n = 0$,
欧式看涨期权、股票、以及货币的单位价值可以表示为
$1 \times 3$ 矩阵
\begin{equation}\label{eq:price_DSM_0}
\begin{pmatrix} 1.21 & 4 & 1 \end{pmatrix}.
\end{equation}
由式~\eqref{eq:portfolio_DSM_high}
及~\eqref{eq:price_DSM_0},
我们有
\begin{equation}\label{eq:value_high_0}
X_0
=
\begin{pmatrix} 1.21 & 4 & 1 \end{pmatrix}
\begin{pmatrix} -1 \\ 0.5 \\ -0.79 \end{pmatrix}
=
0.
\end{equation}
$n = 1$,
如果 $\omega = H$,
欧式看涨期权、股票、以及货币的单位价值可以表示为
$1 \times 3$ 矩阵
\begin{equation}\label{eq:price_DSM_1_H}
\begin{pmatrix} 3 & 8 & 1.25 \end{pmatrix}.
\end{equation}
由式~\eqref{eq:portfolio_DSM_high}
及~\eqref{eq:price_DSM_1_H},
我们有
\begin{equation}\label{eq:value_high_H}
X_1(H)
=
\begin{pmatrix} 3 & 8 & 1.25 \end{pmatrix}
\begin{pmatrix} -1 \\ 0.5 \\ -0.79 \end{pmatrix}
=
0.0125.
\end{equation}
$n = 1$,
如果 $\omega = T$,
欧式看涨期权、股票、以及货币的单位价值可以表示为
$1 \times 3$ 矩阵
\begin{equation}\label{eq:price_DSM_1_T}
\begin{pmatrix} 0 & 2 & 1.25 \end{pmatrix}.
\end{equation}
由式~\eqref{eq:portfolio_DSM_high}
及~\eqref{eq:price_DSM_1_T},
我们有
\begin{equation}\label{eq:value_high_T}
X_1(T)
=
\begin{pmatrix} 0 & 2 & 1.25 \end{pmatrix}
\begin{pmatrix} -1 \\ 0.5 \\ -0.79 \end{pmatrix}
=
0.0125.
\end{equation}
根据式子~\eqref{eq:price_DSM_1_H}
及~\eqref{eq:price_DSM_1_T},
\begin{equation}\label{eq:pv_period_1_prob}
\mathbb{P}\{X_1 = 0.0125\} = 1.
\end{equation}
因此,
$\mathbb{P}\{X_1 > 0\} > 0$
$\mathbb{P}\{X_1 < 0\} = 0$.
又根据~\eqref{eq:value_high_0},
$X_0 = 0$.
所以,
交易策略~\eqref{eq:portfolio_DSM_high} 可以套利。
\item
如果 $\gamma_0 < 1.2$ (比如 $\gamma_0 = 1.19$),
考虑如下策略:
\begin{equation}\label{eq:sol_portfolio_DSM_low}
\begin{pmatrix}
\varGamma_0 \\ \varDelta_0 \\ \varXi_0
\end{pmatrix}
=
\begin{pmatrix} 1 \\ -0.5 \\ 0.81 \end{pmatrix}.
\end{equation}
用~\ref{itm:a} 中同样的方法,
我们可以证明此交易策略可以套利。
\end{enumerate}
\end{enumerate}
\end{solution}
\bigskip
\begin{solution}[问题~\ref{pr:matrix_approach}]\label{sol:pr:matrix_approach}
\noindent
% \faHandPointerO ~
\hyperref[pr:matrix_approach]{返回问题~\ref{pr:matrix_approach}}
$S_1(\omega)$, $\omega\in\{H, T\}$,
可以用如下 $1 \times 2$ 矩阵表示:
\begin{equation*}
\begin{pmatrix} S_1(H) \\ S_1(T) \end{pmatrix}.
\end{equation*}
$M_1$ 为矩阵
\begin{equation*}
\begin{pmatrix} 1 + r \\ 1 + r \end{pmatrix}.
\end{equation*}
记投资组合为
\begin{equation*}%\label{eq:matrix_portfolio}
\begin{pmatrix} \varDelta_0 \\ \varXi_0 \end{pmatrix}.
\end{equation*}
$V_1(\omega)$, $\omega\in\{H, T\}$,
可以用如下 $1 \times 2$ 矩阵表示:
\begin{equation*}
\begin{pmatrix} V_1(H) \\ V_1(T) \end{pmatrix}.
\end{equation*}
则方程组
\begin{align*}
\begin{cases}
8 \varDelta_0 + 1.25 \varXi_0 = 3 \\
2 \varDelta_0 + 1.25 \varXi_0 = 0.
\end{cases}
\end{align*}
等同于
\begin{equation*}
\begin{pmatrix} S_1 & M_1 \end{pmatrix}
\begin{pmatrix} \varDelta_0 \\ \varXi_0 \end{pmatrix}
=
V_1.
\end{equation*}
代入数值后,我们有
\begin{equation*}
\begin{pmatrix}
8 & 1.25 \\
2 & 1.25
\end{pmatrix}
\begin{pmatrix} \varDelta_0 \\ \varXi_0 \end{pmatrix}
=
\begin{pmatrix} 3 \\ 0 \end{pmatrix}.
\end{equation*}
因为系数矩阵可逆,所以有
\begin{equation*}
\begin{pmatrix} \varDelta_0 \\ \varXi_0 \end{pmatrix}
=
\begin{pmatrix}
8 & 1.25 \\
2 & 1.25
\end{pmatrix}^{-1}
\begin{pmatrix} 3 \\ 0 \end{pmatrix}.
\end{equation*}
\end{solution}
% \begin{solution}[问题~\ref{pr:one_period_general}]
% \label{sol:pr:one_period_general}
% \noindent
% \faHandPointerO ~
% \hyperref[pr:one_period_general]{返回问题~\ref{pr:one_period_general}}
% \begin{enumerate}
% \item 使得 $X_1 = V_1$ 的投资组合 $(\varDelta_0, \varXi_0)$ 是否存在
% 等价于方程组 \eqref{eq:system_portfolio_derivative_matrix}
% 是否有解。
% 由于
% \begin{equation*}
% 0 < S_1(T) < S_1(H),
% \end{equation*}
% 此方程组的系数矩阵一定可逆,
% 所以解一定存在。
% 我们有
% \begin{align}
% \begin{pmatrix} \varDelta_0 \\ \varXi_0 \end{pmatrix}
% &=
% \begin{pmatrix}
% S_1(H) & 1 + r \\
% S_1(T) & 1 + r
% \end{pmatrix}
% ^{-1}
% \begin{pmatrix} V_1(H) \\ V_1(T) \end{pmatrix}%
% \label{eq:system_portfolio_derivative_sol_0} \\
% &=
% \begin{pmatrix}
% \frac{V_1(H) - V_1(T)}{S_1(H) - S_1(T)} \\
% \\ % next: 调整间距
% \frac{V_1(T)S_1(H) - V_1(H)S_1(T)}{(1 + r)(S_1(H) - S_1(T))}
% \end{pmatrix}.\label{eq:system_portfolio_derivative_sol_1} \\
% &=
% \begin{pmatrix}
% \frac{V_1(H) - V_1(T)}{S_0(u - d)} \\
% \\ % next: 调整间距
% \frac{uV_1(T) - dV_1(H)}{(1 + r)(u - d)}
% \end{pmatrix}.\label{eq:system_portfolio_derivative_sol}
% \end{align}
% \item 由式 \eqref{eq:matrix_X_0_computation},
% 我们得到 $X_0$.
% \item 证明方法类似于问题 \ref{pr:one_period_general}
% 中小问~\ref{itm:pr:one_period_4}.
% \end{enumerate}
% \end{solution}
\section{多时段二叉树模型}%
\label{sec:multi_period_binomial_model}
\subsection{预备知识}%
\label{sub:multi_period_binomial_model_0}
\begin{definition}[多时段股票模型]\label{def:multi_period_stock}
我们已经在定义~\ref{def:one_period_stock}
中定义了单时段股票模型。
我们称研究多时段 (multi-period) 问题的模型为多时段模型。
设起点为 0, 终点为 $N$.
每当 $n = 1, 2, \ldots, N$, 我们抛掷一次硬币,
如果是正面,
则股票的价格涨至 $uS_{n-1}$,
反之,
跌至 $dS_{n-1}$,
其中 $S_n$$n$ 维抛掷硬币概率空间 (参见定义 \ref{def:multi_coin_space})
上的随机变量,
表示股票在时刻 $n$ 的价格。
\end{definition}
\begin{definition}[多时段欧式衍生证券模型]\label{def:multi_period_derivative}
我们称当且仅当时刻 $N$ 执行的衍生证券为多时段
\gls{osyszq} (\gls{european_derivative_security}).
多时段衍生证券在时刻 $N$ 的价值为随机变量 $S_N$ 的函数,
我们通常以随机变量 $V_N$ 来表示。
其当 $n = 0, 1, \ldots, N-1$ 的价格,
我们用
$n$ 维抛掷硬币概率空间 (参见定义 \ref{def:multi_coin_space})
上的随机变量 $V_n$ 来表示。
\end{definition}
\begin{definition}[多时段欧式衍生证券的复制]\label{def:multi_period_replication}
我们已经在定义~\ref{def:one_period_replication}
中定义了单时段衍生证券的复制。
$n = 0$,
包含股票和货币的投资组合 $(\varDelta_0, \varXi_0)$
的价值为
\begin{equation}\label{eq:pv_period_0}
X_0 = \varDelta_0 S_0 + \varXi_0.
\end{equation}
$n = 1, 2, \ldots, N-1$,
调整组合中股票和货币的数量到 $(\varDelta_n, \varXi_n)$.
$n = 1, 2, \ldots, N$,
时组合的价值为
\begin{equation}\label{eq:pv_period_n}
X_n = \varDelta_{n-1}S_n + \varXi_{n-1}(1 + r).
\end{equation}
如果 $X_N = V_N$,
我们称组合 $(\varDelta_0, \varXi_0)$
为该\gls{osyszq}的复制 (replication).
\end{definition}
\begin{definition}[多时段欧式看涨期权]\label{def:eco_multi_period}
如果一个期权赋予持有人在时刻 $N$\gls{qdjg} $\gls{K}$
购买一份股票的权利(但非义务),
我们称之为多时段欧式看涨期权。
\footnote{根据定义 \ref{def:multi_period_replication} 中省略上标的原则,
我们省略了它。}
在时刻 $N$,
$V_N(\omega) = (S_N(\omega) - \gls{K})^+$,
$\omega \in \Omega$,
其中 $\Omega$$N$ 维抛掷硬币样本空间。
为简化符号,
我们也记 $V_N(\omega)$$V_N^{\omega}$.
\end{definition}
\subsection{关键问题}%
\label{sub:sec_1_2_problem_to_solve}
\begin{figure}[htpb]
\centering
\includegraphics[width=0.9\textwidth]{image/BT-2.pdf}
\caption
{两时段一般欧式衍生证券的定价(问题 \ref{pr:two_period_general} 示意图)}
\label{fig:two_period_general}
\end{figure}
\begin{problem}[(两时段一般欧式衍生证券的定价)]\label{pr:two_period_general}
此问题是问题~\ref{pr:one_period_general} 在时间上的一个推广。
如图 \ref{fig:two_period_general} 所示,
我们用 $n = 0, 1, 2$ 来表示时刻,
同样地假设 $u$, $d$, 和 $r$ 满足限制条件~\eqref{eq:dru_constraint}.
在这个两时段模型中,
$n = 1$ 抛掷一次硬币,
如果是正面,
则股票的价格涨至 $uS_0$,
反之,
跌至 $dS_0$.
$t = 2$ 再抛掷一次硬币,
如果是正面,
则股票的价格涨至 $uS_1$,
反之,
跌至 $dS_1$.
此问题的样本空间为
\begin{equation*}
\mathbb{C}^{(2)} = \{HH, HT, TH, TT\}.
\end{equation*}
\gls{osyszq}当只当 $t = 2$ 才能执行,
其在执行时的价值为 $\mathbb{C}^{(2)}$ 上的随机变量 $V_2$.
$n = 0$,
假设我们持有一个包含股票和货币的投资组合 $(\varDelta_0, \varXi_0)$,
其价值为 $X_0$.
在抛掷硬币第 1 次后,
此组合的价值为 $X_1$.
此时可以调整组合至 $(\varDelta_1, \varXi_1)$.
注意到 $X_1$$\varDelta_1$ 都是依赖于第 1 次抛掷结果的随机变量,
并且 $\varXi_1$$\varDelta_1$$(\varDelta_0, \varXi_0)$ 唯一确定。
在抛掷硬币第 2 次后,
此组合的价值为 $X_2$.
\begin{enumerate}
% next \item 证明期权的方差大于股票的方差;
\item
使得 $X_2 = V_2$ 的初始投资组合
$(\varDelta_0, \varXi_0)$ 是否存在?
如果存在,请求出此组合以及 $\varDelta_1$.
\item
求出此投资组合当 $n = 0$ 的价值 $X_0$,
以及当 $n = 1$ 的价值 $X_1$.
\item
证明此衍生证券的 \emph{无套利价格}~为 $X_0$.
\end{enumerate}
\end{problem}
将两时段问题推广到 $N$-时段,
相似的结果同样成立。
\begin{theorem}[多时段二叉树模型中的复制]\label{thm:bookI_1_2_2}
\footnote{此定理即书 \cite{shreve_stochastic_2004} 中的定理 1.2.2.}
考虑一个 \gls{nsdecs}
\begin{equation}\label{eq:risk_neutral_prob}
\tilde{p} = \frac{1+r-d}{u-d},
\quad
\tilde{q} = \frac{u-1-r}{u-d},
\end{equation}
其中\gls{ssyz} $u$, 下降因子 $d$, 和利率 $r$ 满足
限制条件~\eqref{eq:dru_constraint}.
我们用概率空间
$\left(\mathbb{C}^{(N)}, \mathbb{P}\right)$
上的随机变量 $V_N$ 表示\gls{osyszq}在到期时刻 $N$ 时的价值。
% next: 欧式,更严格
% next: Better than the translation.
% next: 没有了译本中的 "其中 $n$ 在 0 到 N - 1 变化"。
$n = N-1, N-2, \ldots, 0$,
\footnote{$\omega^{(n)}$ 即书
中的 $\omega_1\omega_2\cdots\omega_n$;
参见定义 \ref{def:multi_coin_space}.}
我们按时间倒向递归定义随机变量序列
\begin{equation}\label{eq:derivative_value_n}
V_n\left( \omega^{(n)}\right) =
\frac{1}{1+r}
\left[
\tilde{p} V_{n+1}\left( \omega^{(n)}H\right)
+
\tilde{q} V_{n+1}\left( \omega^{(n)}T\right)
\right],
\end{equation}
以及随机变量序列
\begin{equation}\label{eq:stock_quantity_n}
\varDelta_n\left( \omega^{(n)}\right) =
\frac{
V_{n+1}\left( \omega^{(n)}H\right)
-
V_{n+1}\left( \omega^{(n)}T\right)
}{
S_{n+1}\left( \omega^{(n)}H\right)
-
S_{n+1}\left( \omega^{(n)}T\right)
}.
\end{equation}
接下来我们让 $X_0 = V_0$,
并且按时间前向递归定义在时刻 $n = 0, 1, \ldots, N-1$
的投资组合价值
\begin{equation}\label{eq:wealth_equation}
X_{n+1} = \varDelta_{n}S_{n+1} + (1 + r)(X_n - \varDelta_n S_n).
\end{equation}
在以上设定下,
我们可以证明:
\begin{equation}\label{eq:equality_portfolio_derivative}
X_N\left( \omega^{(N)}\right)
=
V_N\left( \omega^{(N)}\right),
\quad \forall \; \omega^{(N)}.
\end{equation}
\end{theorem}
我们称式子 \eqref{eq:wealth_equation}
\gls{cffc} (\gls{wealth_equation}).
\begin{definition}[在各个时刻欧式衍生证券的价格]\label{def:bookI_1_2_3}
\footnote{此定义即书 \cite{shreve_stochastic_2004} 中的定义 1.2.3.}
对于 $n = 1, 2, \ldots, N$,
我们把定理~\ref{thm:bookI_1_2_2} 中的随机变量
$V_n\left( \omega^{(n)}\right)$
定义为前 $n$ 次抛掷结果为 $\left( \omega^{(n)}\right)$
在时刻 $n$ 的衍生证券价格。
在时刻 0 的衍生证券价格为 $V_0$.
\end{definition}
% \begin{proof}[定理~\ref{thm:bookI_1_2_2}]
% dd
% \end{proof}
\begin{figure}[htpb]
\centering
\includegraphics[width=0.9\textwidth]{image/BT-3-lookback.pdf}
\caption{计算回望期权的价格 (例 \ref{ex:lookback_option})}
\label{fig:ex:lookback_option}
\end{figure}
\begin{example}\label{ex:lookback_option}
如图 \ref{fig:ex:lookback_option} 所示:
$S_0 = 4$, $u = 2$, $d = \frac{1}{2}$,
又假设利率 $r = \frac{1}{4}$.
考虑一个
\gls{hwqq} (\gls{lookback_option}),
其在时刻 3 的
\gls{zf} (\gls{payoff})
\begin{equation}\label{eq:lookback_option}
V_3 = \max_{0\leq n\leq 3} (S_n - S_3).
\end{equation}
请计算\gls{hwqq}在各个时刻的价格。
\end{example}
\section{模型的计算}%
\label{sec:computation}
待完成
\section{}%
\label{sec:martingales}
\subsection{预备知识}%
\label{sub:martingales_0}
\begin{definition}
[二叉树模型中的适应随机过程]\label{def:adapted_stochastic_process}
考虑 \gls{nsdecs}
对于随机过程 $M_0,M_1,\ldots,M_N$
如果每个 $M_n$, $n = 1, \ldots, N$
只依赖前 $n$ 次抛掷硬币($M_0$ 为常量),
我们称这样的随机过程为
\gls{sysjgc} (\gls{adapted_stochastic_process}).
\end{definition}
\begin{definition}[贴现股票价格过程]\label{def:npv_stock_price}
考虑 \gls{nsdecs}
我们称
\begin{equation}\label{eq:stock_price_process_npv}
\frac{S_n}{(1+r)^n},
\quad n = 0, 1, \ldots, N.
\end{equation}
\gls{txgpjggc} (\gls{discounted_stock_price_process}).
\end{definition}
\begin{definition}[贴现财富过程]\label{def:npv_wealth_process}
考虑 \gls{nsdecs}
$X_0$ 为实数,
基于式子 \eqref{eq:wealth_equation_2},
按时间前向递归定义在时刻 $n = 0, 1, \ldots, N-1$
的投资组合价值
\begin{equation}%
\label{eq:wealth_equation_2}
X_{n+1} = \varDelta_{n}S_{n+1} + (1 + r)(X_n - \varDelta_n S_n),
\end{equation}
其中 $\varDelta_n$, $n = 0, 1, \ldots, N-1$,
\gls{sysjgc}
我们称
\begin{equation}\label{eq:npv_derivative_value}
\frac{X_n}{(1+r)^n},
\quad n = 0, 1, \ldots, N.
\end{equation}
\gls{txcfgc} (\gls{discounted_wealth_process}).
\end{definition}
\begin{definition}[贴现衍生证券价格过程]%
\label{def:npv_derivative_price_process}
考虑 \gls{nsdecs}
我们称
\begin{equation}\label{eq:npv_derivative_price_process}
\frac{V_n}{(1+r)^n},
\quad n = 0, 1, \ldots, N.
\end{equation}
\gls{txyszqjggc} (\gls{discounted_derivative_price_process}).
\end{definition}
\subsection{关键问题}%
\label{sub:martingales}
\begin{introduction}
\item 鞅的引入及定义
\item 鞅的多步超前性质
\item \gls{txgpjggc}是鞅
\item 贴现财富过程是鞅
\item 贴现衍生证券价格过程是鞅
\item 现金流定价
\end{introduction}
将定理 \ref{thm:bookI_1_2_2} 中的
式子 \eqref{eq:derivative_value_n} 做一个变换,
可以得到如下等式:
\begin{equation}\label{eq:derivative_value_n_martingale}
\frac{V_n}{(1+r)^n} =
\widetilde{\mathbb{E}}_n\left[
\frac{V_{n+1}}{(1+r)^{n+1}} \right], \quad n = 0, 1, \ldots, N-1.
\end{equation}
如果让 \[
M_n = \frac{V_n}{(1+r)^n}, \quad n = 0, 1, \ldots, N-1,
\]
则我们有
\begin{equation*}
M_n = \widetilde{\mathbb{E}}_n[M_{n+1}],\quad n = 0, 1, \ldots, N-1.
\end{equation*}
在我们研究的二叉树资产定价模型中,
满足此条件的随机过程并不是个例。
下面我们将给出这类随机过程的正式定义。
\begin{definition}[鞅]\label{def:martingales_bookI_2.4.1}
\footnote{此定义即书 \cite{shreve_stochastic_2004} 中的定义 2.4.1.}
设二叉树资产定价模型中的过程 $M_0,M_1,\ldots,M_N$
\gls{sysjgc}
\begin{enumerate}[label=\roman*)]
\item 如果
\begin{equation}\label{eq:martingale}
M_n = \mathbb{E}_n[M_{n+1}],\quad n = 0, 1, \ldots, N-1,
\end{equation}
% teaching: 强调是随机变量的相等,即强调条件期望。
则我们称这个过程为
\gls{yang} (\gls{martingale}).
\item 如果
\begin{equation}\label{eq:martingale_sub}
M_n \leq \mathbb{E}_n[M_{n+1}],\quad n = 0, 1, \ldots, N-1,
\end{equation}
则我们称这个过程为
\gls{xy} (\gls{submartingale}).
\item 如果
\begin{equation}\label{eq:martingale_super}
M_n \geq \mathbb{E}_n[M_{n+1}],\quad n = 0, 1, \ldots, N-1,
\end{equation}
则我们称这个过程为
\gls{sy} (\gls{supermartingale}).
\end{enumerate}
\end{definition}
\begin{figure}[htpb]
\centering
\includegraphics[width=0.9\textwidth]{image/martingale.png}
\caption{鞅通常指套在马颈或马腹上的皮带}
\label{fig:image-martingale-png}
\end{figure}
根据鞅的定义,
我们可以证明鞅的多步超前性质。
\begin{proposition}\label{pro:multistep-ahead}
设二叉树资产定价模型中的过程 $M_0,M_1,\ldots,M_N$\gls{yang}
我们有:
\begin{enumerate}[label=\roman*)]
\item {\bf 两步超前性质}
对于任一 $n$, $n\leq N-2$,
\begin{equation}\label{eq:two_step_ahead}
M_n = \mathbb{E}[M_{n+2}].
\end{equation}
\item {\bf 多步超前性质}
对于任一 $n, m$, $0\leq n\leq m\leq N$,
\begin{equation}\label{eq:multistep_ahead}
M_n = \mathbb{E}[M_m].
\end{equation}
\item {\bf 恒定期望性质}
鞅的期望为不依赖于时间的常量,即
\begin{equation}\label{eq:constant_expectation}
M_0 = \mathbb{E}[M_n],
\quad n = 0, 1, \dots, N.
\end{equation}
\end{enumerate}
\end{proposition}
\begin{theorem}[二叉树模型中贴现股票价格过程是鞅]\label{thm:bookI_2_4_4}
\footnote{此定理即书 \cite{shreve_stochastic_2004} 中的定理 2.4.4.}
考虑 \gls{nsdecs}
\gls{fxzxglcd}下,
\gls{txgpjggc}
是一个 \gls{yang}
\end{theorem}
%
% \subsection{风险中性测度下的股价}%
% \label{sub:feng_xian_zhong_xing_ce_du_xia_de_gu_jie_}
$W(t), 0\le t\le T$
概率空间 $(\Omega, \mathcal{F}, \mathbb{P})$
上的布朗运动,
$\mathcal{F}(t), 0\le t \le T$ 是关于该布朗运动的域流。
这里, $T$ 是给定的终端时刻。
考虑服从广义几何布朗运动的股价过程:
\begin{equation}\label{eq:bookII_5_2_16}
S(t) = S(0) \exp \left\{\int_{0}^{t} \sigma(s) \mathop{}\!\mathrm{d}{W(s)} +
\int_{0}^{t} \left( \alpha(s) - \frac{1}{2}\sigma^2(s)\right)
\mathop{}\!\mathrm{d}{s} \right\}.
\end{equation}
假设我们有适应的利率过程 $R(t)$.
定义 {\bf 贴现过程}
\begin{equation}\label{eq:bookII_5_2_17}
D(t) = \exp \left\{ -\int_0^t R(s) \mathop{}\!\mathrm{d}s\right\}
\end{equation}
{\bf 贴现股价过程}$D(t) S(t)$.
\begin{theorem}[贴现股票价格过程是鞅]\label{thm:bookII_page_176}
% \footnote{此定理即书 \cite{shreve_stochastic_2004} 中的定理 2.4.4.}
\gls{fxzxglcd}下,
\gls{txgpjggc} $D(t) S(t)$
是一个 \gls{yang}
\end{theorem}
% \begin{problem}[]\label{pr:S_risk_neutral_measure}
% 验证贴现股价过程是鞅。
% \end{problem}
% \begin{hint}\label{hnt:pr:S_risk_neutral_measure}
% \end{hint}
% \noindent\faHandPointerO ~\hyperlink{target:pr:S_risk_neutral_measure}{前往答案}
% \noindent\faHandPointerO ~\hyperref[pr:S_risk_neutral_measure]{返回问题~\ref{pr:S_risk_neutral_measure}}
% \hypertarget{target:pr:S_risk_neutral_measure}{}
% \begin{solution}\label{sol:pr:S_risk_neutral_measure}
%
% \end{solution}
\begin{problem}\label{pr:measure_stock_price_martingale}
在定理 \ref{thm:bookI_2_4_4} 中,
我们已经证明了在风险中性测度下,
\gls{txgpjggc}是鞅。
验证在没有贴现的情况下,
\gls{gpjggc}在风险中性测度下不是\gls{yang}
能不能找到一个测度,
使得\gls{gpjggc}是鞅?
\end{problem}
\begin{hint}\label{hnt:pr:measure_stock_price_martingale}
结合定理 \ref{thm:bookI_2_4_4} 的结果,
按照定义证明\gls{gpjggc}不是鞅;
列出\gls{gpjggc}是鞅的方程,
求解满足条件的测度。
\end{hint}
% \noindent
% \faHandPointerO ~
% \hyperref[sol:pr:measure_stock_price_martingale]{前往答案}
% \begin{solution}\label{sol:pr:measure_stock_price_martingale}
% \end{solution}
% % next: Add the solution.
\begin{theorem}[二叉树模型中贴现财富过程是鞅]\label{thm:bookI_2_4_5}
\footnote{此定理即书 \cite{shreve_stochastic_2004} 中的定理 2.4.5.}
考虑 \gls{nsdecs}
% 设 $X_0$ 为实数,
% 基于式子 \eqref{eq:pv_period_n_thm:bookI_2_4_5},
% 按时间前向递归定义在时刻 $n = 0, 1, \ldots, N-1$
% 的投资组合价值
% \begin{equation}\label{eq:pv_period_n_thm:bookI_2_4_5}
% X_{n+1} = \varDelta_{n}S_{n+1} + (1 + r)(X_n - \varDelta_n S_n),
% \end{equation}
% 其中 $\varDelta_n$, $n = 0, 1, \ldots, N-1$,
% 为 \gls{sysjgc}。
\gls{fxzxglcd}下,
\gls{txcfgc}是一个 \gls{yang}
\end{theorem}
\begin{theorem}[贴现财富过程是鞅]\label{thm:bookII_page_177}
% \footnote{此定理即书 \cite{shreve_stochastic_2004} 中的定理 2.4.5.}
\gls{fxzxglcd}下,
\gls{txcfgc}是一个 \gls{yang}
\end{theorem}
\begin{corollary}[财富的期望值为常数]\label{cor:bookI_2_4_6}
\footnote{此推论即书 \cite{shreve_stochastic_2004} 中的推论~2.4.6.}
在定理 \ref{thm:bookI_2_4_5} 的条件下,我们有:
\begin{equation}\label{eq:constant_wealth}
\widetilde{\mathbb{E}}\left[ \frac{X_n}{(1+r)^n} \right]
=
X_0,
\quad n = 0, 1, \ldots, N.
\end{equation}
\end{corollary}
\begin{theorem}[二叉树模型中贴现衍生证券价格过程是鞅]\label{thm:bookI_2_4_7}
\footnote{此定理即书 \cite{shreve_stochastic_2004} 中的定理 2.4.7.}
考虑 \gls{nsdecs}
证明在\gls{fxzxglcd}下,
\gls{txyszqjggc}是一个\gls{yang}
\end{theorem}
\begin{theorem}[贴现衍生证券价格过程是鞅]\label{thm:bookII_page_177_178}
% \footnote{此定理即书 \cite{shreve_stochastic_2004} 中的定理 2.4.7.}
证明在\gls{fxzxglcd}下,
\gls{txyszqjggc}是一个\gls{yang}
\end{theorem}
\begin{corollary}[二叉树模型中风险中性定价公式]
\label{cor:re_pricing_formula_BT}
在定理 \ref{thm:bookI_2_4_5} 的条件下,我们有:
\begin{equation}\label{eq:rn_pricing_formula_BT}
V_n =
\widetilde{\mathbb{E}}_n\left[ \frac{V_N}{(1+r)^{N-n}} \right],
\quad n = 0, 1, \ldots, N.
\end{equation}
\end{corollary}
我们称式子 \eqref{eq:rn_pricing_formula_BT}
\gls{fxzxdjgs} (\gls{risk_neutral_pricing_formula}).
\begin{corollary}[风险中性定价公式]
\label{cor:re_pricing_formula}
对于贴现股价过程 $D(t) S(t)$,我们有:
\begin{equation}\label{eq:rn_pricing_formula}
V(t) =
\widetilde{\mathbb{E}}
\left[
\exp \left\{ -\int_t^T R(u)du \right\} V(T) \middle | \mathcal{F}(t)
\right],
\quad 0\le t\le T.
\end{equation}
\end{corollary}
我们称式子 \eqref{eq:rn_pricing_formula}
\gls{fxzxdjgs} (\gls{risk_neutral_pricing_formula}).
\begin{problem}[]\label{pr:bookII_page_178_179}
通过变换到风险中性测度,
推导布莱克--斯科尔斯--默顿偏微分方程的解。
\end{problem}
% \begin{hint}\label{hnt:pr:bookII_page_178_179}
% \end{hint}
% \noindent\faHandPointerO ~\hyperlink{target:pr:bookII_page_178_179}{前往答案}
% \noindent\faHandPointerO ~\hyperref[pr:bookII_page_178_179]{返回问题~\ref{pr:bookII_page_178_179}}
% \hypertarget{target:pr:bookII_page_178_179}{}
% \begin{solution}\label{sol:pr:bookII_page_178_179}
%
% \end{solution}
到目前为止,
我们只讨论了在期末执行的\gls{osyszq}
而许多衍生证券,
如附息债券和利率互换等,
会有一系列的
\gls{zf}
对于这样的衍生证券,
我们有如下的定价和对冲公式。
\begin{theorem}[现金流定价]\label{thm:bookI_2_4_8}
% next: 修改 “现金流定价” 为一个更贴切的名字。
\footnote{此定理即书 \cite{shreve_stochastic_2004} 中的定理 2.4.8.}
考虑基于\gls{fxzxglcd}
\gls{nsdecs}
设现金流过程
$C_0, C_1, \ldots, C_N$
\gls{sysjgc}
在时刻 $n$, $n = 0, 1, \ldots, N$,
衍生证券的价值包含从时刻 $n$$N$ 的所有\gls{zf}
$C_n, \ldots, C_N$.
我们定义衍生证券在时刻 $n$ 的价格为:
\begin{equation}\label{eq:derivative_cash_flow_value}
V_n = \widetilde{\mathbb{E}}_n
\left[ \sum_{k=n}^{N} \frac{C_k}{(1+r)^{k-n}} \right],
\quad n = 0, 1, \ldots, N.
\end{equation}
$n = N-1, N-2, \ldots, 0$,
我们按时间倒向递归定义随机变量序列
\footnote{符号 $\omega^{(n)}$ 即书
中的 $\omega_1\omega_2\cdots\omega_n$;
参见定义 \ref{def:multi_coin_space}.}
随机变量序列
\begin{equation}
\varDelta_n\left( \omega^{(n)}\right) =
\frac{
V_{n+1}\left( \omega^{(n)}H\right)
-
V_{n+1}\left( \omega^{(n)}T\right)
}{
S_{n+1}\left( \omega^{(n)}H\right)
-
S_{n+1}\left( \omega^{(n)}T\right)
}.
\end{equation}
接下来我们让 $X_0 = V_0$,
并且按时间前向递归定义在时刻 $n = 0, 1, \ldots, N-1$
的投资组合价值
\begin{equation}
X_{n+1} = \varDelta_{n}S_{n+1} + (1 + r)(X_n - C_n - \varDelta_n S_n).
\end{equation}
在以上设定下,
我们可以证明:
\begin{equation}
X_n\left( \omega^{(n)}\right)
=
V_n\left( \omega^{(n)}\right),
\quad \forall \; n,
\forall \; \omega^{(n)}.
\end{equation}
\end{theorem}
\begin{figure}[htpb]
\centering
\includegraphics[width=0.9\textwidth]{image/BT-1-cash-flow.pdf}
\caption{单时段现金流定价(问题~\ref{pr:bookI_2_4_8_exa_1} 示意图)}
\label{fig:pr:bookI_2_4_8_exa_1}
\end{figure}
\begin{problem}[
\hyperlink{target:pr:bookI_2_4_8_exa_1}{前往答案}
% \faHandPointerO
(现金流定价的单时段例子)]\label{pr:bookI_2_4_8_exa_1}
如图所示,考虑定理 \ref{thm:bookI_2_4_8} 中的单时段特例。
\begin{enumerate}
\item 求解能够复制衍生证券的 $X_0$$\varDelta_0$.
\item 证明在时刻 0,
衍生证券的无套利定价为上一步中求出的 $X_0$.
\end{enumerate}
\end{problem}
% \begin{hint}\label{hnt:pr:bookI_2_4_8_exa_1}
% \end{hint}
\begin{problem}[
\hyperlink{target:pr:bookI_2_4_8_exa_2}{前往答案}
% \faHandPointerO
(现金流定价的二时段例子)]\label{pr:bookI_2_4_8_exa_2}
如图所示,考虑定理 \ref{thm:bookI_2_4_8} 中的二时段特例。
\begin{enumerate}
\item 求解能够复制衍生证券的 $X_0$, $\varDelta_0$,
$\varDelta_1(H)$$\varDelta_1(T)$.
\item 证明在时刻 0,
衍生证券的无套利定价为上一步中求出的 $X_0$.
\end{enumerate}
\end{problem}
\begin{figure}[htpb]
\centering
\includegraphics[width=0.9\textwidth]{image/BT-2-cash-flow.pdf}
\caption{二时段现金流定价(问题~\ref{pr:bookI_2_4_8_exa_2} 示意图)}
\label{fig:pr:bookI_2_4_8_exa_2}
\end{figure}
% \begin{hint}\label{hnt:pr:bookI_2_4_8_exa_2}
% \end{hint}
\pagebreak
\section{马尔可夫过程}%
\label{sec:markov_process}
\subsection{预备知识}%
\label{sub:markov_process_prerequisites}
\begin{definition}[股票价格过程]\label{def:stock_price_process}
考虑 \gls{nsdecs}
我们称
\begin{equation}\label{eq:stock_price_process}
S_n, \quad n = 0, 1, \ldots, N,
\end{equation}
\gls{gpjggc} (\gls{stock_price_process}).
\end{definition}
\begin{definition}[迄今最大值过程]\label{def:maximum_to_date_process}
考虑 \gls{nsdecs}
我们称
\begin{equation}\label{eq:maximum_to_date_process}
M_n = \max_{0\leq n\leq N} S_n, \quad n = 0, 1, \ldots, N,
\end{equation}
\gls{qjzdzgc} (\gls{maximum_to_date_process}).
\end{definition}
\begin{definition}[二维随机过程]\label{def:2_markov_process}
考虑 \gls{nsdecs}
我们称过程
\eqref{eq:stock_price_process}
\eqref{eq:maximum_to_date_process}
合并成的过程
\begin{equation}\label{eq:2_markov_process}
(S_n, M_n), \quad n = 0, 1, \ldots, N,
\end{equation}
为二维随机过程。
\end{definition}
\begin{definition}[衍生证券价格过程]%
\label{def:derivative_price_process}
考虑 \gls{nsdecs}
我们称
\begin{equation}\label{eq:derivative_price_process}
V_n, \quad n = 0, 1, \ldots, N.
\end{equation}
\gls{yszqjggc} (\gls{derivative_price_process}).
\end{definition}
\begin{definition}[马尔可夫过程]\label{def:markov_process}
考虑 \gls{nsdecs}
$X_0, X_1, \ldots, X_N$
为适应过程。
如果对每个 $0$$N-1$ 之间的 $n$ 以及每个函数 $f(x)$,
存在另一个函数 $g(x)$ (依赖于 $n$$f$),
使得:
\begin{equation}\label{eq:markov_process}
\mathbb{E}_n\left[ f(X_{n+1})\right] = g(X_n),
\end{equation}
则称
$X_0, X_1, \ldots, X_N$
是一个
\gls{mekfgc} (\gls{markov_process}).
\end{definition}
\begin{definition}[马尔可夫链]\label{def:markov_chain}
我们称一个随机过程 $\{X(t),t\in \mathscr{T}\}$
\textit{\gls{mekfl}},如果:
\begin{enumerate}
\item 指标集 $\mathscr{T}$ 和状态空间 $\mathscr{X}$ 都是可数的;
\item 对任意 $n$, 只要过程当前的状态为 $i$,
那么在下一个时刻进入状态 $j$ 的概率为一个固定的概率 \[
P_{ij}^n\equiv\mathbb{P}\{X_{n+1}=j|X_n=i\}.
\]
即对于一切状态 $i_0,i_1,\cdots,i_{n-1},$
\begin{equation}\label{eq:markov_chain_prob}
\mathbb{P}\{X_{n+1}=j|X_n=i,X_{n-1} =
i_{n-1},\ldots,X_0=i_0\}=P_{ij}^n.
\end{equation}
\end{enumerate}
除非特别提醒,我们记
$\mathscr{T} = \{0,1,2,\ldots\}, \mathscr{X} = \{0,1,2,\ldots\}$.
\end{definition}
\begin{lemma}[独立性]\label{lem:independence}
考虑 \gls{nsdecs}
假设随机变量 $X$ 只依赖第 1 到 $n$ 次抛掷结果,
随机变量 $Y$ 只依赖第 $n+1$ 到第 $N$ 次抛掷结果。
$f(x, y)$ 是哑变量 $x$$y$ 的函数。
我们定义:
\begin{equation}\label{eq:lem:independence}
g(x) = \mathbb{E}[f(x, Y)].
\end{equation}
则有:
\begin{equation}\label{eq:lem:independence_2}
\mathbb{E}_n[f(X, Y)] = g(X).
\end{equation}
\end{lemma}
\begin{lemma}[独立性-多维]\label{lem:independence-KL}
考虑 \gls{nsdecs}
假设随机变量 $X^1,\ldots,X^K$ 只依赖第 1 到 $n$ 次抛掷结果,
随机变量 $Y^1,\ldots,Y^L$ 只依赖第 $n+1$ 到第 $N$ 次抛掷结果。
$f(x^1,\ldots,x^K, y^1,\ldots,y^L)$
是哑变量 $x^1,\ldots,x^K$$y^1,\ldots,y^L$ 的函数。
我们定义:
\begin{equation}\label{eq:lem:independence-KL}
g(x) = \mathbb{E}[f(x^1,\ldots,x^K, Y^1,\ldots,Y^L)].
\end{equation}
则有:
\begin{equation}\label{eq:lem:independence-KL_2}
\mathbb{E}_n[f(X^1,\ldots,X^K, Y^1,\ldots,Y^L)] = g(X^1,\ldots,X^K).
\end{equation}
\end{lemma}
\begin{definition}[$K$-维马尔可夫过程]\label{def:k_markov_process}
考虑 \gls{nsdecs}
$X_0, X_1, \ldots, X_N$
\begin{equation}\label{eq:k_adapted_stochastic_process}
\left\{
\left( X_n^1, X_n^2, \ldots, X_n^K \right);
\quad n = 0, 1, \ldots, N
\right\}
\end{equation}
为一个 $K$-维适应过程。
如果对每个 $0$$N-1$ 之间的 $n$ 以及每个函数 $f(x^1,\ldots,x^K)$,
存在另一个函数 $g(x^1,\ldots,x^K)$ (依赖于 $n$$f$),
使得:
\begin{equation}\label{eq:k_markov_process}
\mathbb{E}_n\left[ f(X_{n+1}^1, X_{n+1}^2, \ldots, X_{n+1}^K)\right]
=
g(X_n^1, X_n^2, \ldots, X_n^K),
\end{equation}
则称
过程 \eqref{eq:k_adapted_stochastic_process}
是一个
\gls{mekfgc}
\end{definition}
\subsection{关键问题}%
\label{sub:markov_process}
\begin{introduction}
\item 股票价格过程的马尔可夫性
\item 迄今最大值过程的非马尔可夫性
\item 二维过程的马尔可夫性
\item 衍生证券价格可由\gls{mekfgc}表示
\end{introduction}
\bigskip
\begin{proposition}[股票价格过程的马尔可夫性]\label{pro:markov_stock_price}
考虑 \gls{nsdecs}
根据定义 \ref{def:markov_process} 证明:
不论是在测度 \gls{P} 还是在风险中性测度 \gls{Pn} 下,
\gls{gpjggc}
是一个\gls{mekfgc}
\end{proposition}
\bigskip
\begin{hint}\label{hnt:pro:markov_stock_price}
写出\gls{gpjggc}的具体表达式,
求出满足定义 \ref{def:markov_process} 的函数 $g(\cdot)$.
\end{hint}
\bigskip
\gls{mekfl}是一类特殊的\gls{mekfgc}
如果我们能够证明股票价格过程是一个\gls{mekfl}
那就证明了其是一个\gls{mekfgc}
\bigskip
\begin{problem}[股票价格过程的马尔可夫性]\label{pr:markov_stock_price_2}
考虑 \gls{nsdecs}
根据定义 \ref{def:markov_chain} 证明:
不论是在测度 \gls{P} 还是在风险中性测度 \gls{Pn} 下,
\gls{gpjggc}
是一个\gls{mekfl}
也即一个\gls{mekfgc}
\end{problem}
\bigskip
\begin{proposition}[马尔可夫性定义的等价性]\label{pro:markov_definitions}
考虑 \gls{nsdecs}
证明:
定义 \ref{def:markov_process}
和定义 \ref{def:markov_chain}
是等价的。
\end{proposition}
% \begin{hint}\label{hnt:pro:markov_definitions}
% \end{hint}
% \noindent\faHandPointerO ~\hyperlink{target:pro:markov_definitions}{前往证明}
\noindent ~\hyperlink{target:pro:markov_definitions}{前往证明}
\bigskip
\ref{sec:multi_period_binomial_model} 中,
我们求出了\gls{osyszq}
在各个时刻的定价,
这一序列的定价构成了一个\gls{yszqjggc}
鉴于\gls{gpjggc}
是一个\gls{mekfgc}
那么我们希望用\gls{gpjggc}
来表示\gls{yszqjggc}
如果一个\gls{osyszq}
在期末执行时的价值只和当期的股票价格相关,
我们可以用股票价格的函数来表示衍生证券价格。
\bigskip
\begin{problem}
[(衍生证券价格是股价的函数)]\label{pr:derivative_as_function_of_stock}
考虑 \gls{nsdecs}
假设\gls{osyszq}在执行时刻 $N$\gls{zf}
$V_N$ 为股票价格 $S_N$ 的函数
$v_N$, 即 $V_N = v_N(S_N)$.
请用一个递归算法,
对每个 $n = N-1, \dots, 1, 0$,
求出函数 $v_n(\cdot)$,
使得 $v_n = v_n(S_n)$ 成立。
\end{problem}
\bigskip
\begin{hint}\label{hnt:pr:derivative_as_function_of_stock}
根据定理 \ref{thm:bookI_2_4_7},
利用\gls{mekfgc}的定义可得 $v_n$.
% 根据定理 \ref{thm:bookI_1_2_2},
我们可以通过递归算出函数。
\end{hint}
\bigskip
\begin{problem}
[(判断\gls{qjzdzgc}是否\gls{mekfgc})]\label{pr:non-markov_process}
在图 \ref{fig:ex:lookback_option} 的二叉树模型中,
考虑\gls{qjzdzgc},
其中
$p = \frac{2}{3}, q = \frac{1}{3}$.
请问此过程是\gls{mekfgc}吗?
\end{problem}
\bigskip
\begin{hint}\label{hnt:non-markov_process}
不是\gls{mekfgc},根据图 \ref{fig:ex:lookback_option},
可知前一期的信息不足以决定当期随机变量的取值。
我们也可以严格地证明。
\end{hint}
\bigskip
当我们遇到非\gls{mekfgc}时,
我们可以通过增加一些所谓的状态变量来重新获得马尔可夫性质。
在问题 \ref{pr:non-markov_process} 中,
% teaching: 从图中可以看到,迄今最大值过程损失掉了很多信息(分叉数变少),
% 如果我们只通过前一期的信息的化,是不能够唯一决定本期过程的走向的。
% 注意:没有马尔可夫性的部分一定和 path 被重合的地方相关。
我们可以增加哪些状态变量来获得马尔可夫性?
\bigskip
\begin{proposition}[$(S_n, M_n)$ 的马尔可夫性]\label{pro:s_n_markov_process}
考虑 \gls{nsdecs}
证明
二维随机过程(参见定义 \ref{def:2_markov_process}
\begin{equation*}%\label{eq:2_markov_process}
(S_n, M_n), \quad n = 0, 1, \ldots, N,
\end{equation*}
是一个\gls{mekfgc}
\end{proposition}
\bigskip
书~\cite{shreve_stochastic_2004} 中采用定义 \ref{def:k_markov_process}
证明了这个命题。
我们也可以考虑用\gls{mekfl}的定义来证明它。
% \vspace{1cm}
\begin{problem}[~$(S_n, M_n)$ 的马尔可夫性]\label{pr:s_n_markov_process}
考虑 \gls{nsdecs}
根据定义 \ref{def:markov_chain} 证明:
二维随机过程
\begin{equation*}%\label{eq:2_markov_process}
(S_n, M_n), \quad n = 0, 1, \ldots, N,
\end{equation*}
是一个\gls{mekfl}
也即一个\gls{mekfgc}
\end{problem}
% \vspace{2cm}
\begin{theorem}[衍生证券价格是股价的函数]\label{thm:bookI_2_5_8}
考虑基于测度 \gls{Pn}\gls{nsdecs}
$X_0, X_1, \ldots, X_N$
\gls{mekfgc}
如果欧式衍生证券在时刻 $N$\gls{zf}
$v_N(X_N)$,
我们有:
\begin{equation}\label{eq:v_function_of_x}
V_n = v_n(X_n), \quad n = 0, 1, \ldots, N.
\end{equation}
存在一个计算 $v_n(\cdot)$ 的递归算法,
确切的公式依赖于基本的\gls{mekfgc}
对于多维\gls{mekfgc}
$X_0, X_1, \ldots, X_N$,
类似结果也成立。
\end{theorem}
\pagebreak
\section*{答案}%
\hypertarget{target:pr:bookI_2_4_8_exa_1}{}
\begin{solution}\label{sol:pr:bookI_2_4_8_exa_1}
\hyperref[pr:bookI_2_4_8_exa_1]{返回问题~\ref{pr:bookI_2_4_8_exa_1}}
% \faHandPointerO
(现金流定价的单时段例子)
\begin{enumerate}
\item
假设初始财富是 $X_0$,
股票的头寸为 $\varDelta_0$,
则投资者在时刻 1 的财富是如下的一个随机变量:
\begin{equation}\label{eq:sol:pr:bookI_2_4_8_X_1}
X_1 = \varDelta_0 S_1 + (1 + r) (X_0 - C_0 - \varDelta_0 S_0)
\end{equation}
如果组合 $(X_0, \varDelta_0)$ 能够复制衍生证券,
必有 \[X_1(\omega) = C_1(\omega), \quad \omega\in\{H, T\}.\]
所以我们有方程组
\begin{align}
\varDelta_0 S_1(H) + (1 + r) (X_0 - C_0 - \varDelta_0 S_0)
&= C_1(H)
\label{eq:sol:pr:bookI_2_4_8_system_1} \\
\varDelta_0 S_1(T) + (1 + r) (X_0 - C_0 - \varDelta_0 S_0)
&= C_1(T)
\label{eq:sol:pr:bookI_2_4_8_system_2}.
\end{align}
求解此方程组,
我们得到
\begin{equation}\label{eq:sol:pr:bookI_2_4_8_varDelta_0}
\varDelta_0 =
\frac{
C_1\left(H\right)
-
C_1\left(T\right)
}{
S_1\left(H\right)
-
S_1\left(T\right)
}.
\end{equation}
$\varDelta_0$ 代入 \eqref{eq:sol:pr:bookI_2_4_8_system_2},
我们有
\begin{equation}\label{eq:sol:pr:bookI_2_4_8_X_0}
X_0 =
C_0 + \frac{1}{1 + r}[\tilde{p}C_1(H) + \tilde{q}C_1(T)].
\end{equation}
\item 要证明衍生证券的无套利定价为 $X_0$,
我们先证明如果存在无套利定价 $V_0$
那么 $V_0$ 必定为 $X_0$.
然后我们证明当 $V_0 = X_0$ 时,
一个包含衍生证券、股票、以及货币的投资组合无法套利。
\begin{enumerate}
\item
要证明如果存在无套利定价 $V_0$
那么 $V_0 = X_0$.
我们只需证明如果 $V_0\neq X_0$,
则必存在套利。
不失一般性,我们假设 $C_1(H) > C_1(T)$.
\begin{enumerate}
\item$V_0 > X_0$,
则作出如下的操作:
卖出 1 个单位的衍生证券,按照式
\eqref{eq:sol:pr:bookI_2_4_8_varDelta_0}
买入 $\varDelta_0$ 份股票;
在支付 $C_0$ 后,
将所有的财富投资于货币市场。
我们将证明这样的操作可以套利,
并算出套利的金额。
在时刻 0 且未支付 $C_0$ 时,
包含衍生证券、股票、以及货币的投资组合如下:
\begin{equation}\label{eq:sol:pr:bookI_2_4_8_portfolio_DSM}
\begin{pmatrix}
-1 \\ \varDelta_0 \\ V_0 - \varDelta_0 S_0
\end{pmatrix}
.
\end{equation}
此刻
衍生证券、股票、以及货币的单位价值可以表示为
\begin{equation}\label{eq:sol:pr:bookI_2_4_8_price_DSM_0}
\begin{pmatrix} V_0 & S_0 & 1 \end{pmatrix}.
\end{equation}
在时刻 0 投资者的财富为
\begin{equation}\label{eq:sol:pr:bookI_2_4_8_value_0}
X_0
=
\begin{pmatrix} V_0 & S_0 & 1 \end{pmatrix}
\cdot
\begin{pmatrix}
-1 \\ \varDelta_0 \\ V_0 - \varDelta_0 S_0
\end{pmatrix}
=
0.
\end{equation}
现在我们要计算式 \eqref{eq:sol:pr:bookI_2_4_8_portfolio_DSM}
给出的投资组合在时刻 1 的价值 $X_1$.
在时刻 1,
衍生证券的支付、股票、以及货币的单位价值可以表示为
\begin{equation}\label{eq:sol:pr:bookI_2_4_8_price_DSM_1}
\begin{pmatrix} C_1 & S_1 & (1 + r) \end{pmatrix}.
\end{equation}
所以
\begin{equation}\label{eq:sol:pr:bookI_2_4_8_value_1}
X_1
=
\begin{pmatrix} C_1 & S_1 & (1 + r) \end{pmatrix}
\begin{pmatrix}
-1 \\ \varDelta_0 \\ V_0 - \varDelta_0 S_0 - C_0
\end{pmatrix}
.
\end{equation}
注意到我们已经算得
$\varDelta_0$ 份股票和现金 $X_0 - \varDelta_0 S_0$
可以复制一份衍生证券,
我们可以将上式中的
\begin{equation*}
\begin{pmatrix}
-1 \\ \varDelta_0 \\ V_0 - \varDelta_0 S_0 - C_0
\end{pmatrix}
\end{equation*}
作如下分解:
\begin{equation}\label{eq:sol:pr:bookI_2_4_8_portfolio_DSM_2}
\begin{pmatrix}
0 \\ 0 \\ V_0 - X_0
\end{pmatrix}
+
\begin{pmatrix}
-1 \\ \varDelta_0 \\ X_0 - \varDelta_0 S_0 - C_0
\end{pmatrix}
.
\end{equation}
由式
\eqref{eq:sol:pr:bookI_2_4_8_system_1}
\eqref{eq:sol:pr:bookI_2_4_8_system_2},
\begin{equation}\label{eq:sol:pr:bookI_2_4_8_value_1_part}
\begin{pmatrix} C_1 & S_1 & (1 + r) \end{pmatrix}
\begin{pmatrix}
-1 \\ \varDelta_0 \\ X_0 - \varDelta_0 S_0 - C_0
\end{pmatrix}
=
0
.
\end{equation}
综上,
在时刻 1 的财富 $X_1$ 为一个常数,
其值为
\begin{align}
X_1
&=
\begin{pmatrix}
C_1
& S_1
& 1 + r
\end{pmatrix}
\begin{pmatrix}
0 \\ 0 \\ V_0 - X_0
\end{pmatrix}
\notag \\
&= (1 + r) (V_0 - X_0)
\label{eq:sol:pr:bookI_2_4_8_value_1_arbitrage}.
\end{align}
这样式 \eqref{eq:sol:pr:bookI_2_4_8_portfolio_DSM}
给出的组合
在时刻 0 的财富为 0,
在时刻 1 的财富为常数 \[(1 + r) (V_0 - X_0).\]
即我们证明了此组合可以套利,
且其数量为时刻 0 的 $V_0 - X_0$.
\item$V_0 < X_0$,
则作出如下的操作:
买入 1 个单位的衍生证券,按照式
\eqref{eq:sol:pr:bookI_2_4_8_varDelta_0}
卖出 $\varDelta_0$ 份股票;
收到支付 $C_0$ 后,
将所有的财富投资于货币市场。
这样的组合就可以套利 $X_0 - V_0$.
我们这里省略了证明,
请读者参照当 $V_0 > X_0$ 时有套利的证明。
\end{enumerate}
\item 我们要证明当衍生证券在时刻 0 的价格为
\eqref{eq:sol:pr:bookI_2_4_8_X_0},
\begin{equation}\label{eq:sol:pr:bookI_2_4_8_V_0}
V_0 = C_0 + \frac{1}{1 + r}[\tilde{p}C_1(H) + \tilde{q}C_1(T)]
\end{equation}
时,
任何初始财富为 0 的包含
衍生证券、股票、以及货币的投资组合
\begin{equation}
\label{eq:sol:pr:bookI_2_4_8_portfolio_DSM_unknown}
\begin{pmatrix}
1 \\ \varDelta_0 \\ \varXi_0
\end{pmatrix}
\end{equation}
无法套利。
注意我们假设此组合中包含数量为非 0 的衍生证券,
并且假设其为正。
如果其为 0,
\cite{shreve_stochastic_2004}
中的习题 1.1 已经证明了套利不存在。
如果衍生证券的数量为负,
可以用类似于其为正的方法证明。
\eqref{eq:sol:pr:bookI_2_4_8_portfolio_DSM_unknown}
中的投资组合在时刻 0 的值 $X_0$ 为 0, 即 \[
X_0 = V_0 + \varDelta_0 S_0 + \varXi_0 = 0.
\]
此投资组合在时刻 1 的价值为 \[
X_1 = C_1 + \varDelta_0 S_1 + (1+r)(\varXi_0 + C_0).
\]
计算 $X_1$\gls{fxzxglcd}下的期望,可得
\begin{align}
\widetilde{\mathbb{E}}\left[ X_1 \right]
&=
\widetilde{\mathbb{E}}\left[ C_1 \right]
+
\varDelta_0
\widetilde{\mathbb{E}}\left[ S_1 \right]
+
(1+r)(\varXi_0 + C_0)
\notag \\
&= (1+r) \left( V_0 + \varDelta_0 S_0 + \varXi_0\right)
\notag \\
&= 0 \notag.
\end{align}
由此可以推出如果 $X(H) > 0$,
$X(T) < 0$.
反之亦然。
所以组合 \eqref{eq:sol:pr:bookI_2_4_8_portfolio_DSM_unknown}
不可能套利。
\end{enumerate}
\end{enumerate}
\end{solution}
\hypertarget{target:pr:bookI_2_4_8_exa_2}{}
\begin{solution}\label{sol:pr:bookI_2_4_8_exa_2}
\hyperref[pr:bookI_2_4_8_exa_2]{返回问题~\ref{pr:bookI_2_4_8_exa_2}}
% \faHandPointerO
(现金流定价的二时段例子)
\begin{enumerate}
\item
假设初始财富是 $X_0$,
股票的头寸为 $\varDelta_0$,
则投资者在时刻 1 的财富是如下的一个随机变量:
\begin{equation}\label{eq:sol:pr:bookI_2_4_8_exa_2_X_1}
X_1^{\omega_1} = \varDelta_0 S_1^{\omega_1} +
(1 + r) (X_0 - C_0 - \varDelta_0 S_0),
\end{equation}
所以我们有方程
\begin{align}
\varDelta_0 S_1^H + (1 + r) (X_0 - C_0 - \varDelta_0 S_0)
&= X_1^H,
\label{eq:sol:pr:bookI_2_4_8_exa_2_system_1} \\
\varDelta_0 S_1^T + (1 + r) (X_0 - C_0 - \varDelta_0 S_0)
&= X_1^T
\label{eq:sol:pr:bookI_2_4_8_exa_2_system_2}.
\end{align}
其中 $\omega_1$ 表示第一次抛掷硬币的结果,
$S_1^{\omega_1}$$S_1(\omega_1)$,
$X_1^{\omega_1}$$X_1(\omega_1)$,
$\omega_1\in\{H, T\}$.
假设投资者在时刻 1 将股票的头寸调整为 $\varDelta_1$,
则其在时刻 2 的财富是如下的一个随机变量:
\begin{equation*}\label{eq:sol:pr:bookI_2_4_8_exa_2_X_2}
X_2 = \varDelta_1 S_2 +
(1 + r) (X_1 - C_1 - \varDelta_1 S_1),
\end{equation*}
如果组合 $(X_0, \varDelta_0, \varDelta_1)$ 能够复制衍生证券,
必有 \[X_2^{\omega} = C_2^{\omega},
\quad \omega\in\{HH, HT, TH, TT\}.\]
所以,
当第一次抛掷硬币的结果为 $H$,
我们有方程
\begin{align}
\varDelta_1^H S_2^{HH} +
(1 + r) (X_1^H - C_1^H - \varDelta_1^H S_1^H)
&= C_2^{HH},
\label{eq:sol:pr:bookI_2_4_8_exa_2_system_3} \\
\varDelta_1^H S_2^{HT} +
(1 + r) (X_1^H - C_1^H - \varDelta_1^H S_1^H)
&= C_2^{HT}
\label{eq:sol:pr:bookI_2_4_8_exa_2_system_4}.
\end{align}
当第一次抛掷硬币的结果为 $T$,
我们有方程
\begin{align}
\varDelta_1^T S_2^{TH} +
(1 + r) (X_1^T - C_1^T - \varDelta_1^T S_1^T)
&= C_2^{TH},
\label{eq:sol:pr:bookI_2_4_8_exa_2_system_5} \\
\varDelta_1^T S_2^{TT} +
(1 + r) (X_1^T - C_1^T - \varDelta_1^T S_1^T)
&= C_2^{TT}
\label{eq:sol:pr:bookI_2_4_8_exa_2_system_6}.
\end{align}
其中 $\omega_1$ 表示第一次抛掷硬币的结果,
$\varDelta_1^{ \omega_1}$$\varDelta_1(\omega_1)$,
$\omega_1\in\{H, T\}$;
$X_2^{\omega}$$X_2(\omega)$,
$C_2^{\omega}$$C_2(\omega)$,
$S_2^{\omega}$$S_2(\omega)$, $\omega\in\{HH, HT, TH, TT\}$.
求解方程组
\eqref{eq:sol:pr:bookI_2_4_8_exa_2_system_3}
-
\eqref{eq:sol:pr:bookI_2_4_8_exa_2_system_4}
我们得到
\begin{equation}\label{eq:sol:pr:bookI_2_4_8_exa_2_varDelta_1H}
\varDelta_1^H = \frac{C_2^{HH} - C_2^{HT}}{S_2^{HH} - S_2^{HT}},
\end{equation}
$\varDelta_1^H$ 代入 \eqref{eq:sol:pr:bookI_2_4_8_exa_2_system_3},
并注意到
\begin{equation}\label{eq:sol:pr:bookI_2_4_8_exa_2_S_1H}
S_1^H = \frac{1}{1+r}[\tilde{p}S_2^{HH} + \tilde{q}S_2^{HT}],
\end{equation}
我们有
\begin{align}
X_1^H
&= C_1^H + \frac{1}{1 + r}[\tilde{p}C_2^{HH} + \tilde{q}C_2^{HT}]
\notag \\
&= C_1^H + \frac{1}{1 + r}\widetilde{\mathbb{E}}_1[C_2](H)
\label{eq:sol:pr:bookI_2_4_8_exa_2_X_1H}.
\end{align}
求解方程组
\eqref{eq:sol:pr:bookI_2_4_8_exa_2_system_5}
-
\eqref{eq:sol:pr:bookI_2_4_8_exa_2_system_6}
我们得到
\begin{equation}\label{eq:sol:pr:bookI_2_4_8_exa_2_varDelta_1T}
\varDelta_1^T = \frac{C_2^{TH} - C_2^{TT}}{S_2^{TH} - S_2^{TT}},
\end{equation}
$\varDelta_1^T$ 代入 \eqref{eq:sol:pr:bookI_2_4_8_exa_2_system_5},
并注意到
\begin{equation}\label{eq:sol:pr:bookI_2_4_8_exa_2_S_1T}
S_1^T = \frac{1}{1+r}[\tilde{p}S_2^{TH} + \tilde{q}S_2^{TT}],
\end{equation}
我们有
\begin{align}
X_1^T
&= C_1^T + \frac{1}{1 + r}[\tilde{p}C_2^{TH} + \tilde{q}C_2^{TT}]
\notag \\
&= C_1^T + \frac{1}{1 + r}\widetilde{\mathbb{E}}_1[C_2](T)
\label{eq:sol:pr:bookI_2_4_8_exa_2_X_1T}.
\end{align}
求解方程组
\eqref{eq:sol:pr:bookI_2_4_8_exa_2_system_1}
-
\eqref{eq:sol:pr:bookI_2_4_8_exa_2_system_2}
我们得到
\begin{align}
\varDelta_0
&= \frac{X_1^H - X_1^T}{S_1^H - S_1^T} \notag \\
&= \frac{C_1^H -C_1^T}{S_1^H - S_1^T}
+ \frac{1}{1 + r}
\frac{\widetilde{\mathbb{E}}_1[C_2](H) -
\widetilde{\mathbb{E}}_1[C_2](T)}
{S_1^H - S_1^T}
\label{eq:sol:pr:bookI_2_4_8_exa_2_varDelta_0}.
\end{align}
$\varDelta_0$ 代入 \eqref{eq:sol:pr:bookI_2_4_8_exa_2_system_1},
我们有
\begin{equation}\label{eq:sol:pr:bookI_2_4_8_exa_2_X_0_temp}
X_0 =
C_0 + \frac{1}{1 + r}[\tilde{p}X_1^H + \tilde{q}X_1^T].
\end{equation}
\eqref{eq:sol:pr:bookI_2_4_8_exa_2_X_1H}
\eqref{eq:sol:pr:bookI_2_4_8_exa_2_X_1T}
代入 \eqref{eq:sol:pr:bookI_2_4_8_exa_2_X_0_temp},
我们有
\begin{equation}\label{eq:sol:pr:bookI_2_4_8_exa_2_X_0}
X_0 = C_0
+ \frac{\widetilde{\mathbb{E}}[C_1]}{1 + r}
+ \frac{\widetilde{\mathbb{E}}[C_2]}{(1 + r)^2}.
\end{equation}
综上,
式~\eqref{eq:sol:pr:bookI_2_4_8_exa_2_X_0} 中的 $X_0$,
式~\eqref{eq:sol:pr:bookI_2_4_8_exa_2_varDelta_0} 中的 $\varDelta_0$,
式~\eqref{eq:sol:pr:bookI_2_4_8_exa_2_varDelta_1H}
中的 $\varDelta_1^H$,
式~\eqref{eq:sol:pr:bookI_2_4_8_exa_2_varDelta_1T}
中的 $\varDelta_1^T$
能够复制给定的现金流衍生证券。
\item 要证明衍生证券的无套利定价为 $X_0$,
我们先证明如果存在无套利定价 $V_0$
那么 $V_0$ 必定为 $X_0$.
然后我们证明当 $V_0 = X_0$ 时,
一个包含衍生证券、股票、以及货币的投资组合无法套利。
\begin{enumerate}
\item
要证明如果存在无套利定价 $V_0$
那么 $V_0 = X_0$.
我们只需证明如果 $V_0\neq X_0$,
则必存在套利。
$V_0 > X_0$,
则作出如下的操作:
在时刻 0,
卖出 1 个单位的衍生证券,按照式
\eqref{eq:sol:pr:bookI_2_4_8_exa_2_varDelta_0}
建立股票头寸 $\varDelta_0$;
在支付 $C_0$ 后,
将所有的财富投资于货币市场。
在时刻 1,
如果抛掷硬币的结果为 $H$,
调整股票头寸到式
\eqref{eq:sol:pr:bookI_2_4_8_exa_2_varDelta_1H} 给出的数量,
否则调整到
\eqref{eq:sol:pr:bookI_2_4_8_exa_2_varDelta_1T} 给出的数量;
在支付 $C_1$ 后,
将所有的财富投资于货币市场。
我们将证明这样的操作可以套利,
并算出套利的金额。
在时刻 0 且未支付 $C_0$ 时,
包含衍生证券、股票、以及货币的投资组合如下:
\begin{equation}
\label{eq:sol:pr:bookI_2_4_8_exa_2_portfolio_DSM}
\begin{pmatrix}
-1 \\ \varDelta_0 \\ V_0 - \varDelta_0 S_0
\end{pmatrix}
.
\end{equation}
此刻
衍生证券、股票、以及货币的单位价值可以表示为
\begin{equation}
\label{eq:sol:pr:bookI_2_4_8_exa_2_price_DSM_0}
\begin{pmatrix} V_0 & S_0 & 1 \end{pmatrix}.
\end{equation}
在时刻 0 投资者的财富为
\begin{equation}\label{eq:sol:pr:bookI_2_4_8_exa_2_value_0}
X_0
=
\begin{pmatrix} V_0 & S_0 & 1 \end{pmatrix}
\begin{pmatrix}
-1 \\ \varDelta_0 \\ V_0 - \varDelta_0 S_0
\end{pmatrix}
=
0.
\end{equation}
现在我们要计算
\eqref{eq:sol:pr:bookI_2_4_8_exa_2_portfolio_DSM}
给出的投资组合在时刻 1 的价值 $X_1$.
在时刻 1,
衍生证券的支付、股票、以及货币的单位价值可以表示为
\begin{equation}\label{eq:sol:pr:bookI_2_4_8_exa_2_price_DSM_1}
\begin{pmatrix}
C_1 + \frac{\widetilde{\mathbb{E}}_1[C_2]}{1 + r}
& S_1
& 1 + r
\end{pmatrix}.
\end{equation}
所以
\begin{equation}\label{eq:sol:pr:bookI_2_4_8_exa_2_value_1}
X_1 =
\begin{pmatrix}
C_1 + \frac{\widetilde{\mathbb{E}}_1[C_2]}{1 + r}
& S_1
& 1 + r
\end{pmatrix}
\begin{pmatrix}
-1 \\ \varDelta_0 \\ V_0 - \varDelta_0 S_0 - C_0
\end{pmatrix}
.
\end{equation}
我们可以将上式中的
\begin{equation*}
\begin{pmatrix}
-1 \\ \varDelta_0 \\ V_0 - \varDelta_0 S_0 - C_0
\end{pmatrix}
\end{equation*}
作如下分解:
\begin{equation}
\label{eq:sol:pr:bookI_2_4_8_exa_2_portfolio_DSM_2}
\begin{pmatrix}
0 \\ 0 \\ V_0 - X_0
\end{pmatrix}
+
\begin{pmatrix}
-1 \\ \varDelta_0 \\ X_0 - \varDelta_0 S_0 - C_0
\end{pmatrix}
.
\end{equation}
式~\eqref{eq:sol:pr:bookI_2_4_8_exa_2_system_1},
式~\eqref{eq:sol:pr:bookI_2_4_8_exa_2_system_1},
式~\eqref{eq:sol:pr:bookI_2_4_8_exa_2_X_1H},
以及
式~\eqref{eq:sol:pr:bookI_2_4_8_exa_2_X_1T},
我们可知
\begin{equation}
\label{eq:sol:pr:bookI_2_4_8_exa_2_value_1_part}
\begin{pmatrix}
C_1 + \frac{\widetilde{\mathbb{E}}_1[C_2]}{1 + r}
& S_1
& 1 + r
\end{pmatrix}
\begin{pmatrix}
-1 \\ \varDelta_0 \\ X_0 - \varDelta_0 S_0 - C_0
\end{pmatrix}
=
0
.
\end{equation}
综上,
在时刻 1 的财富 $X_1$ 为一个常数,
其值为
\begin{align}
X_1
&=
\begin{pmatrix}
C_1 + \frac{\widetilde{\mathbb{E}}_1[C_2]}{1 + r}
& S_1
& 1 + r
\end{pmatrix}
\begin{pmatrix}
0 \\ 0 \\ V_0 - X_0
\end{pmatrix}
\notag \\
&= (1 + r) (V_0 - X_0)
\label{eq:sol:pr:bookI_2_4_8_exa_2_value_1_arbitrage}.
\end{align}
这样式 \eqref{eq:sol:pr:bookI_2_4_8_exa_2_portfolio_DSM}
给出的组合
在时刻 0 的财富为 0,
在时刻 1 的财富为常数 $(1 + r) (V_0 - X_0)$.
用相似的方法,
我们可以证明
在时刻 2 的财富为常数 $(1 + r)^2 (V_0 - X_0)$.
综上,我们证明了此组合可以套利,
且其数量为时刻 0 的 $V_0 - X_0$.
$V_0 < X_0$,
则作出如下的操作:
在时刻 0,
买入 1 个单位的衍生证券,按照式
\eqref{eq:sol:pr:bookI_2_4_8_exa_2_varDelta_0}
的相反数,即 $-\varDelta_0$, 建立股票头寸;
在收到支付 $C_0$ 后,
将所有的财富投资于货币市场。
在时刻 1,
如果抛掷硬币的结果为 $H$,
按照式~\eqref{eq:sol:pr:bookI_2_4_8_exa_2_varDelta_1H}
调整股票头寸到
$-\varDelta_1^H$.
否则按照式~\eqref{eq:sol:pr:bookI_2_4_8_exa_2_varDelta_1T}
调整股票头寸到
$-\varDelta_1^H$.
在收到支付 $C_1$ 后,
将所有的财富投资于货币市场。
我们可以证明这样的操作可以套利,
并算出套利的金额为 $X_0 - V_0$.
我们这里省略了证明,
请读者参照当 $V_0 > X_0$ 时有套利的证明。
\item 我们要证明当衍生证券在时刻 0 的价格为
\eqref{eq:sol:pr:bookI_2_4_8_exa_2_X_0},
\begin{equation}\label{eq:sol:pr:bookI_2_4_8_exa_2_V_0}
V_0 = C_0
+ \frac{\widetilde{\mathbb{E}}[C_1]}{1 + r}
+ \frac{\widetilde{\mathbb{E}}[C_2]}{(1 + r)^2}.
\end{equation}
时,
任何初始财富为 0 的包含
衍生证券、股票、以及货币的投资组合
\begin{equation}
\label{eq:sol:pr:bookI_2_4_8_exa_2_portfolio_DSM_unknown}
\begin{pmatrix}
1 \\ \varDelta_0 \\ \varXi_0
\end{pmatrix}
\end{equation}
无法套利。
注意我们假设此组合中包含数量不为 0 的衍生证券,
并且假设其为正。
% 如果其为 0,
% 用相似的方法可以证明了套利不存在。
% next: 严格地写出这部分的证明
如果衍生证券的数量为负,
可以用类似于其为正的方法证明。
\eqref{eq:sol:pr:bookI_2_4_8_exa_2_portfolio_DSM_unknown}
中的投资组合在时刻 0 的值 $X_0$ 为 0, 即 \[
X_0 = V_0 + \varDelta_0 S_0 + \varXi_0 = 0.
\]
此投资组合在时刻 1(支付发生前)的价值为
\begin{equation}
\label{eq:sol:pr:bookI_2_4_8_exa_2_portfolio_DSM_X1}
X_1 = C_1 +
\frac{\widetilde{\mathbb{E}}_1[C_2]}{1 + r} +
\varDelta_0 S_1 + (1+r)(\varXi_0 + C_0).
\end{equation}
计算 $X_1$\gls{fxzxglcd}下的期望,可得
\begin{align}
\widetilde{\mathbb{E}}\left[ X_1 \right]
&=
\widetilde{\mathbb{E}}\left[ C_1 \right]
+
\frac{\widetilde{\mathbb{E}}[C_2]}{1 + r}
+
\varDelta_0
\widetilde{\mathbb{E}}\left[ S_1 \right]
+
(1+r)(\varXi_0 + C_0)
\notag \\
&= (1+r) \left( V_0 + \varDelta_0 S_0 + \varXi_0\right)
\notag \\
&= 0 \notag.
\end{align}
由此可以推出如果 $X_1^H > 0$,
$X_1^T < 0$.
反之亦然。
所以组合 \eqref{eq:sol:pr:bookI_2_4_8_exa_2_portfolio_DSM_unknown}
在时刻 1 不可能套利。
并且,
如果此组合能够套利,
必须要满足
\begin{equation}
\label{eq:sol:pr:bookI_2_4_8_exa_2_portfolio_DSM_X1_condition}
X_1^H = X_1^T = 0.
\end{equation}
我们接着证明当第一次抛掷结果为 $H$$T$ 时,
对于任意的 $\varDelta_1^H$$\varDelta_1^T$,
在时刻 2 不可能有套利。
我们只需要证明 $H$ 的情形,$T$ 的情形类似。
由式~\eqref{eq:sol:pr:bookI_2_4_8_exa_2_portfolio_DSM_X1},
\begin{equation}
\label{eq:sol:pr:bookI_2_4_8_exa_2_portfolio_DSM_X1H}
X_1^H = C_1^H +
\frac{\widetilde{\mathbb{E}}_1[C_2](H)}{1 + r} +
\varDelta_0 S_1^H + (1+r)(\varXi_0 + C_0),
\end{equation}
% 其中 $\widetilde{\mathbb{E}}_1^H[C_2]$
% 即 $\widetilde{\mathbb{E}}_1[C_2](H)$.
如果将股票的头寸调整为 $\varDelta_1^H$,
\begin{multline}
\label{eq:sol:pr:bookI_2_4_8_exa_2_portfolio_DSM_X2H}
X_2(H\omega_2) =
C_2(H\omega_2) + \varDelta_1^H S_2(H\omega_2) + \\
(1 + r) \left[
C_1^H + (1+r)(\varXi_0 + C_0) -
(\varDelta_1^H - \varDelta_0) S_1^H
\right]
,
\end{multline}
其中
% $X_2^H(\omega_2)$ 即 $X_2(H\omega_2)$,
% $S_2^H(\omega_2)$ 即 $S_2(H\omega_2)$.
对式~\eqref{eq:sol:pr:bookI_2_4_8_exa_2_portfolio_DSM_X2H}
求条件期望,
我们有
\begin{multline}
\label{eq:sol:pr:bookI_2_4_8_exa_2_portfolio_DSM_X2HE_temp}
\widetilde{\mathbb{E}}_1[X_2](H) =
\widetilde{\mathbb{E}}_1[C_2](H) +
\varDelta_1^H \widetilde{\mathbb{E}}_1[S_2](H) \; + \\
(1 + r) \left[
C_1^H + (1+r)(\varXi_0 + C_0) -
(\varDelta_1^H - \varDelta_0) S_1^H
\right].
\end{multline}
$\widetilde{\mathbb{E}}_1[S_2](H) = (1 + r) S_1^H$
代入上式,
再由式~\eqref{eq:sol:pr:bookI_2_4_8_exa_2_portfolio_DSM_X1_condition},
我们有
\begin{equation}
\label{eq:sol:pr:bookI_2_4_8_exa_2_portfolio_DSM_X2HE}
\widetilde{\mathbb{E}}_1[X_2](H)
= (1 + r) X_1^H
= 0.
\end{equation}
由此可以推出如果 $X_2(HH) > 0$,
$X_2(HT) < 0$.
反之亦然。
所以组合 \eqref{eq:sol:pr:bookI_2_4_8_exa_2_portfolio_DSM_unknown}
$\varDelta_1^H$ 在第一次抛掷结果为正面时,
在时刻 2 不可能套利。
综上,我们证明了给定式子~\eqref{eq:sol:pr:bookI_2_4_8_exa_2_V_0}
中的 $V_0$,
任何初始财富为 0 的包含
衍生证券、股票、以及货币的投资组合
不能套利。
\end{enumerate}
\end{enumerate}
\end{solution}
\hypertarget{target:pro:markov_definitions}{}
\begin{proof}\label{pf:pro:markov_definitions}
\hyperref[pro:markov_definitions]{返回命题~\ref{pro:markov_definitions}}
% \faHandPointerO
(马尔可夫性定义的等价性)
我们先证明
定义~\ref{def:markov_process}
蕴含
定义~\ref{def:markov_chain}.
即证明,
如果对每个 $0$$N-1$ 之间的 $n$ 以及每个函数 $f(x)$,
存在另一个函数 $g_n(x)$ (依赖于 $f$),
使得:
\begin{equation}\label{eq:markov_process_2}
\mathbb{E}_n\left[ f(X_{n+1})\right] = g_n(X_n),
\end{equation}
则有
\begin{equation}\label{eq:markov_chain_prob_2}
\mathbb{P}\{X_{n+1}=j|X_n=i,X_{n-1} = i_{n-1},\ldots,X_0=i_0\}
=
P_{ij}^n.
\end{equation}
我们构造如下 $f^j$, $j \in \mathscr{X}$:
\begin{equation}\label{eq:function_f}
f^j(x) =
\begin{cases}
1, & \text{if } x = j, \\
0, & \text{if otherwise}.
\end{cases}
\end{equation}
如果等式 \eqref{eq:markov_chain_prob_2} 的左边有定义,
那么存在 $\omega^{(n)}$,
使得 \[
X_n\left(\omega^{(n)}\right) =i,
X_{n-1}\left(\omega^{(n)}\right) = i_{n-1},
\ldots,
X_0\left(\omega^{(n)}\right)=i_0.
\]
于是我们有
\begin{align}
&\quad\; \mathbb{P}\{X_{n+1}=j |
X_n=i,X_{n-1} = i_{n-1},\ldots,X_0=i_0\} \notag \\
&= \mathbb{E}_n\left[ f^j(X_{n+1})\right]
\left(\omega^{(n)}\right) \notag \\
&= g_n^j\left(X_n\left(\omega^{(n)}\right)\right)
\quad \mbox{由式~\eqref{eq:markov_process_2}} \notag \\
&= g_n^j(i). \notag
\end{align}
由于 $g_n^j(i)$ 是只与时刻 $n$ 以及状态 $i$$j$ 有关的常数,
$P_{ij}^n \equiv g_n^j(i)$,
则式子 \eqref{eq:markov_chain_prob_2} 成立。
所以,我们就证明了
定义~\ref{def:markov_process}
蕴含
定义~\ref{def:markov_chain}.
下面,我们证明
定义~\ref{def:markov_chain}
蕴含
定义~\ref{def:markov_process}.
对每个 $0$$N-1$ 之间的 $n$ 以及每个函数 $f(x)$,
\begin{equation}\label{eq:function_g}
g_n(i) = \sum_{j\in \mathscr{X}} f(j) P_{ij}^n,
\end{equation}
对于任一 $\omega^{(n)}$,
我们有
\begin{align}
&\quad\; \mathbb{E}_n\left[ f(X_{n+1})\right]
\left(\omega^{(n)}\right) \notag \\
&= \sum_{j\in \mathscr{X}} f(j)
\mathbb{P}
\left\{
X_{n+1} = j |
X_n = X_n\left(\omega^{(n)}\right),
% X_{n-1} = i_{n-1},
\ldots,
X_0 = X_0\left(\omega^{(n)}\right)
\right\} \notag \\
&= \sum_{j\in \mathscr{X}} f(j)
\mathbb{P}
\left\{
X_{n+1} = j | X_n = X_n\left(\omega^{(n)}\right)
\right\} \notag \\
&= g_n\left(X_n\left(\omega^{(n)}\right)\right). \notag
\end{align}
所以,
我们证明了
对每个 $0$$N-1$ 之间的 $n$ 以及每个函数 $f(x)$,
存在另一个函数 $g_n(x)$ (依赖于 $f$),
使得:
\begin{equation*}
\mathbb{E}_n\left[ f(X_{n+1})\right] = g_n(X_n).
\end{equation*}
所以,
我们证明了
定义~\ref{def:markov_process}
蕴含
定义~\ref{def:markov_chain}.
\end{proof}
\chapter{状态价格}%
\label{cha:state_price}
\begin{figure}[htpb]
\centering
\includegraphics[width=0.90\textwidth]{image/mindmap-ch03.pdf}
\caption{第三章思维导图}
\label{fig:mindmap_03}
\end{figure}
在二叉树无套利模型中,
我们引入了 \gls{fxzxglcd} \gls{Pn}.
虽然这个测度是``虚构''的,
但却十分有用,
它使得我们能够简洁地表述某些方程组的求解结果。
更进一步,
我们证明了在这个测度下,
贴现股票价格过程(参见定理 \ref{thm:bookI_2_4_4}),
贴现财富过程(参见定理 \ref{thm:bookI_2_4_5}),
以及贴现衍生证券价格过程(参见定理 \ref{thm:bookI_2_4_6})都是鞅。
另外一个重要的测度是真实概率测度 $ \mathbb{P}$,
它是我们通过模型参数的经验估计而得到的。
这两个概率测度对于模型中的资产价格路径给出了不同的权重。
然而,关于哪些价格路径是可能的(即出现的概率为正),
它们是一致的。
\ref{sec:change_of_measure} 中,
我们引入拉东--尼柯迪姆导数,
使得在\gls{fxzxglcd}下的期望可以很容易地写成真实测度下的期望。
\ref{sec:rn_process} 中,
我们将研究拉东--尼柯迪姆导数过程,
这个随机过程由拉东--尼柯迪姆导数在不同时段的条件期望组成。
\ref{sec:capital_asset_pricing_model} 中,
我们将学习关于资产价格的另外一种模式---资本资产定价模型。
注意,
这节的内容和前面两节的内容是相对独立的,
和前面内容的联系在于引入拉东--尼柯迪姆导数,
可以适当地简化运算。
本章的主要内容,
可以参见图 \ref{fig:mindmap_03}.
\section{测度变换}%
\label{sec:change_of_measure}
\begin{problem}[(两个测度下期望值的关系)]\label{pr:expectations_two_measures}
在问题 \ref{pr:one_period_general} 中,
我们研究了单时段一般衍生证券的无套利定价问题。
由式 \eqref{eq:linear_combination_s},
\begin{equation*}
S_0 = \widetilde{\mathbb{E}}\left[ \frac{S_1}{1+r} \right].
\end{equation*}
是否存在一个函数 $g(\cdot)$,
使得
\begin{equation*}
\widetilde{\mathbb{E}}\left[ \frac{S_1}{1+r} \right]
=
\mathbb{E}\left[ g\left(\frac{S_1}{1+r}\right) \right]?
\end{equation*}
对于一个一般的随机变量 $Y$,
是否存在一个函数 $g(\cdot)$,
使得
\begin{equation*}
\widetilde{\mathbb{E}}\left[ Y \right]
=
\mathbb{E}\left[ g\left(Y\right) \right]?
\end{equation*}
\end{problem}
% \begin{solution}\label{sol:expectations_two_measures}
%
% \end{solution}
\begin{definition}[有限空间中的\gls{ldnkdmds}]\label{def:rn_derivative_finite}
考虑一般的有限样本空间 $\Omega$ 上的两个概率测度
$ \mathbb{P}$$\widetilde{\mathbb{P}}$.
如果对任一 $\omega\in\Omega$,
$ \mathbb{P}(\omega) > 0$
并且
$\widetilde{\mathbb{P}}(\omega) > 0$,
则称
\begin{equation}\label{eq:rn_derivative_finite}
Z(\omega) = \frac{\widetilde{\mathbb{P}}(\omega)}{\mathbb{P}(\omega)}
\end{equation}
$\widetilde{\mathbb{P}}$ 关于 $\mathbb{P}$
\gls{ldnkdmds} (\gls{radon_nikodym_derivative}).
\end{definition}
\pagebreak
\begin{definition}[\gls{ldnkdmds}]\label{def:rn_derivative}
考虑一般的概率空间 $(\Omega, \mathcal{F}, \mathbb{P})$,
以及满足 $\mathbb{E}[Z] = 1$ 的非负随机变量 $Z$.
由以下公式定义一个新的概率测度
$\widetilde{\mathbb{P}}$:
\begin{equation}\label{eq:bookII_5_2_1}
\widetilde{\mathbb{P}}(A) =
\int_A Z(\omega) \mathop{}\!\mathrm{d}\mathbb{P}(\omega),
\forall A\in \mathcal{F}.
\end{equation}
我们称 $Z$
$\widetilde{\mathbb{P}}$ 关于 $\mathbb{P}$
\gls{ldnkdmds} (\gls{radon_nikodym_derivative}).
\end{definition}
% \vspace{1cm}
\begin{figure}[htpb]
\centering
\includegraphics[width=0.75\textwidth]{image/BT-3-Z.pdf}
\caption{计算三时段模型中的\gls{ldnkdmds}}
\label{fig:compute_rn}
\end{figure}
\begin{problem}\label{pr:compute_rn}
考虑图 \ref{fig:compute_rn} 中的三时段模型。
样本空间为:\[
\Omega = \{HHH, HHT, HTH, HTT, THH, THT, TTH, TTT\}.
\]
假设 $p = 2 / 3$ 为出现正面的真实概率,
$\tilde{p} = 1 / 2$ 为出现正面的\gls{fxzxglcd}
请计算\gls{ldnkdmds}
\end{problem}
% \vspace{1cm}
\begin{problem}[($Z(\omega)$ 的一般表达式)]\label{pr:Z_omega}
对于任一 $\omega = \omega_1\ldots\omega_N$,
写出
$Z(\omega)$
的一般表达式。
\end{problem}
\begin{solution}\label{sol:Z_omega}
\begin{align}
Z(\omega)
&= \frac{\widetilde{\mathbb{P}}(\omega)}{\mathbb{P}(\omega)} \notag \\
&=
\left( \frac{\tilde{p}}{p} \right)^{\#H_1^N}
\left( \frac{\tilde{q}}{q} \right)^{\#T_1^N}
\label{eq:Z_omega},
\end{align}
其中 $\#H_1^N$ 表示序列中 $\omega_1\ldots\omega_N$ 中出现正面的次数,
$\#T_1^N$ 表示序列中 $\omega_1\ldots\omega_N$ 中出现背面的次数。
\end{solution}
\begin{theorem}[\gls{ldnkdmds}性质]\label{thm:bookI_3_1_1}
\footnote{此定理即书 \cite{shreve_stochastic_2004} 中的定理 3.1.1.}
\gls{ldnkdmds} $Z$ 有如下性质:
\begin{enumerate}[label=\roman*)]
\item $ \mathbb{P}(Z > 0) = 1$;
\item $ \mathbb{E}[Z] = 1$;
\item 对任意的随机变量 $Y$, 有:
\begin{equation}\label{eq:rn_property_3}
\widetilde{\mathbb{E}}[Y] =
\mathbb{E}[ZY].
\end{equation}
\end{enumerate}
\end{theorem}
在本章中,
如无特殊说明,
$Z$$Z(\omega)$ 指的是\gls{ldnkdmds}
\gls{fxzxdjgs}
我们有
\begin{align}
V_0
&= \widetilde{\mathbb{E}}\left[ \frac{V_N}{(1+r)^N} \right] \notag \\
&= \mathbb{E}\left[ Z \frac{V_N}{(1+r)^N} \right] \notag \\
&= \sum_{\omega\in\Omega}
\frac{Z(\omega) \mathbb{P}(\omega)}{(1+r)^N} V_N(\omega)
\label{eq:derivative_price_Z}.
\end{align}
在式子 \eqref{eq:derivative_price_Z} 中,
衍生证券相应于样本点 $\omega$的支付为
\begin{equation*}
\frac{Z(\omega) \mathbb{P}(\omega)}{(1+r)^N} V_N(\omega).
\end{equation*}
注意到上式中的 \[
\frac{Z(\omega) \mathbb{P}(\omega)}{(1+r)^N}
\]
不依赖于衍生证券的价格而仅依赖于状态 $\omega$,
我们称其为状态价格,
称其中的 \[
\frac{Z(\omega)}{(1+r)^N}
\]
为状态价格密度。
\begin{definition}[状态价格]\label{def:bookI_3_1_3_state_price}
\footnote{此定义即书 \cite{shreve_stochastic_2004} 中的定义 3.1.3.}
考虑 \gls{nsdecs}
定义 {\bf \gls{ztjgmd} (\gls{state_price_density})} 为:
\begin{equation}\label{eq:state_price_density}
\zeta(\omega) =
\frac{Z(\omega)}{(1+r)^N},
\end{equation}
定义 {\bf \gls{ztjg} (\gls{state_price})} 为相应于 $ \omega$
\begin{equation}\label{eq:state_price}
\zeta(\omega) \mathbb{P}\left(\omega\right).
\end{equation}
\end{definition}
\section{拉东--尼柯迪姆导数过程}%
\label{sec:rn_process}
\begin{theorem}[随机变量的条件期望是鞅]\label{thm:bookI_3_2_1}
\footnote{此定理即书 \cite{shreve_stochastic_2004} 中的定理 3.2.1.}
考虑 \gls{nsdecs}中的随机变量 $X$
定义:
\begin{equation}\label{eq:bookI_3_2_1}
X_n = \mathbb{E}_n[X],
\quad n = 0, 1, \ldots, N,
\end{equation}
$X_n, n = 0, 1, \ldots, N$ 在真实概率测度 \gls{P} 下是一个\gls{yang}
\end{theorem}
\bigskip
考虑 \gls{nsdecs}
\gls{ldnkdmds} $Z$ 依赖于模型中的 $N$ 次抛掷硬币结果。
为了得到依赖于较少次抛掷硬币结果的相应的随机变量,
我们可以基于时刻 $n<N$ 的信息,
$Z$ 进行估计。
\begin{definition}[二叉树模型中的拉东--尼柯迪姆导数过程]\label{def:bookI_3_2_4}
\footnote{此定义即书 \cite{shreve_stochastic_2004} 中的定义 3.2.4.}
考虑 \gls{nsdecs}中的随机变量 $X$
定义:
\begin{equation}\label{eq:bookI_3_2_4}
Z_n = \mathbb{E}_n[Z],
\quad n = 0, 1, \ldots, N,
\end{equation}
则称 $Z_n, n = 0, 1, \ldots, N$
{\bf \gls{ldnkdmdsgc} (\gls{radon_nikodym_derivative_process})}
特别地,
$Z_N = Z$, $Z_0 = 1$.
\end{definition}
\begin{definition}[拉东--尼柯迪姆导数过程]\label{def:radon_nikodym_derivative_process}
% \footnote{此定义即书 \cite{shreve_stochastic_2004} 中的定义 3.2.4.}
$(\Omega, \mathcal{F}, \mathbb{P})$ 是概率空间,
$\mathcal{F}, 0\le t\le T$ 是域流,
其中 $T$ 是给定的终端时刻。
假定随机变量 $Z$ 几乎必然为正,
并且满足 $\mathbb{E}[Z] = 1$,
$\widetilde{\mathbb{P}}$ 由式~\ref{eq:bookII_5_2_1} 给出。
定义{\bf \gls{ldnkdmdsgc} (\gls{radon_nikodym_derivative_process})}为:
\begin{equation}\label{eq:radon_nikodym_derivative_process}
Z_t = \mathbb{E}[Z|\mathcal{F}],
\quad 0\le t\le T.
\end{equation}
\end{definition}
\begin{lemma}[二叉树模型中测度期望值的关系---仅依赖于前 $n$ 次抛掷结果]
\label{lem:bookI_3_2_5}
\footnote{此引理即书 \cite{shreve_stochastic_2004} 中的引理 3.2.5.}
假设条件如定义 \ref{def:bookI_3_2_4},
如果随机变量 $Y$ 仅依赖于前 $n$
次抛掷结果,
$n = 1, 2, \ldots, N$,
则:
\begin{equation}\label{eq:book_I_3_2_5}
\widetilde{\mathbb{E}}\left[Y\right] =
\mathbb{E}[Z_nY].
\end{equation}
\end{lemma}
\begin{lemma}[测度期望值的关系]
\label{lem:bookII_5_2_1}
\footnote{此引理即书 \cite{shreve_stochastic_2008} 中的引理 5.2.1.}
给定 $0\le t\le T$,
$Y$$\mathcal{Y}$$\mathcal{F}(t)$-可测随机变量,
则:
\begin{equation}\label{eq:book_II_5_2_8}
\widetilde{\mathbb{E}}\left[Y\right] =
\mathbb{E}[YZ(t)].
\end{equation}
\end{lemma}
% \vspace{1cm}
\begin{lemma}[测度条件期望值的关系]\label{lem:bookI_3_2_6}
\footnote{此引理即书 \cite{shreve_stochastic_2004} 中的引理 3.2.6.}
假设条件如定义 \ref{def:bookI_3_2_4},
如果随机变量 $Y$ 仅依赖于前 $m$ 次抛掷结果,
$0 \leq n \leq m \leq N$,
则:
\begin{equation}\label{eq:bookI_3_2_6}
\widetilde{\mathbb{E}}_n\left[Y\right] =
\frac{1}{Z_n}\mathbb{E}_n[Z_mY].
\end{equation}
\end{lemma}
\teach{
To prove the lemma,
it suffices to prove
\begin{equation*}
\widetilde{\mathbb{E}}_n\left[Y\right] =
\mathbb{E}_n\left[\frac{Z_m}{Z_n}Y\right].
\end{equation*}
}
\begin{lemma}[测度条件期望值的关系---仅依赖于 $\mathcal{F}(t)$]
\label{lem:bookII_5_2_2}
\footnote{此引理即书 \cite{shreve_stochastic_2008} 中的引理 5.2.2.}
给定 $0\le s\le t\le T$,
$Y$$\mathcal{Y}$$\mathcal{F}(t)$-可测随机变量,
\begin{equation}\label{eq:book_II_5_2_9}
\widetilde{\mathbb{E}}\left[Y | \mathcal{F}(s)\right] =
\frac{1}{Z(s)}\mathbb{E}[YZ(t)| \mathcal{F}(s)].
\end{equation}
\end{lemma}
\begin{theorem}[哥萨诺夫定理,一维情形]\label{thm:bookII_5_2_3}
$W(t), 0\le t\le T$
概率空间 $(\Omega, \mathcal{F}, \mathbb{P})$
上的布朗运动,
$\mathcal{F}(t), 0\le t \le T$ 是关于该布朗运动的域流。
$\varTheta, 0\le t\le T$ 是一个适应过程。
定义:
\begin{equation}\label{eq:bookII_5_2_11}
Z(t) = \exp \left\{
-\int_0^t\varTheta(u) \mathop{}\!\mathrm{d}W(u)
- \frac{1}{2}\int_0^t \varTheta^2(u) \mathop{}\!\mathrm{d}u\right\},
\end{equation}
\begin{equation}\label{eq:bookII_5_2_12}
\widetilde{W}(t) = W(t) + \int_0^t \varTheta(u) \mathop{}\!\mathrm{d}du.
\end{equation}
并且假设
\begin{equation}\label{eq:bookII_5_2_13}
\mathbb{E}\left[ \int_0^T \varTheta^2(u) Z^2(u)\right] < \infty.
\end{equation}
$Z = Z(T)$,
$\mathbb{E}[Z] = 1$,
并且在式~\eqref{eq:bookII_5_2_1} 给出的概率测度 $\widetilde{\mathbb{P}}$ 下,
过程 $\widetilde{W}(t), 0\le t\le T$ 是一个布朗运动。
\end{theorem}
\begin{definition}[状态价格密度过程]\label{def:bookI_3_2_7_0}
考虑 \gls{nsdecs}
假设条件如定义 \ref{def:bookI_3_1_3_state_price},
定义:
\begin{equation}\label{eq:bookI_3_2_7_0}
\zeta_n = \frac{Z_n}{(1+r)^n} ,
\quad n = 0, 1, \ldots, N,
\end{equation}
则称 $\zeta_n, n = 0, 1, \ldots, N$
{\bf \gls{ztjgmdgc} (\gls{state_price_density_process})}
\end{definition}
\begin{theorem}[利用状态密度价格过程计算衍生证券的价格]\label{thm:bookI_3_2_7}
\footnote{此定理即书 \cite{shreve_stochastic_2004} 中的定理 3.2.7.}
考虑 \gls{nsdecs}
如定义 \ref{def:bookI_3_2_7_0},
我们让 $\zeta_n$, $n = 1, 2, \ldots, N$,
表示\gls{ztjgmdgc}
则:
\begin{equation}\label{eq:derivative_value_state_price_density_process}
V_n =
\frac{1}{\zeta_n}\mathbb{E}_n\left[ \zeta_N V_N \right],
\quad n = 0, 1, \ldots, N,
\end{equation}
\end{theorem}
\section{资本资产定价模型}%
\label{sec:capital_asset_pricing_model}
\begin{definition}[效用函数]\label{def:utility_function}
我们定义
{\bf \gls{xyhs} (\gls{utility_function})}
为实数集上的一个非减凹函数。
\end{definition}
% \vspace{3cm}
\begin{problem}[(最优投资)]\label{pr:optimal_investment}
给定 $X_0$,
求自适应的资产组合过程
$\varDelta_n$, $n = 0, 1, \ldots, N-1$,
使得在
\gls{cffc} (\gls{wealth_equation})
\begin{equation}%
\label{eq:wealth_equation_3}
X_{n+1} = \varDelta_{n}S_{n+1} + (1 + r)(X_n - \varDelta_n S_n)
\end{equation}
约束下,
最大化
\begin{equation}\label{eq:optimal_investment_objective}
\mathbb{E}\left[ U(X_N) \right].
\end{equation}
\end{problem}
% \begin{solution}\label{sol:optimal_investment}
%
% \end{solution}
考虑图 \ref{fig:compute_rn} 中的三时段模型。
样本空间为:\[
\Omega = \{HHH, HHT, HTH, HTT, THH, THT, TTH, TTT\}.
\]
假设 $p = 2 / 3$ 为出现正面的真实概率,
$\tilde{p} = 1 / 2$ 为出现正面的\gls{fxzxglcd}
请计算\gls{ldnkdmds}
\begin{figure}[htpb]
\centering
\includegraphics[width=0.9\textwidth]{image/BT-1-opt.pdf}
\caption{单时段最优投资问题}
\label{fig:opt_investment_1_period}
\end{figure}
\begin{problem}[(最优投资---单时段问题)]\label{pr:opt_investment_1_period}
考虑图 \ref{fig:opt_investment_1_period} 中的单时段模型。
样本空间为:\[
\Omega = \{H, T\}.
\]
假设利率 $r = 1 / 4$, \gls{ssyz} $u = 2$, \gls{xjyz} $d = 1 / 2$,
投资者的初始财富 $X_0 = 4$.
假设
$p = 2 / 3$$q = 1 / 3$
分别为抛掷硬币出现正面(股价上升)和背面(股价下跌)的真实概率。
求最优的投资决策 $\varDelta_0$,
使得在
\begin{equation}%
\label{eq:wealth_equation_1_period}
X_1 = \varDelta_0S_1 + (1 + r)(X_0 - \varDelta_0 S_0)
\end{equation}
约束下,
最大化
\begin{equation}\label{eq:optimal_investment_objective_1_period}
\mathbb{E}\left[ \ln(X_1) \right].
\end{equation}
\end{problem}
% \begin{solution}\label{sol:opt_investment_1_period}
%
% \end{solution}
\begin{figure}[htpb]
\centering
\includegraphics[width=0.9\textwidth]{image/BT-2-opt.pdf}
\caption{二时段最优投资问题}
\label{fig:opt_investment_2_period}
\end{figure}
\begin{problem}[(最优投资---二时段问题)]\label{pr:opt_investment_2_period}
考虑图 \ref{fig:opt_investment_2_period} 中的二时段模型。
样本空间为:\[
\Omega = \{HH, HT, TH, TT\}.
\]
假设利率 $r = 1 / 4$, \gls{ssyz} $u = 2$, \gls{xjyz} $d = 1 / 2$,
投资者的初始财富 $X_0 = 4$.
假设
$p = 2 / 3$$q = 1 / 3$
分别为抛掷硬币出现正面(股价上升)和背面(股价下跌)的真实概率。
求最优的投资决策 \[
(\varDelta_0, \varDelta_1(H), \varDelta_1(T)),
\]
使得在
\begin{equation}%
\label{eq:wealth_equation_2_period}
X_{n+1} = \varDelta_{n}S_{n+1} + (1 + r)(X_n - \varDelta_n S_n),
\quad n = 0, 1,
\end{equation}
约束下,
最大化
\begin{equation}\label{eq:optimal_investment_objective_2_period}
\mathbb{E}\left[ \ln(X_2) \right].
\end{equation}
\end{problem}
% \begin{solution}\label{sol:opt_investment_1_period}
%
% \end{solution}
\teach{
在单时段问题中求解出的最优 $\varDelta_0$
在二时段问题中求解出的最优 $\varDelta_0$
相比,
有何不同?
为什么?
}
\begin{problem}
[(最优投资---不涉及资产组合过程)]\label{pr:optimal_investment_2}
给定 $X_0$,
求随机变量 $X_N$(并不涉及资产组合过程),
使得在
\begin{equation}\label{eq:constant_wealth_0_N}
\widetilde{\mathbb{E}}\left[ \frac{X_N}{(1+r)^N} \right]
=
X_0,
\end{equation}
约束下,
最大化
\begin{equation}\label{eq:optimal_investment_objective_2}
\mathbb{E}\left[ U(X_N) \right].
\end{equation}
\end{problem}
% \begin{solution}\label{sol:optimal_investment_2}
%
% \end{solution}
\begin{lemma}[两个问题解的相互关系]\label{lem:bookI_3_3_4}
\footnote{此引理即书 \cite{shreve_stochastic_2004} 中的引理 3.3.4.}
假设
$\varDelta_n^*$, $n = 0, 1, \ldots, N-1$,
是问题 \ref{pr:optimal_investment}
的一个最优资产组合过程,
$X_N^*$ 是相应最优的时刻 $N$ 的财富随机变量,
$X_N^*$ 是问题 \ref{pr:optimal_investment_2}
的最优解。
反之,
假设 $X_N^*$ 是问题 \ref{pr:optimal_investment_2} 的最优解,
则存在一个资产组合过程,
使得相应的初始值为 $X_0$ 的财富过程在时刻 $N$ 的值为 $X_N^*$,
并且这一资产组合过程是
问题 \ref{pr:optimal_investment}
的最优解。
\end{lemma}
% \vspace{1cm}
根据引理 \ref{lem:bookI_3_3_4},
我们可以将最优投资问题分解为易于操作的两个步骤:
\begin{enumerate}
\item 求解问题 \ref{pr:optimal_investment_2},
得到最优投资决策下得到的期末财富,记做 $X_N^*$.
\item 利用定理 \ref{thm:bookI_1_2_2} 中的算法
构建一个能够复制 $X_N^*$ 的资产组合过程。
\end{enumerate}
问题 \ref{pr:optimal_investment_2}
的陈述中同时出现真实概率测度
\gls{fxzxglcd}
使问题略显复杂。
我们可以引入\gls{ldnkdmds}
使其不涉及\gls{fxzxglcd}
在问题 \ref{pr:optimal_investment_2} 中,
只有限制条件
\eqref{eq:constant_wealth_0_N}
涉及\gls{fxzxglcd}
其等价于
\begin{equation}\label{eq:constant_wealth_0_N_P}
\mathbb{E}\left[ \zeta X_N \right]
=
X_0.
\end{equation}
基于此,
问题 \ref{pr:optimal_investment_2}
可以重新描述为下面的问题。
\begin{problem}
[(最优投资---不涉资产组合过程且仅含真实测度)]
\label{pr:optimal_investment_3}
给定 $X_0$,
求随机变量 $X_N$(并不涉及资产组合过程),
使得在
\begin{equation}\label{eq:constant_wealth_0_N_P_2}
\mathbb{E}\left[ \zeta X_N \right]
=
X_0
\end{equation}
约束下,
最大化
\begin{equation}\label{eq:optimal_investment_objective_3}
\mathbb{E}\left[ U(X_N) \right].
\end{equation}
\end{problem}
% \begin{solution}\label{sol:optimal_investment_2}
%
% \end{solution}
% 2020-10-09 06:47
$N$-时段模型中,
共有 $M = 2^N$ 个可能的抛掷硬币结果序列,
标记为:
\begin{equation}\label{eq:sequence_of_coin_toss}
\omega^1, \omega^2, \ldots, \omega^M.
\end{equation}
我们用上标来表明 $\omega^m$ 是整个抛掷硬币结果序列,
而不是某个序列中的第 $m$ 次抛掷结果。
定义
\begin{equation}\label{eq:sequence_of_coin_toss_notations}
\zeta_m \equiv \zeta\left(\omega^m\right),
p_m \equiv \mathbb{P}\left(\omega^m\right),
x_m \equiv X_N\left(\omega^m\right),
\end{equation}
则问题 \ref{pr:optimal_investment_3} 可被重述如下:
\begin{problem}
[(最优投资)]
\label{pr:optimal_investment_4}
给定 $X_0$,
求向量 $(x_1, x_2, \ldots, x_M)$,
使得在
\begin{equation}\label{eq:constant_wealth_0_M}
\sum_{m=1}^{M} p_m x_m \zeta_m
=
X_0
\end{equation}
约束下,
最大化
\begin{equation}\label{eq:optimal_investment_objective_4}
\sum_{m=1}^{M} p_m U(x_m).
\end{equation}
\end{problem}
% 2020-10-09 07:03
\begin{solution}\label{sol:pr:optimal_investment_4}
问题 \ref{pr:optimal_investment_4} 的拉格朗日函数为:
\begin{equation}\label{eq:lagrange_function}
L =
\sum_{m=1}^{M} p_m U(x_m)
-
\lambda\left(\sum_{m=1}^{M} p_m x_m \zeta_m - X_0\right).
\end{equation}
拉格朗日乘子方程为:
\begin{equation}\label{eq:lagrange_multiplier_equations}
\frac{\partial L}{\partial x_m} =
p_m U^{\prime}(x_m) - \lambda p_m \zeta_m =
0,
\quad m = 1, 2, \ldots, M.
\end{equation}
回顾 $x_m$$\zeta_m$ 的定义,
我们可将其重写为:
\begin{equation}\label{eq:lagrange_multiplier_equations_2}
U^{\prime}(X_N) = \frac{\lambda Z}{(1+r)^N}.
\end{equation}
由于函数 $U$ 为严格凹函数,
其反函数存在,
记为 $I$.
由式子 \eqref{eq:lagrange_multiplier_equations_2},
\begin{equation}\label{eq:inverse_function}
X_N = I \left( \frac{\lambda Z}{(1+r)^N}\right).
\end{equation}
这就给出了通过乘子 $\lambda$ 表示的最优解 $X_N$ 的公式。
为求乘子 $ \lambda$,
可将 $X_N$ 代入式 \eqref{eq:constant_wealth_0_N_P_2}:
\begin{equation}\label{eq:constant_wealth_0_N_P_3}
\mathbb{E}\left[
\frac{Z}{(1+r)^N} I \left( \frac{\lambda Z}{(1+r)^N}\right)
\right]
= X_0.
\end{equation}
解此方程,
求出 $\lambda$,
代入式子 \eqref{eq:lagrange_multiplier_equations_2},
得到 $X_N$,
再利用定理 \ref{thm:bookI_1_2_2} 中的算法,
确定最优资产组合过程
$\varDelta_n$, $n = 0, 1, \ldots, N-1$.
以及相应的资产组合价值过程
$X_n$, $n = 0, 1, \ldots, N$.
\end{solution}
\begin{theorem}[最优投资]\label{thm:bookI_3_3_6}
问题 \ref{pr:optimal_investment}
可以如下求解:
先解关于 $\lambda$ 的方程 \eqref{eq:constant_wealth_0_N_P_3},
再由式 \eqref{eq:inverse_function}
计算 $X_N$,
最后利用定理 \ref{thm:bookI_1_2_2} 中的算法,
确定最优资产组合过程
$\varDelta_n$, $n = 0, 1, \ldots, N-1$.
以及相应的资产组合价值过程
$X_n$, $n = 0, 1, \ldots, N$.
\end{theorem}
\begin{problem}[(单时段最优投资问题的另一解法)]
\label{pr:opt_investment_1_period_lagrange}
用定理 \ref{thm:bookI_3_3_6} 中的方法求解
问题~\ref{pr:opt_investment_1_period}.
\end{problem}
% \begin{solution}\label{sol:pr:opt_investment_1_period_lagrange}
%
% \end{solution}
\begin{problem}[(二时段最优投资问题的另一解法)]
\label{pr:opt_investment_2_period_lagrange}
用定理 \ref{thm:bookI_3_3_6} 中的方法求解
问题~\ref{pr:opt_investment_2_period}.
\end{problem}
% \begin{solution}\label{sol:pr:opt_investment_2_period_lagrange}
%
% \end{solution}
% \subsection*{答案}%
% \label{sub:solution_03}
\chapter{美式衍生证券}%
\label{cha:america_derivative}
\section{非路径依赖美式衍生产品}%
\label{sec:non_path_dependent_american_derivative}
这一节中我们主要研究美式看跌期权。
因为美式看跌期权的支付仅依赖于行权时刻的股票价格,
而不依赖于股价路径,
所以美式看跌期权是一个典型的{\bf 非路径依赖 (Non-path-dependent)}美式衍生产品。
首先,我们回顾欧式期权。
欧式期权合约中指定了到期日,
而期权一旦被行权的话,
也只能在到期日实施。
\vspace{1cm}
\begin{figure}[htpb]
\centering
\includegraphics[width=0.9\textwidth]{image/BT-2-EP.pdf}
\caption{二时段欧式看跌期权 (问题 \ref{pr:EP_2_period})}
\label{fig:pr:EP_2_period}
\end{figure}
\begin{problem}[
% \hyperlink{target:pr:EP_2_period}{前往答案}
% \faHandPointerO
(二时段欧式看跌期权
)]\label{pr:EP_2_period}
如图 \ref{fig:pr:EP_2_period},
假设利率 $r = 1 / 4$,
期初股价 $S_0 = 4$,
\gls{ssyz} $u = 2$, \gls{xjyz} $d = 1 / 2$,
\gls{oskdqq}\gls{qdjg} $K = 5$.
在定理 \ref{thm:bookI_2_5_8} 中,
我们已经证明了
此期权在任何时刻 $n$, $n = 0, 1, 2$, 的价值(基于无套利定价)
$V_n$ 可以表示为该时刻股价的函数,
$V_n = v_n\left( S_n\right) $.
请计算
$v_n\left( S_n\right) $, $n = 0, 1, 2$.
\end{problem}
% \begin{hint}\label{hnt:pr:EP_2_period}
% \end{hint}
% \hypertarget{target:pr:EP_2_period}{}
\begin{solution}\label{sol:pr:EP_2_period}
% \noindent\faHandPointerO ~\hyperref[pr:EP_2_period]{返回问题~\ref{pr:EP_2_period}}
答案如图~\ref{fig:sol:pr:EP_2_period} 所示。
\begin{figure}[H]
\centering
\includegraphics[width=0.9\textwidth]{image/BT-2-EP-answer.pdf}
\caption{二时段欧式看跌期权在各时刻的价值}
\label{fig:sol:pr:EP_2_period}
\end{figure}
\end{solution}
欧式期权确定了一个固定的行权日期,
而美式期权允许在到期日前(包括到期日)的任何时期行权。
% \vspace{2cm}
\begin{definition}[美式期权]\label{def:american_derivative}
如果期权持有人可以在到期日之前的任何时刻(包括到期日)行权,
这样的期权被称为 {\bf \gls{msqq} (\gls{american_option})}.
\end{definition}
\begin{definition}[内在价值]\label{def:intrinsic_value}
对于\gls{msqq}
即时行权可以获得的支付为
{\bf \gls{nzjz} (\gls{intrinsic_value})}.
如果在某时刻的内在价值为负,
则称期权在此刻为
{\bf 虚值 (out of money)}
\end{definition}
% teaching 2020-10-19 06:30 问学生是否仍然有价值?
% \vspace{3cm}
\begin{figure}[htpb]
\centering
\includegraphics[width=0.9\textwidth]{image/BT-2-AP.pdf}
\caption{二时段美式看跌期权 (问题 \ref{pr:AP_2_period})}
\label{fig:pr:AP_2_period}
\end{figure}
% \pagebreak
\begin{problem}[
% \hyperlink{target:pr:AP_2_period}{前往答案}
% \faHandPointerO
(二时段美式看跌期权
)]\label{pr:AP_2_period}
如图 \ref{fig:pr:AP_2_period},
假设利率 $r = 1 / 4$,
期初股价 $S_0 = 4$,
\gls{ssyz} $u = 2$, \gls{xjyz} $d = 1 / 2$,
\gls{mskdqq}\gls{qdjg} $K = 5$.
\begin{enumerate}
\item 请计算此期权的\gls{nzjz}
$g\left( S_n\right)$, $n = 0, 1, 2$.
\item 在第一次抛掷硬币的结果为{\bf 正面}时,
期权的持有者是否应该行权?
\item 在第一次抛掷硬币的结果为{\bf 背面}时,
期权的持有者是否应该行权?
\item 请计算此期权的价值
$v_n\left( S_n\right)$, $n = 0, 1, 2$.
注意此价值只与当期的股价有关,
与之前的路径无关。
\end{enumerate}
\end{problem}
\begin{hint}\label{hnt:pr:AP_2_period}
美式看跌期权的价值
$v_n\left( S_n\right)$
应该取
\begin{equation}\label{eq:g_v_n}
\max \left\{
g\left( S_n\right),
\frac{1}{1+r}\widetilde{\mathbb{E}}_n\left[v_{n+1}(S_{n+1})\right]
\right\}.
\end{equation}
% 欧式看跌期权价值
% 中的最大值。
\end{hint}
% \hypertarget{target:pr:AP_2_period}{}
\begin{solution}
% (二时段美式看跌期权
% \faHandPointerO~\hyperref[pr:AP_2_period]{返回问题~\ref{pr:AP_2_period}}
% )
\label{sol:pr:AP_2_period}
\begin{enumerate}
\item 如图~\ref{fig:sol:pr:AP_2_period_1},
美式看跌期权的内在价值为 $g\left( S_n\right) $, $n = 0, 1, 2$.
\begin{figure}[htpb]
\centering
\includegraphics[width=0.9\textwidth]
{image/BT-2-AP-intrinsic-value.pdf}
\caption{美式看跌期权的内在价值}
\label{fig:sol:pr:AP_2_period_1}
\end{figure}
\item 如第一次抛掷硬币的结果为{\bf 正面}
期权的持有者不应该行权,
因为此时行权的内在价值 -3 小于
$\widetilde{\mathbb{E}}_1\left[ v_2 \right](H) = 0.4$.
\item 如第一次抛掷硬币的结果为{\bf 背面}
期权的持有者应该行权,
因为此时行权的内在价值 3 大于
$\widetilde{\mathbb{E}}_1\left[ v_2 \right](T) = 2$.
\item
\begin{figure}[htpb]
\centering
\includegraphics[width=0.9\textwidth]{image/BT-2-AP-answer.pdf}
\caption{美式看跌期权的价值}
\label{fig:sol:pr:AP_2_period}
\end{figure}
如图~\ref{fig:sol:pr:AP_2_period},
美式看跌期权的价值为
$v_n\left( S_n\right)$, $n = 0, 1, 2$.
\end{enumerate}
\end{solution}
% \pagebreak
\section{停时}%
\label{sec:stopping_time}
从问题 \ref{pr:AP_2_period} 中可以看到,
\gls{mskdqq}的最优实施时刻是一个随机变量。
我们引入停时的概念来刻划这个随机变量。
% \pagebreak % Comment out in normal model
\begin{definition}[停时]\label{def:stopping_time}
\gls{nsdecs}中,
{\bf \gls{ts} (\gls{stopping_time})}
$\tau$\gls{Nwpzybybkj} \gls{CN} 上的一个随机变量,
取值为 $0, 1, \ldots, N$ 或者 $\infty$,
并且满足条件:如果
\[
\tau(\omega_1\omega_2\ldots\omega_n\omega_{n+1}\ldots\omega_N) = n,
\]
那么对所有的 $\omega_{n+1}^{\prime}\ldots\omega_N^{\prime}$,
\[
\tau(\omega_1\omega_2\ldots\omega_n
\omega_{n+1}^{\prime}\ldots\omega_N^{\prime}) = n.
\]
\end{definition}
\begin{problem}[(用\gls{ts}表示\gls{mskdqq}的最优实施时刻)]
\label{pr:stopping_time_2_mskdqq}
将问题 \ref{pr:AP_2_period} 中的最优实施时刻写成行权法则,
并用\gls{ts}来表示。
\end{problem}
% \begin{hint}\label{hnt:pr:stopping_time_2_mskdqq}
% 对于每个 $\omega\in\mathbb{C}^2$,
% 计算最优实施时刻 $\tau(\omega)$.
% \end{hint}
% \noindent \faHandPointerO ~ \hyperlink{target:pr:stopping_time_2_mskdqq}{前往答案}
% \noindent\faHandPointerO ~\hyperref[pr:stopping_time_2_mskdqq]{返回问题~\ref{pr:stopping_time_2_mskdqq}}
% \hypertarget{target:pr:stopping_time_2_mskdqq}{}答案
\begin{solution}\label{sol:pr:stopping_time_2_mskdqq}
最优实施时刻如图 \ref{fig:pr:stopping_time_2_mskdqq} 所示。
\begin{figure}[H]
\centering
\includegraphics[width=0.9\textwidth]{image/BT-2-AP-stopping-time.pdf}
\caption{行权法则 $\tau$}
\label{fig:pr:stopping_time_2_mskdqq}
\end{figure}
\end{solution}
% \pagebreak % Comment out in normal model
只要有一个随机过程和一个停时,
我们就能定义一个停止过程。
\begin{definition}[停止过程]\label{def:stopped_process}
给定一个随机过程,
如果它在规定的(可能是随机的,比如\gls{ts})时间之后被强制假定为相同的值,
则称这样的过程为
{\bf \gls{tzgc} (\gls{stopped_process})}.
\end{definition}
在问题~\ref{pr:AP_2_period}的第 4 小问中,
我们算出了\gls{mskdqq}的价值
$v_n\left( S_n\right)$, $n = 0, 1, 2$.
这个过程的贴现过程
如图~\ref{fig:sol:pr:AP_2_period_discounted} 所示。
在问题 \ref{pr:stopping_time_2_mskdqq} 中求出的行权法则 $\tau$
如图 \ref{fig:pr:stopping_time_2_mskdqq} 所示。
\begin{figure}[htpb]
\centering
\includegraphics[width=0.9\textwidth]{image/BT-2-AP-answer-discounted.pdf}
\caption{美式看跌期权的贴现价值过程}
\label{fig:sol:pr:AP_2_period_discounted}
\end{figure}
% \pagebreak
\begin{problem}[
% \hyperlink{target:pr:stopping_process}{前往答案}
% \faHandPointerO
(根据一个过程和停时构造停止过程
)]\label{pr:stopped_process}
根据图~\ref{fig:sol:pr:AP_2_period_discounted}
中的贴现过程
\begin{equation}\label{eq:stopped_process}
Y_n = \frac{1}{(1+r)^n}v_n\left( S_n\right), \quad n = 0, 1, 2,
\end{equation}
以及图 \ref{fig:pr:stopping_time_2_mskdqq}中的行权法则 $\tau$,
构造一个停止过程。
\end{problem}
% \begin{hint}\label{hnt:pr:stopped_process}
% 根据定义 \ref{def:stopped_process} 构造这个停止过程。
% \end{hint}
% \hypertarget{target:pr:stopping_process}{}
\begin{solution}\label{sol:pr:stopped_process}
% (根据一个随机过程和停时构造停止过程
% \faHandPointerO ~\hyperref[pr:stopped_process]{返回问题~\ref{pr:stopped_process}}
% )
构造的停止过程如图 \ref{fig:sol:pr:stopped_process} 所示。
\begin{figure}[H]
\centering
\includegraphics[width=0.9\textwidth]{image/BT-2-AP-stopped-process.pdf}
\caption{停止过程(问题 \ref{pr:stopped_process} 示意图)}
\label{fig:sol:pr:stopped_process}
\end{figure}
\end{solution}
% \pagebreak % Comment out in normal model
\begin{theorem}[可选抽样---第一定理]\label{thm:bookI_4_3_2}
\footnote{此定理即书 \cite{shreve_stochastic_2004} 中的定理 4.3.2.}
将一个鞅停止于停时得到的停止过程仍是一个鞅。
将一个上鞅(或下鞅)停止于停时得到的停止过程仍是一个上鞅(或下鞅)。
\end{theorem}
\begin{theorem}[可选抽样---第二定理]\label{thm:bookI_4_3_3}
\footnote{此定理即书 \cite{shreve_stochastic_2004} 中的定理 4.3.3.}
$X_n$, $n = 1, 2, \ldots, N$,
是一个下鞅,
$ \tau$ 是一个停时,
$ \mathbb{E}[X_{n\land\tau}]
\leq
\mathbb{E}[X_n]$;
如果 $X_n$ 是一个上鞅,
$ \mathbb{E}[X_{n\land\tau}]
\geq
\mathbb{E}[X_{n}]$;
如果 $X_n$ 是一个鞅,
$ \mathbb{E}[X_{n\land\tau}]
=
\mathbb{E}[X_{n}]$.
\end{theorem}
% \pagebreak
\section{一般美式衍生产品}%
\label{sec:general_american_derivative}
\ref{sec:non_path_dependent_american_derivative} 中,
我们学习了非路径依赖的美式衍生产品。
在本节中,
我们学习内在价值可能路径依赖的美式衍生证券。
我们的讨论限于\gls{nsdecs}的框架,
其中\gls{ssyz} $u$, 下降因子 $d$, 和利率 $r$ 满足
限制条件~\eqref{eq:dru_constraint}.
在这样的模型中,
定义 $\varphi_n$ 为取值在\[
\{n, n+1, \ldots, N, \infty\}
\]
中的所有停时 $ \tau$ 的集合。
\begin{definition}[美式衍生证券价格过程]\label{def:AS_pricing_formula}
对每一个 $n$, $n = 1, 2, \ldots, N$,
$G_n$ 是依赖于前 $n$ 次抛掷硬币结果的随机变量。
具有内在价值过程 $G_n$ 的美式衍生证券
可以在时刻 $N$ 之前的任何时刻(包括时刻 $N$ )被实施,
并且如果在时刻 $n$ 被实施,
支付为 $G_n$.
对每一个 $n$, $n = 1, 2, \ldots, N$,
我们通过以下美式风险中性定价公式来定义这一衍生证券的价格过程:
\begin{equation}\label{eq:AS_pricing_formula}
V_n = \max_{\tau\in\varphi_n}\widetilde{\mathbb{E}}_n
\left[
\mathbb{I}_{\tau\leq N} \frac{1}{(1+r)^{\tau-n}}G_{\tau}
\right].
\end{equation}
\end{definition}
% \pagebreak
在由定义 \ref{def:AS_pricing_formula}
推得其它结论之前,
我们回顾问题 \ref{pr:AP_2_period},
验证此定义与该例中得到的美式看跌期权的价格是一致的。
我们在图~\ref{fig:sol:pr:AP_2_period_1}
已经计算了美式看跌期权在各个时刻的内在价值。
修改一下符号,
我们可以写出内在价值过程
$G_n$, $n = 0, 1, 2$.
(图~\ref{fig:image-BT-2-AP-intrinsic-value-process})。
\begin{figure}[htpb]
\centering
\includegraphics[width=0.9\textwidth]
{image/BT-2-AP-intrinsic-value-process.pdf}
\caption{美式看跌期权的内在价值过程}
\label{fig:image-BT-2-AP-intrinsic-value-process}
\end{figure}
基于此过程,应用式~\eqref{eq:AS_pricing_formula},
我们可以算出
$V_n$, $n = 0, 1, 2$,
(图~\ref{fig:image-BT-2-AP-value-process})。
\begin{figure}[htpb]
\centering
\includegraphics[width=0.9\textwidth]{image/BT-2-AP-value-process.pdf}
\caption{美式看跌期权的价格过程}
\label{fig:image-BT-2-AP-value-process}
\end{figure}
\teach{
通过式~\eqref{eq:AS_pricing_formula} 计算 $V_n$ 是很复杂的,
有没有简便的方法?
答:书~\cite{shreve_stochastic_2004} 定理 4.4.3.
在学习定理 4.4.3 之前,
我们先证明式~\eqref{eq:AS_pricing_formula} 是合理的。
}
% \pagebreak
\begin{theorem}[美式衍生证券价格过程的性质]\label{thm:bookI_4_4_2}
\footnote{此定理即书 \cite{shreve_stochastic_2004} 中的定理 4.4.2.}
由定义 \ref{def:AS_pricing_formula}
给出的美式衍生证券价格过程具有如下性质:
\begin{enumerate}[label=\roman*)]
\item 对所有的 $n$, $V_n\geq \max\{G_n, 0\}$;
\item 贴现过程 $\frac{1}{(1+r)^n}V_n$ 是一个上鞅;
\item 如果另有过程 $Y_n$ 满足:
对所有的 $n$,
\[
Y_n\geq \max\{G_n, 0\}
\]
$\frac{1}{(1+r)^n}Y_n$ 是一个上鞅,
则对所有的 $n$,
$Y_n\geq V_n$.
\end{enumerate}
\end{theorem}
% \pagebreak % Comment out in normal model
从衍生证券的出售者角度看,
性质 (i) 确保了其可以成功地对冲衍生证券的空头,
无论衍生证券持有人何时行权,
对冲资产组合的价值足以支付该衍生证券。
性质 (ii) 确保了投资者能以初始资本 $V_0$ 构建对冲投资组合,
% next 参见定理。。。
使得在任何时刻 $n$,
资产组合的价值为 $V_n$.
所以性质 (i) 和 (ii) 保证了式 \eqref{eq:AS_pricing_formula}
中的价格 $V_n$ 可以被衍生证券出售者接受。
性质 (iii) 则表明,
为确保出售者能接受,
\eqref{eq:AS_pricing_formula}
中的价格 $V_n$ 是最低的。
因此性质 (iii) 保证了价格 $V_n$ 对于衍生证券的购买者是公平的。
% \pagebreak
\begin{theorem}[美式衍生证券定价算法]\label{thm:bookI_4_4_3}
\footnote{此定理即书 \cite{shreve_stochastic_2004} 中的定理 4.4.3.}
对于定义~\ref{def:AS_pricing_formula}
中给出的路径依赖的衍生证券价格过程,
我们有如下美式定价算法:
\begin{equation}\label{eq:AP_price_N}
V_N(\omega_1\ldots\omega_N) =
\max\{G_N(\omega_1\ldots\omega_N), 0\},
\end{equation}
\begin{equation}\label{eq:AP_price_n}
V_n\left(\omega^{(n)}\right) =
\max \left\{
G_n\left(\omega^{(n)}\right),
\frac{1}{1+r}\widetilde{\mathbb{E}}_n
[V_{n+1}]\left(\omega^{(n)}\right)
\right\},
\end{equation}
$n = N-1, \ldots, 0$.
\end{theorem}
% \pagebreak % Comment out in normal model
\begin{theorem}[路径依赖美式衍生产品的复制]\label{thm:bookI_4_4_4}
\footnote{此定理即书 \cite{shreve_stochastic_2004} 中的定理 4.4.4.}
考虑 \gls{nsdecs}
对于任意 $n$, $n = 0, 1, \ldots, N$,
$G_n$ 是依赖于前 $n$ 次抛掷硬币结果的随机变量。
利用定义~\ref{def:AS_pricing_formula}
中给出的 $V_n$,
定义:
\begin{equation}\label{eq:AP_replicate_stock}
\varDelta_n\left(\omega^{(n)}\right) =
\frac{
V_{n+1}\left(\omega^{(n)}H\right)
-
V_{n+1}\left(\omega^{(n)}T\right)
}{
S_{n+1}\left(\omega^{(n)}H\right)
-
S_{n+1}\left(\omega^{(n)}T\right)
},
\end{equation}
\begin{equation}\label{eq:AP_replicate_consumption}
C_n\left(\omega^{(n)}\right)
=
V_n\left(\omega^{(n)}\right)
-
\frac{1}{1+r}\widetilde{\mathbb{E}}_n
[V_{n+1}]\left(\omega^{(n)}\right),
\end{equation}
其中 $n = 1, 2, \ldots, N$.
对于所有的 $n$,
我们有 $C_n \geq 0$.
若令 $X_0 = V_0$ 并且按时间前向递归定义在时刻 $n = 0, 1, \ldots, N-1$
的投资组合价值
\begin{equation}
X_{n+1} = \varDelta_{n}S_{n+1} + (1 + r)(X_n - C_n - \varDelta_n S_n),
\end{equation}
则对所有的 $n$ 和所有的 $\omega^{(n)}$, 我们有:
\begin{equation}
X_n\left( \omega^{(n)}\right)
=
V_n\left( \omega^{(n)}\right)
\mbox{并且 }
X_n \geq G_n.
\end{equation}
\end{theorem}
% \pagebreak
\begin{theorem}[最优实施]\label{thm:bookI_4_4_5}
\footnote{此定理即书 \cite{shreve_stochastic_2004} 中的定理 4.4.5.}
对于 $n=0$,
使得式~\ref{def:AS_pricing_formula} 的右端取最大值的停时为:
\begin{equation}\label{eq:AP_stopping_time}
\tau^* = \min\{n; V_n = G_n\}.
\end{equation}
即:
\begin{equation}\label{eq:AP_price_star}
V_n = \widetilde{\mathbb{E}}_n
\left[
\mathbb{I}_{\tau^*\leq N} \frac{1}{(1+r)^{\tau^*-n}}G_{\tau^*}
\right].
\end{equation}
\end{theorem}
\teach{
用一个例子验证这个定理。“机不可失,失不再来”。
}
\pagebreak
\section{美式看涨期权}%
\label{sec:AC_option}
\begin{problem}[
% \hyperlink{target:pr:AC_2_period}{前往答案}
% \faHandPointerO
(二时段美式看涨期权
)]\label{pr:AC_2_period}
如图 \ref{fig:pr:AC_2_period},
假设利率 $r = 1 / 4$,
期初股价 $S_0 = 4$,
\gls{ssyz} $u = 2$, \gls{xjyz} $d = 1 / 2$,
美式看涨期权的\gls{qdjg} $K = 5$.
\begin{figure}[htpb]
\centering
\includegraphics[width=0.9\textwidth]{image/BT-2-AC.pdf}
\caption{二时段美式看涨期权 (问题 \ref{pr:AC_2_period})}
\label{fig:pr:AC_2_period}
\end{figure}
\begin{enumerate}
\item 请计算此期权的\gls{nzjz}
$g\left( S_n\right)$, $n = 0, 1, 2$.
\item 在第一次抛掷硬币的结果为{\bf 正面}时,
期权的持有者是否应该行权?
\item 在第一次抛掷硬币的结果为{\bf 背面}时,
期权的持有者是否应该行权?
\item 请计算此期权的价值
$v_n\left( S_n\right)$, $n = 0, 1, 2$.
注意此价值只与当期的股价有关,
与之前的路径无关。
\end{enumerate}
\end{problem}
\begin{hint}\label{hnt:pr:AC_2_period}
美式看涨期权的价值
$v_n\left( S_n\right)$
应该取
\begin{equation}\label{eq:g_v_n_AC}
\max \left\{
g\left( S_n\right),
\frac{1}{1+r}\widetilde{\mathbb{E}}_n\left[v_{n+1}(S_{n+1})\right]
\right\}.
\end{equation}
% 欧式看涨期权价值
% 中的最大值。
\end{hint}
% \hypertarget{target:pr:AC_2_period}{}
\begin{solution}
% (二时段美式看涨期权
% \faHandPointerO~\hyperref[pr:AC_2_period]{返回问题~\ref{pr:AC_2_period}}
% )
\label{sol:pr:AC_2_period}
\begin{enumerate}
\item 如图~\ref{fig:sol:pr:AC_2_period_1},
美式看涨期权的内在价值为 $g\left( S_n\right) $, $n = 0, 1, 2$.
\begin{figure}[htpb]
\centering
\includegraphics[width=0.9\textwidth]
{image/BT-2-AC-intrinsic-value.pdf}
\caption{美式看涨期权的内在价值}
\label{fig:sol:pr:AC_2_period_1}
\end{figure}
\item 如第一次抛掷硬币的结果为{\bf 正面}
期权的持有者不应该行权,
因为此时行权的内在价值 3 小于
$\widetilde{\mathbb{E}}_1\left[ v_2 \right](H) = 4.4$.
\item 如第一次抛掷硬币的结果为{\bf 背面}
期权的持有者不应该行权,
因为此时行权的内在价值 -3 小于
$\widetilde{\mathbb{E}}_1\left[ v_2 \right](T) = 0$.
\item
\begin{figure}[htpb]
\centering
\includegraphics[width=0.9\textwidth]{image/BT-2-AC-answer.pdf}
\caption{美式看涨期权的价值}
\label{fig:sol:pr:AC_2_period}
\end{figure}
如图~\ref{fig:sol:pr:AC_2_period},
美式期权的价值为
$v_n\left( S_n\right)$, $n = 0, 1, 2$.
\end{enumerate}
\end{solution}
\begin{theorem}[美式看涨期权的价值与欧式看涨期权的价值相同]
\label{thm:bookI_4_5_1}
\footnote{此定理即书 \cite{shreve_stochastic_2004} 中的定理 4.5.1.}
考虑 \gls{nsdecs}
设美式衍生证券的支付为满足 $g(0) = 0$ 的凸函数 $g(s)$,
且该衍生证券在时刻 $0$ 的价值 (见定义~\ref{def:AS_pricing_formula}):
\begin{equation}\label{eq:AC_pricing_formula}
V_0^A = \max_{\tau\in\varphi_0}\widetilde{\mathbb{E}}
\left[
\mathbb{I}_{\tau\leq N} \frac{1}{(1+r)^{\tau}}g(S_{\tau})
\right]
\end{equation}
相同于在到期日 $N$ 支付为 $g(S_N)$ 的欧式衍生证券的价值:
\begin{equation}\label{eq:EC_pricing_formula}
V_0^E = \widetilde{\mathbb{E}}
\left[
\frac{1}{(1+r)^N}\max\{g(S_N), 0\}
\right].
\end{equation}
\end{theorem}
\teach{
提问:这个定理的证明的关键点是什么?
重点讲解 $\frac{1}{(1+r)^n}g^+(S_n)$ 为一个下鞅,即通过函数 $g^+$ 的传递,
鞅变成了下鞅。
提问:对这个结果和美式看跌期权结果不同的直观解释是什么?
}
\pagebreak
% \pagebreak
\subsection{永久美式看跌期权}%
\label{sub:perpetual_american_put}
这一节,
我们将给出 {\bf 永久美式看跌期权} 的一个特例的定价与对冲。
\begin{figure}[htpb]
\centering
\includegraphics[width=0.85\textwidth]{image/BT-infty-AP.pdf}
\caption{永久美式看跌期权 (问题 \ref{pr:AP_infty_period})}
\label{fig:pr:AP_infty_period}
\end{figure}
\begin{problem}[
% \hyperlink{target:pr:AP_infty_period}{前往答案}
% \faHandPointerO
(永久美式看跌期权
)]\label{pr:AP_infty_period}
如图 \ref{fig:pr:AP_infty_period},
假设利率 $r = 1 / 4$,
\gls{ssyz} $u = 2$, \gls{xjyz} $d = 1 / 2$,
永久\gls{mskdqq}\gls{qdjg} $K = 4$.
假定期初股价 $S_0 = 4$,
分析在以下三个策略下期权的价值。
\begin{enumerate}
\item 立即行权;
\item 在股价跌至 2 的第一时刻行权;
\item 在股价跌至 1 的第一时刻行权。
\end{enumerate}
\end{problem}
% \begin{hint}\label{hnt:pr:AP_infty_period}
% 美式看跌期权的价值
% $v_n\left( S_n\right)$
% 应该取
% \begin{equation}\label{eq:g_v_n_infty}
% \max \left\{
% g\left( S_n\right),
% \frac{1}{1+r}\widetilde{\mathbb{E}}_n\left[v_{n+1}(S_{n+1})\right]
% \right\}.
% \end{equation}
% % 欧式看跌期权价值
% % 中的最大值。
% \end{hint}
% \hypertarget{target:pr:AP_infty_period}{}
% \begin{solution}
% % (二时段美式看跌期权
% % \faHandPointerO~\hyperref[pr:AP_infty_period]{返回问题~\ref{pr:AP_infty_period}}
% % )
% \label{sol:pr:AP_infty_period}
% \begin{enumerate}
% \item
% \item
% \item
% \end{enumerate}
% \end{solution}
% \pagebreak
\begin{problem}[(计算永久美式看跌期权的价值)]\label{pr:AP_infty_value}
基于问题~\ref{pr:AP_infty_period} 的假设,
计算永久美式看跌期权的价值。
\end{problem}
% \begin{hint}\label{hnt:pr:AP_infty_value}
% \end{hint}
% \noindent\faHandPointerO ~\hyperlink{target:pr:AP_infty_value}{前往答案}
% \noindent\faHandPointerO ~\hyperref[pr:AP_infty_value]{返回问题~\ref{pr:AP_infty_value}}
% \hypertarget{target:pr:AP_infty_value}{}
% \begin{solution}\label{sol:pr:AP_infty_value}
%
% \end{solution}
我们只考虑形如 $2^j$ 的股价,
通过最优行权策略(只要股价低于或者等于 2 就行权)的猜测,
我们可以猜测期权价值函数为:
\begin{equation}\label{eq:value_function}
v(2^j) =
\begin{cases}
4 - 2^j, & \text{if } j\leq 1, \\
\frac{4}{2^j}, & \text{if } j\geq 1.
\end{cases}
\end{equation}
我们要证明其满足如下三条性质:
\begin{enumerate}[label=\roman*)]
\item 对所有的 $n$, $v(S_n) \geq (4 - S_n)^+$;
\item 贴现过程 $\frac{1}{(1+r)^n}v(S_n)$ 是一个上鞅;
\item $v(S_n)$ 是满足前两个性质的最小过程。
\end{enumerate}
% \pagebreak
对于 $s = 2^j$,
\eqref{eq:value_function} 可以写成
\begin{equation}\label{eq:value_function_s}
v(s) =
\begin{cases}
4 - s, & \text{if } s\leq 2, \\
\frac{4}{s}, & \text{if } s\geq 4.
\end{cases}
\end{equation}
如果 $v(s)$ 满足性质 i), ii), iii),
我们可以证明其满足下面三条性质:
\begin{enumerate}
\item 对所有的 $n$, $v(s) \geq (4 - s)^+$;
\item $v(s) \geq \frac{4}{5}\left[ \frac{1}{2}v(2s)
+ \frac{1}{2}v(s / 2) \right] $;
\item 对每一个形如 $s = 2^j$$s$, 以上两个等式中至少有一个成立。
\end{enumerate}
以上三个性质,可以归结为贝尔曼方程:
\begin{equation}\label{eq:bellman_equation}
v(s) = \max\left\{ 4-s, \frac{4}{5} \left[
\frac{1}{2}v(2s) + \frac{1}{2}v(\frac{s}{2})\right] \right\}
\end{equation}
% \pagebreak
\begin{problem}[(贝尔曼方程)]\label{pr:bellman_equation}
证明永久美式看跌期权的价值 $v(s)$ 必定满足贝尔曼方程。
举例证明满足贝尔曼方程的 $v(s)$ 不一定是永久美式看跌期权的价值。
\end{problem}
% \begin{hint}\label{hnt:pr:bellman_equation}
% \end{hint}
% \noindent\faHandPointerO ~\hyperlink[sol:pr:bellman_equation]{前往答案}
% \noindent\faHandPointerO ~\hyperref[pr:bellman_equation]{返回问题~\ref{pr:bellman_equation}}
% \hypertarget{sol:pr:bellman_equation}{}答案
% \begin{proof}\label{pf:pr:bellman_equation}
%
% \end{proof}
在由式 \eqref{eq:bellman_equation} 求解 $v(s)$ 时,我们可以用边界条件来排除无关的解。
比如,永久美式看跌期权的价值必须满足:
\begin{equation}\label{eq:terminal_condition}
\lim_{s \downarrow 0} v(s) = 4, \quad \lim_{s \to \infty} v(s) = 0.
\end{equation}
% \section*{答案}%
% \label{sec:answers_04}
% \vspace{3cm}
% \nocite{*}
\printbibliography
\printglossary[title={术语表}]
\end{document}
马建仓 AI 助手
尝试更多
代码解读
代码找茬
代码优化
1
https://gitee.com/sierxue/sc4f.git
git@gitee.com:sierxue/sc4f.git
sierxue
sc4f
sc4f
master

搜索帮助

D67c1975 1850385 1daf7b77 1850385